ORTHOPEDIC MCQS ADULT RECONSTRUCTIVE OB 20 1C

ORTHOPEDIC MCQS ONLINE RECON OB 20 1C

 

 

 

 

 

 

 

2020

 

  1. A 72-year-old female presents with progressive left thigh and knee pain for the last year. 5 years ago she sustained a femoral neck fracture treated with the implant seen in Figures A-C (current radiographs). The thigh pain is worse with weight-bearing. C-reactive

    protein and erythrocyte sedimentation levels are within defined limits. Which of the following is the most likely cause of her pain?

     

     

     

     

     

     

     

     

     

    1. Acetabular erosion

    2. Degenerative arthritis of the ipsilateral knee

    3. Abductor tear

    4. Spinal stenosis

    5. Aseptic loosening

     

    Corrent answer: 5

     

    Figures A-C show a cementless Austin-Moore hemiarthroplasty with pedestal formation at the distal aspect of the stem with a large radiolucent line in Gruen zones 5 and 6. This, along with her clinical presentation of thigh and knee pain, is most consistent with aseptic loosening.

     

    Aseptic loosening is a common reason for revision hip arthroplasty and should be considered in any patient who presents with groin, buttock or thigh pain after a hip replacement. In this situation, ESR and CRP values were within defined limits, making infection highly unlikely. Common reasons for aseptic loosening include poor initial fixation, osteolysis, and mechanical loss of fixation over time. Signs that are suggestive of aseptic loosening include progressive radiolucent lines, change of implant position on sequential radiographs, a positive bone scan, and pedestal formation at the distal aspect of a femoral stem. In this situation, there were no prior radiographs available, but her clinical presentation, pedestal formation, and large radiolucent line suggest motion of the prosthesis within the canal.

     

    Portillo et al. reviewed 112 patients undergoing revision surgery after hip,

    knee, shoulder and elbow arthroplasty. Aseptic loosening was the most common reason for revision. Revisions <2 years from the time of surgery were most commonly due to infection.

     

    Lombardi et al. reviewed 498 patients who underwent hip arthroplasty surgery (both primary and revision) with ultraporous acetabular components to determine if smoking is a risk factor for early failure. They found failure rates of 10% in both current and past smokers, compared to 3% in non-smokers.

     

    Illustrations A and B show the post-operative images with a tapered cone femoral prosthesis. Illustration C shows a closer view of the pedestal at the end of the uncemented femoral stem.

     

    Incorrect Answers:

    Answer 1: While it is certainly possible this patient has a degree of acetabular erosion (arthritis), her presentation and radiographs are more suggestive of aseptic loosening.

    Answer 2: In this situation, and without radiographs to support the diagnosis, aseptic loosening of the femoral component is a better answer.

    Answer 3: Abductor tears would more commonly cause a limb and weakness. Answer 4: Spinal stenosis should always be considered in patients with lower extremity pain, but given her radiographic findings, aseptic loosening is the better answer.

     

     

     

     

     

     

     

     

     

     

    2020

     

  2. An 86-year-old man complains of knee pain and swelling 10 years after total knee arthroplasty. On examination, he has no fever, and his knee is neither red nor warm. The clinical appearance of his knee is shown in Figures A and B. An immediate post-operative radiograph taken 10 years ago is shown in Figure C. A current radiograph is shown in Figure D. Knee aspiration reveals 700 WBC/mm3 with 55% neutrophils. What is the most likely diagnosis?

     

     

     

     

     

     

     

     

     

     

     

    1. Infection

    2. Osteolysis

    3. Fracture

    4. Implant loosening

    5. Patellofemoral overstuffing

     

    Corrent answer: 4

    This patient has patellar component loosening with migration of the patellar component to a subcutaneous position via a capsular defect.

     

    Patellar resurfacing during TKA is a controversial issue. The rate of patella component loosening is 0.6-4.8% of cases. The etiology includes: (1) maltracking (from malalignment) or overstuffing the patellofemoral joint (PFJ) leading to high shear, displacing the patellar component through the capsule, and (2) subclinical infection. Other risks include metal-backed patella, obesity, lateral release, joint-line elevation, postoperative flexion >100°, poor bone stock, asymmetric patella resection, inadequate implant fixation (small pegs), secondary osteonecrosis and osteolysis.

     

    Schroer et al. assessed the etiology of TKA revision in 6 arthroplasty centers retrospectively. They determined that aseptic loosening (31%) was the most common cause of failure (instability 19%, infection 16%, polyethylene wear 10%, arthrofibrosis 7%, and malalignment 7%). Mean time to failure was 6 years. Polyethylene wear is less common because of improved implant design and sterilization techniques.

     

    Pilling et al. performed a meta-analysis on patella resurfacing during 3465 knee replacements in 16 randomized controlled trials. There were no differences in infection rates and operative time. They concluded that postoperative satisfaction and anterior knee pain rates in both resurfaced and non-resurfaced groups were not different, but the non-resurfaced groups had more additional surgical procedures.

     

    Jacobs et al. report on a case of recurrent patellar component loosening (twice) with no apparent cause. There was no maltracking, bone necrosis, or tilting preoperatively. They eventually removed the patellar button.

     

    Figure A shows a subcutaneous prominence, suggestive of patellar component loosening and migration with associated fluid collection or bursa. Figure B shows the component being pinched between the examiners fingers with the edge readily visible. Figure C shows an intact patellar component on the lateral view, suggested by the radiolucent space in the PFJ. Figure D shows the patellar component superficial to the patella, and collapse of the radiolucent space in the PFJ with patella bone-femoral component contact.

     

    Incorrect Answers:

    Answer 1: The clinical presentation and arthrocentesis results do not suggest infection.

    Answer 2: There are signs of osteolysis around the femoral and tibial components. While osteolysis may have played a role, the main diagnosis is

    loosening and dislocation of the patellar component. Answer 3: There are no signs of periprosthetic fracture.

    Answer 5: While overstuffing is a possible cause for loosening and dislocation (increased patellofemoral joint reaction force, leading to high shear at high flexion angles), the main diagnosis is loosening of the patella component.

     

     

     

     

     

     

    2020

     

  3. A 62-year-old male undergoes a routine total knee arthroplasty with a cruciate-retaining prosthesis. Six months after the operation, the patient feels and hears a pop while working with physical therapy. On physical exam, he has a small but palpable defect in his quadriceps tendon. He is able to maintain active, although weak, extension of the knee from a flexed position. The patient is treated non-operatively with a knee-immobilizer for 6 weeks. Which of the following is the most likely outcome?

    1. Minor extension lag with significantly decreased flexion

    2. Minor extension lag with acceptable flexion

    3. Rerupture of the quadriceps tendon after resumption of physical therapy

    4. Acceptable flexion with an extension lag of 20 degrees

    5. Early failure of the total knee prosthesis requiring revision Corrent answer: 2

    Excellent results have been reported with the non-operative treatment of partial quadriceps tendon ruptures following total knee arthroplasty.

    Quadriceps tendon injuries are a rare but devastating complication following total knee arthroplasty. Complete ruptures treated operatively have a high complication rate secondary to predisposing systemic or local factors that may have contributed to the tear. Common complications include rerupture and infection. Fortunately, partial quadriceps tendon ruptures have had better results, both with operative and non-operative treatment. Based on the paper presented below by Dobbs et al., the preferred management for partial ruptures is non-operative treatment in a knee-immobilizer for 4-6 weeks.

     

    Dobbs et al. reviewed 34 patients who sustained a quadriceps tendon rupture following a total knee arthroplasty. 23 of these patients had a partial tear, 7 of which were treated non-operatively. All of these patients had a satisfactory outcome. Of the 16 patients with partial tears treated operatively, 5 had a complication.

     

    Lin et al. present a case report on a patient with a patellar tendon rupture following a total knee arthroplasty treated with a turndown quadriceps flap and circumferential wiring. The patient was doing well 2 years after the operative with no extension lag and acceptable knee flexion.

     

    Kim et al. present a surgical technique for suture anchor repair of a ruptured quadriceps tendon after total knee arthroplasty. They recommend that 2 5.0-mm suture anchors should be placed in the patella with the accompanying fiber-wire secured to the quadriceps tendon.

     

    Incorrect Answers:

    Answer 1: Flexion is not drastically limited following the non-operative treatment of partial quadriceps tendon ruptures.

    Answer 3: Rerupture rates are low following the non-operative treatment of partial quadriceps tendon ruptures.

    Answer 4: In follow-up of patient treated non-operatively, the extension lag was minor, if present at all.

    Answer 5: Early failure has not been a reported complication following nonoperative treatment of partial quadriceps tendon ruptures following total knee arthroplasty.

     

     

     

    2020

     

  4. A 54-year-old male with progressive pain on the medial side of his knee undergoes the procedure seen in Figures A-C. Which of the

    following is the most likely reason this patient would require a revision procedure?

     

     

     

     

     

     

     

    1. Polyethylene wear

    2. Bearing "spin-out"

    3. Infection

    4. Tibial plateau stress fracture

    5. Progression of osteoarthritis Corrent answer: 5

    The most common reason for revision surgery after a unicompartmental arthroplasty is for progression of arthritis to the contralateral (lateral compartment if a medial compartment arthroplasty is performed) and patellofemoral compartments.

     

    Unicompartmental arthroplasty is an excellent procedure with good long-term clinical outcomes if appropriate indications are used for patient selection.

    Common causes for revision include progression of arthritis (most common), aseptic loosening, polyethylene wear, and infection. In order to avoid progression of arthritis, is is important to under-correct the deformity and place the components in the appropriate rotation to avoid impingement.

     

    Argenson et al. evaluated 62 patients (70 knees) at a mean follow-up of 20 years after undergoing unicompartmental arthroplasty. Fourteen of the 70 knee required a revision procedure, with causes including progression of arthritis (n=12), aseptic loosening (n=2), polyethylene wear (n=5), and late infection (n=1).

     

    Price and Svard review the results of 682 medial Oxford unicompartmental arthroplasties. Of these patients, 29 had revision procedures: 10 for arthritis progression, nine for aseptic loosening, five for infection, two bearing dislocations, and three for unexplained pain.

     

    Figures A-C show radiographs of a well-fixed, well-positioned, fixed bearing, medial unicompartmental arthroplasty. Illustration A shows how over-correction of the deformity can lead to contralateral compartment arthritis.

     

    Incorrect Answers:

    Answer 1: Revision for polyethylene wear occurs less frequently than for progression of arthritis.

    Answer 2: Bearing "spin-out" is a complication of mobile bearing unicompartmental arthroplasties. The radiographs do not suggest a mobile bearing device, and additionally, this occurs with less frequency than progression of arthritis.

    Answer 3: Infection is a relatively uncommon reason for revision of unicompartmental arthroplasties.

    Answer 4: Tibial plateau stress fractures are rare and are thought to be caused by multiple drill holes used for tibial instrumentation.

     

     

     

     

     

    2020

     

  5. Which of the following is a benefit of using a ceramic-on-ceramic bearing surface compared to a metal on highly cross-linked polyethylene bearing surface during a total hip arthroplasty?

    1. Increased jump distance

    2. Lower volumetric wear

    3. Decreased cost

    4. Lower rate of stripe wear

    5. Improved range of motion Corrent answer: 2

    Ceramic-on-ceramic bearing surfaces have the lowest volumetric wear when compared to all other combinations of bearing surfaces.

     

    Ceramic bearing surfaces, like other bearing surfaces, have their advantages and disadvantages. The biggest advantage of ceramic-on-ceramic bearing surfaces is that they have the best wear properties of all bearing surfaces due to their low coefficient of friction. Additional advantages include inert wear particles and high biocompatibility. Disadvantages of ceramic bearing surfaces include stripe wear, squeaking, the possibility of catastrophic fracture, and relatively high cost.

     

    Garino et al. review 333 total hip replacement treated with ceramic-on-ceramic bearing surfaces. At 3 years, there were 4 ceramic-related complications, including 3 chipped liners and one eccentric seating of the cup liner. No patients sustained a gross fracture of the ceramic components.

     

    Illustration A shows an example of a ceramic-on-ceramic bearing surface. Illustration B shows an example of stripe wear thought to be caused by edge

    loading.

     

    Incorrect Answers:

    Answer 1: Jump distance is amount of translation prior to dislocation. This is related to the size of the femoral head, not the bearing surface.

    Answer 3: Ceramic bearing surfaces are more expensive than metal on polyethylene surfaces.

    Answer 4: Stripe wear is a characteristic unique to ceramic-on-ceramic bearings. It is thought to be caused by edge-loading.

    Answer 5: Range of motion is related to the size of head that is used, not the bearing surface.

     

     

     

     

     

     

     

     

    2020

     

  6. A patient with osteonecrosis of his hip returns to your office to discuss findings of a recent MRI. Bone marrow edema seen on the MRI is most predictive of which of the following?

    1. Autoimmune disease

    2. Kerboul Grade 3 disease

    3. Ficat stage III disease

    4. Pain

    5. Healing

     

    Corrent answer: 4

     

    Bone marrow edema seen on an MRI of a hip with osteonecrosis is the best indicator of worsening pain and likely progression to collapse of the femoral head.

     

    Osteonecrosis of the hip is commonly evaluated by MRI, which has the highest sensitivity (99%) and specificity (99%) of all imaging modalities. If osteonecrosis is suspected but radiographs appear normal, MRI should be ordered to rule out or stage osteonecrosis. Bone marrow edema, as evidenced by high signal intensity on T2 weighted MRI, is not always seen. Hips with marked bone marrow edema are more likely to be painful, and more likely to progress to worsening pain and collapse of the femoral head.

     

    Ito et al. present a prospective cohort of patients with osteonecrosis of the hip followed with MRI at 6 to 12 month intervals. Those patients with marked bone marrow edema were more likely to experience worsening pain, larger volume of necrosis, and progression of their disease to collapse compared to patients without bone marrow edema on MRI.

     

    Ha et al. present a prospective cohort of patients with osteonecrosis of the hip and used MRI to measure the arc of the femoral head involved on a mid-sagittal and mid-coronal slice, and summed the two angles. This is a modification of a similar technique by Kerboul et al. which used radiographs instead of MRI. They found that a higher combined measurement was predictive of future progression of the disease.

     

    Illustration A is a coronal T2 weighted MR image of a right hip with osteonecrosis of the femoral head. Notice the high signal intensity within the femoral neck indicative of bone marrow edema. Illustration B demonstrates how to calculate the Kerboul Angle on x-ray radiographs. Illustration C demonstrates a similar measurement of necrotic angle using MRI, as was done in the Ha et al. paper cited above. Illustration D is an explanation of how to calculate a Kerboul angle, and how measurements are classified into grades.

     

    Incorrect answers:

    Answer 1: Bone marrow edema may be seen on MRI of hip osteonecrosis regardless of the underlying etiology.

    Answer 2: Kerboul Grade 3 disease is characterized by a combined necrotic angle of 250° to 299°.

    Answer 3: Ficat stage III is characterized by subchondral collapse (the

    "crescent sign").

    Answer 5: Bone marrow edema is predictive of worsening pain and progression to collapse of the femoral head.

     

     

     

     

     

     

     

     

     

     

     

     

     

     

    2020

     

  7. A 65-year-old male undergoes a left total knee arthroplasty for end-stage osteoarthritis. His pre-operative range of motion was 25 to 95 degrees of flexion. After releasing posterior capsule and resecting posterior osteophytes, a trial cruciate-substituting total knee arthroplasty was inserted. The surgeon noted the patients range of motion was 20 to 130 degrees of flexion. The knee was well balanced in flexion. What technical step would help to improve the overall balancing of this patients knee?

    1. Resect an additional 2mm of bone off the tibia

    2. Resect an additional 2mm of bone off the distal femur

    3. Downsize the femoral component and increase the polyethylene insert thickness by 2mm

    4. Increase slope of the tibia resection

    5. Translate the femoral component posterior by 4mm

    Corrent answer: 2

     

    This patient has a pre-operative flexion contracture of his left knee and lacks full extension in the operating room. The best technical step, after releasing posterior capsule and removing posterior osteophytes, would be to resect additional bone from the distal femur.

     

    Several surgical treatment options are available to correct preoperative flexion contractures, including medial and lateral soft tissue balancing, removal of posterior osteophytes, release of the posterior capsule, additional resection of the distal femur, and use of posterior-stabilized (PS) implants. Despite these techniques, residual flexion contracture is a common complication. Some studies have advocated that residual flexion contractures after TKA will improve over time, however results are less convincing when the deformities are more than 15°.

     

    Koh et al. looked at the incidence, predictors, and effects of residual flexion contracture (FC) on clinical outcomes after total knee arthroplasty in 911 knees. They found that patients with severe preoperative FC and anterior knee pain are at risk for developing postoperative FC. In addition, they state the difference between flexion and extension gaps does not necessarily increase the risk for postoperative FC.

     

    Onodera et al. looked at risk of knee flexion contractures associated with differences in posterior femoral condylar offset design after total knee arthroplasty. 100 total knee arthroplasties were evaluated. They found that excess posterior offset of the femoral condyle in prostheses may cause knee joint flexion contractures due to the relative shortening of the posterior soft tissues.

     

    Illustration A shows the technique for measuring the posterior femoral condylar offset DISTANCE. This is done by measuring the distance between a line drawn down the posterior aspect of the femur and the most posterior edge of the femoral condyle. Illustration B shows the technique for measuring the posterior femoral condylar offset RATIO. Two measurements are required.

    First the distance between the most posterior edge of the femoral condyle and edge of the posterior tibial plateau (labeled A). Second the anteroposterior diameter of the tibia (labeled B). ‘Posterior offset ratio’ is calculated by dividing ‘A/B’.

     

    Incorrect Answers:

    Answer 1: Resecting an additional 2mm of bone for the tibia would improve extension gap, however it would also loosen the flexion gap.

    Answer 3: Downsizing the femoral component and increasing the polyethylene

    insert thickness by 2mm would worsen extension tightness. Answer 4: Increasing tibial slope would loosen the flexion gap.

    Answer 5: Translating the femoral component posterior by 4mm would cause shortening of the posterior soft tissues and worsen the flexion contracture. It would also tighten the flexion gap by increasing femoral condylar offset.

     

     

     

     

     

     

     

     

     

    2020

     

  8. A computer navigation system was used during an uncomplicated primary total hip arthroplasty through a posterior approach. After acetabular component insertion, the navigation system shows the cup is positioned in 20 degrees of abduction and 20 degrees of anteversion. If the surgeon desires to position the component into the ideal orientation or "safe zone" to minimize the risk of dislocation, what alteration would need to be performed to the current position?

    1. No repositioning is required

    2. Maintain abduction, decrease anteversion by 10 degrees

    3. Increase abduction by 5 degrees and decrease anteversion by 5 degrees

    4. Increase abduction by 20 degrees, maintain anteversion

    5. Increase abduction and anteversion by 10 degrees each Corrent answer: 4

    30 to 50 degrees of abduction and 5 to 25 degrees of anteversion is traditionally accepted as an acetabular component’s ideal position or “safe zone” within which risk for dislocation is minimized. Out of all the available options, Answer 4 is the only repositioning movement that would position the component in the safe zone (20 [+20] = 40 degrees of abduction; and 20 [+0] = 20 degrees of anterversion).

     

    Four important variables have been shown to affect the stability of total hip arthroplasty. Factors shown to increase stability include (1) component design (e.g., large femoral heads, no neck skirt, increased neck-shaft offset), (2) component position (e.g., acetabular position [30-50 degrees of abduction, 5-25 degrees of anteversion], femoral position (10-15 degrees of anteversion),

    (3) soft-tissue tensioning (e.g., increasing length of femoral neck, decreasing neck-shaft angle, medializing the femoral neck while increasing femoral neck length), and (4) soft tissue function (e.g. no central or peripheral neurological dysfunction to soft tissue function.

     

    Lewinnek et al. reported the dislocation rates in a series of 300 hip replacements. The dislocation rate for cup orientation with anteversion of 15

    +/- 10 degrees and abduction of 40 +/- 10 degrees was 1.5%, while outside this "safe" range the dislocation rate was 6.1%.

     

    Barrack et al. evaluated the accuracy of acetabular component position in total hip arthroplasty. They defined target safe ranges for abduction and anteversion in total hip arthroplasty as 30° to 55° and 5° to 35°, respectively. They showed an increased odds of component malposition with lower-volume surgeons and patients with higher body mass index.

     

    Incorrect Answers:

    Answer 1: At 20 degrees abduction, the component is 'too adducted'. This can lead to impingement in flexion and inferior dislocation. Anteversion is acceptable.

    Answer 2: The final position would be 20 degrees abduction and 10 degrees anteversion. Again the component is 'too adducted'. This can lead to impingement in flexion and inferior dislocation. Anteversion is acceptable.

    Answer 3: The final position would be 25 degrees abduction and 15 degrees anteversion. Again the component is 'too adducted'. This can lead to impingement in flexion and inferior dislocation. Anteversion is acceptable.

    Answer 5: The final position would be 30 degrees abduction and 30 degrees anteversion. Abduction position would be acceptable. However, the component is 'too anteverted'. This would lead to anterior dislocation instability.

     

     

    2020

     

  9. In a well-positioned, metal-on-polyethylene (MoP) total hip replacement, increased wear and corrosion at the taper interface is associated with which of the following complications?

    1. Accelerated polyethylene wear

    2. Dislocation

    3. Pseudotumor

    4. Squeaking

    5. Septic arthritis

     

    Corrent answer: 3

     

    Increased wear and corrosion at the taper interface results in debris and metal ions. This is associated with pseudotumor formation.

     

    A pseudotumour is a type of adverse local tissues reaction (ALTR) that has become associated with metal-on-metal (MoM) joint failures. ALTRs are thought to occur due to the body’s response to metal ions, wear debris, and corrosive products, which are produced when there is motion between two contacting metal surfaces. Therefore, the taper interface also provides a setting where this process can occur. Fretting and corrosion damage to the taper interface have been linked to pseudotumor formation and aseptic loosening for both MoM and MoP joints.

     

    Cook et al. described two cases with large diameter MoP total hip replacements that developed pseudotumors requiring revision surgery. Examination of the retrieved implants revealed a discrete pattern of material loss from the distal end of the head taper/stem trunion interface. The authors postulated that the taper design did not provide a sufficient locking mechanism, allowing the head to toggle on the trunion. This led to an accelerated pattern of wear.

    Illustration A shows an example of a pseudotumor on MRI. Incorrect Answers:

    Answer 1: The primary factors associated with increased polyethylene (PE)

    wear are: PE thickness, sterilization and machining, articular surface design and kinematics.

    Answer 2: Metal ions and wear debris have not been identified as risk factors for dislocations, which usually occur as result of mechanical factors affecting the joint.

    Answer 4: Squeaking is most commonly encountered with ceramic-on-ceramic bearings and has not been associated with taper interface design or wear.

    Answer 5: Although septic arthritis should always be ruled out when considering the diagnose of a pseudotumor, increased taper interface wear has not been found to be associated with development of septic arthritis.

     

     

     

     

     

     

    2020

     

  10. A standard total knee is performed on a 56 year-old female using spinal anesthesia and a tourniquet. After cementation of all the components, the patella is noted to sublux laterally during range of motion. The alignment and rotation of the femoral, tibial, and patellar components all appear perfect. The surgeon should now:

    1. Perform a lateral release

    2. Revise tibial component into more external rotation

    3. Revise the femoral component into more external rotation

    4. Revise the patellar component to a more medial position on the native patella

    5. Reevaluate patellar tracking after deflation of the tourniquet Corrent answer: 5

      The question stem mentions that all components are in good position. Answer choices #2, 3, and 4 could be correct for a malpositioned component, but not in this case.

       

      Marson et al, in 1999, reviewed a total of 75 TKAs in 67 patients. Patients were divided into 3 groups: Group I were knees that tracked properly both before and after tourniquet release [34 of 75 (45.3%]. Group II were knees

      that maltracked with the tourniquet inflated and subsequently corrected with the tourniquet released [34 of 75 (45.3%)]. Group III were knees that maltracked both before and after tourniquet release, therefore requiring a lateral release [5 of 75 (6.7%)]. They concluded that tourniquet application alters intraoperative patellofemoral tracking during TKA and that when contemplating lateral release, tourniquet deflation and reevaluation of patellofemoral tracking should be considered.

       

      Likewise, Husted et al in 2005 in a prospective randomized study of 100 knees reported that if the patella was maltracking, tourniquet deflation led to better patella tracking and saved 5 of 16 patients (31%) from having lateral releases.

       

       

       

      2020

       

  11. Figures A to C are pre- and post-operative images of a 64-year-old male who underwent a primary total knee arthroplasty. He presents today complaining of persistent knee pain. The operative report states the surgeon used a 3 degree external rotation guide based off the posterior femoral condylar axis to prepare the femoral bone cuts. Which of the following technical approaches would have helped to prevent this component placement error?

     

     

     

     

     

     

    1. Utilizing the 0 degree external femoral rotation guide based off the posterior condylar axis

    2. Adding an additional 6 degrees of external rotation to the guide

    3. Upsizing the femoral component

    4. Internally rotating the femoral component 6 degrees relative to the

      transepicondylar axis

    5. Rotating the tibial component by an additional 6 degrees Corrent answer: 2

    Figures A and B show preoperative images of a valgus knee deformity with associated hypopastic lateral femoral condyle. Post-operative images shows 6 degrees of internal rotation of the femoral component relative to the transepicondylar axis. Out of the available options, adding an additional 6 degrees of external rotation would have prevented this component placement error (i.e., internal rotation of femoral component).

     

    The posterior condylar axis (aka posterior condylar line or angle) describes the normal posterior slope of the distal femur. This is typically measured to be 3 degrees of internal rotation relative to the tibial joint line. To create symmetrical gaps during TKA, 3 degrees of external rotation is usually built into the posterior cutting block, so the flexion gap is parallel to the horizontal mechanical axis. In valgus knees with hypopastic lateral femoral condyles, this slope is usually greater than 3 degrees. Therefore, if standard posterior condylar referencing is used, there is tendency to create internal malrotation of the femoral component. In this scenario, the posterior condylar line preoperatively, as shown in Illustration A, was measured to be 9 degree relative to the transepicondylar line. As the standard 3 degree external rotation guide was used based off the posterior femoral condylar axis, this resulted in 6 degrees of residual internal rotation, as shown in Figure 3.

     

    Miller et al. looked at the optimal rotation of the femoral component using different rotations, from 5 degrees internal rotation to 5 degrees external rotation. They showed that femoral component rotation parallel to the epicondylar axis resulted in the most normal patellar tracking and minimized patellofemoral shear forces early in flexion. These beneficial effects of femoral rotation were less reproducibly related to the posterior condyles.

     

    Victor et al. reviewed the literature on rotational alignment of the distal femur after total knee arthroplasty. The ideal angular relationships include posterior condylar line should be on average 3° internally rotated relative to the surgical transepicondylar axis (TEA) and 4° relative to the perpendicular to the trochlear anteroposterior axis (i.e., Whiteside line). They state the optimal rotational alignment of the femoral component in TKA should be parallel to the TEA which is best measured post-operatively with axial CT scan images.

     

    Pagnano et al. reviewed 52 patients with primary knee osteoarthritis. They measured the posterior condylar angle (line) intraoperatively to be on average

    3.98 degrees (range, 0 degrees to 9 degrees). In knees with varus joint lines

    >4 degrees (normal = neutral), the posterior condylar angle was more internally rotated relative to the transepicondylar axis. Therefore, to prevent internal malalignment using standard components, the femoral component should be more externally rotated relative to the posterior condylar axis in this scenario.

     

    Incorrect Answers:

    Answer 1: Utilizing the 0 degree external femoral rotation guide based off the posterior condylar axis would have resulted in 9 degrees of internal malrotation

    Answer 3: Upsizing the femoral component would not alter the rotational malalignment

    Answer 4: Internally rotating the femoral component 6 degrees relative to the transepicondylar axis would have resulted in the same component placement error.

    Answer 5: Rotating the tibial component by an additional 6 degrees would not address the femoral malalignment.

     

     

     

     

     

     

    2020

     

  12. Figure A is an MR image of a 38-year-old male with persistent hip pain. What is this patient's Kerboul angle measurement and corresponding risk for progression to future femoral head collapse?

     

     

     

    1. 126 degrees; severe risk

    2. 94 degrees; moderate risk

    3. 32 degrees; moderate risk

    4. 230 degrees; severe risk

    5. 220 degrees; moderate risk

     

    Corrent answer: 5

     

    The Kerboul combined necrotic angle is a good method to assess future collapse in hips with femoral head osteonecrosis. In this patient, the Kerboul necrotic angle measurement is 220 degrees, which corresponds to a moderate risk for progression to future head collapse.

     

    The Kerboul necrotic angle is calculated by measuring the arc of necrosis in the femoral head using (a) mid-saggital and (b) mid-coronal MR imaging, and adding these two values together (e.g., 126 + 94 = 220 degrees). This combined angle is thought to predict the future risk of femoral head collapse. Using the modified Kerboul method:

     

    Low-risk group = combined necrotic angle less than 190°

    Moderate-risk group = combined necrotic angle between 190° and 240° High-risk group = combined necrotic angle of more than 240°

     

    Ha et al. reviewed the predictors of femoral head collapse with osteonecrosis. In a prospective cohort of patients with osteonecrosis of the hip and MRI imaging was used to measure the arc of the femoral head involved on a mid-sagittal and mid-coronal slice, and summed the two angles together. This is a modification of a similar technique by Kerboul et al. which used radiographs instead of MRI. They found that a higher combined measurement was predictive of future progression of the disease.

     

    Zalavras et al. reviewed the surgical treatment options for osteonecrosis of the femoral head. Surgical options largely consist of preservation and replacement procedures. Preservation of the femoral head is largely considered for younger patients without head collapse. Treatment options include core decompression

    with vascularized bone grafts, avascular grafts, bone morphogenetic proteins, stem cells, or combinations of the above or rotational osteotomies.

     

    Figure A is a mid-coronal and mid-saggital MRI of a necrotic femoral head without evidence of collapse.

     

    Incorrect Answers:

    Answers 1-4: The combined necrotic angle is is calculated as 126 + 94 = 220 degrees. This corresponds to a moderate-rick group for future femoral head collapse.

     

     

     

    2020

     

  13. A 55-year-old woman had total hip arthroplasty 6 years ago. She now presents with hip pain for 1 year. Serum cobalt and chromium levels are 5ng/ml (normal, <0.9ng/mL) and 1ng/mL (normal,

    <0.3ng/mL), respectively. C-reactive protein level and erythrocyte sedimentation rate are normal. MRI with metal-artifact reduction sequence (MARS) shows capsular thickening and intracapsular fluid. A radiograph is shown in Figure A. At surgery, turbid grey fluid is evacuated and findings shown in Figure B are noted. Both femoral and acetabular components are stable. Black discoloration of the female end (bore) is noted. Based on the information provided, what is the next best step?

     

     

     

     

     

     

    1. Single-stage revision of all components

    2. Two-stage revision of all components

    3. Single stage revision of the neck, head and acetabular liner. Retain the femoral stem.

    4. Single-stage revision of the acetabular liner, head, neck and femoral stem using a modular femoral component

    5. Single-stage revision of the acetabular liner, head, neck and femoral stem using a monoblock femoral component

    Corrent answer: 5

     

    There is corrosion at the male part (trunnion) of the neck with surrounding adverse local tissue reaction (ALTR) and visible crevice corrosion on the female end of the neck-stem junction. It is advisable to revise the entire femoral component with a monoblock stem.

     

    Adverse local tissue reactions have been reported to occur after corrosion at

    the head-stem junction (traditional THA) and neck-stem junction (modular THA). Taper-junction corrosion may affect > 1/3 of all modular devices. Most radiographs are normal. The most common abnormal x-ray finding is medial calcar erosion as this area is adjacent to the taper and receives the highest metal ion load. The most common MRI finding is a thickened capsule with fluid in the effective joint space. Late cases have abductor destruction and femoral neck erosion. The most common lab findings are elevated cobalt (more soluble) with normal chromium (precipitates locally as chromium orthophosphate). In the above example, the patient’s immune system played a role in the development of ALTR via hypersensitivity or the patient's local biology created an environment supportive of mechanically assisted crevice corrosion for reasons that are yet unclear. Thus revision of a modular stem with corrosion should be with a monoblock stem to minimize modular taper junctions.

     

    Goyal et al. reviewed head-neck taper corrosion with well fixed femoral components in 86 retrieved femoral heads in metal-on-polyethylene conventional THAs undergoing head/liner exchanges (not for corrosion). They found no difference in survivorship with high-grade vs low-grade corrosion.

    They do not recommend removing well-fixed femoral stems with corrosion.

     

    Pivec et al. reviewed modular neck-stem taper junction corrosion. They outlined the 5 modes of corrosion in THA (galvanic, fretting, crevice, pitting and intergranular, Illustration A). The 2 most important modes in taper corrosion are: (1) Crevice corrosion leads to isolation of the implant surface from the aqueous environment and oxide layer damage that may not be repassivated and (2) Fretting is micromotion causing physical damage to the passivation layer

     

    Wyles et al. explored the utility of synovial fluid aspirations in failed metal-on-metal total hip arthroplasty. They found 39 culture negative cases and 4 culture-positive cases. Poor predictive markers for infection include synovial fluid WBC >3000cells/uL (100% sensitive, 57% specific), and CRP and ESR

    >8mg/L and >22mm/h (75% sensitive and 67% specific). Neutrophil percentage >80% was 100% sensitive and 97% specific and was an accurate marker for diagnosing infection. They hypothesized that inaccuracy arose from metal debris artificially elevating WBC count.

     

    Fig A shows a modular stem THA. Note the neck-stem and head-neck taper junctions. Fig B shows taper corrosion at the neck-stem component.

    Illustration A shows the different modes of corrosion in THA. Illustration B is an algorithm for management of modular THA failure. Illustration C demonstrates fretting of the modular component at the neck stem taper

    junction.

     

    Incorrect Answers:

    Answer 1: Revision of the acetabular components is unnecessary if the acetabular component is well fixed.

    Answer 2: Two-stage revision is only necessary for infection.

    Answer 3: Stem retention is possible if there is no gross corrosion of the female component.

    Answer 4: Revision components should avoid modular tapers to reduce further taper junction corrosion.

     

     

     

     

     

     

     

     

     

     

     

    2020

     

  14. A 65-year-old female scheduled to undergo a total hip replacement is concerned about the need of a blood transfusion. The surgeon should explain that which factor is the best predictor of the need for a blood transfusion?

    1. Shorter height

    2. Lower body weight

    3. Previous total hip replacement on the contralateral side

    4. Lower preoperative hemoglobin

    5. Female gender

     

    Corrent answer: 4

     

    Bierbaum et al in 1999 prospectively evaluated the post operative course of 9482 patients undergoing hip or knee replacement. They found low hemoglobin level < 130g/L preoperatively, revision hip, and bilateral knee to be associated with increased frequency of transfusion post op. In a retrospective study of 296 hip and knee replacement patients by Salido et al in 2002, only preoperative hemoglobin levels and weight of the patient were found to be significant factors in determining need for transfusion post operatively. Gender, height, and surgical duration were not found to be signficant factors in the multivariate analysis. Guerin et al in a prospective study of 162 consecutive THA and TKA found preoperative hemoglobin to be the only predictive factor for need of a transfusion. Weight and age were not significant when assessed by multivariate analysis. Thus, all three cited studies agree that the preoperative hemoglobin predicts need for transfusion.

     

     

    2020

     

  15. A 74-year-old female who underwent a hemiarthroplasty 6 weeks ago now presents to the emergency department after a low energy fall while walking. Her x-ray is shown in Figure A. What is the optimal treatment for this injury?

     

     

     

     

     

    1. Protected weight bearing with the use of a walker

    2. Open reduction internal fixation using cerclage wires

    3. Open reduction internal fixation using a locking plate +/- allograft strut

    4. Fracture fixation and revision arthroplasty with a diaphyseal engaging component

    5. Revision arthroplasty with a proximal femoral replacement Corrent answer: 4

    The question stem describes a Vancouver B2 fracture (fracture around the stem, with a loose stem), and thus the stem should be revised to a longer stem.

     

    Masri et al. describe the treatment of femoral periprosthetic femur fractures based primarily on implant stability, fracture location, and the quality of the surrounding bone. The resultant Vancouver classification utilizes these factors

    to assist with fracture classification and treatment. Only Vancouver B2/B3 fractures compromise stem fixation and thus are the only ones that generally require revision arthroplasty. A Vancouver B2 is a fracture about the stem where the implant is no longer stable, and is generally treated with revision arthroplasty to a longer stem. A Vancouver B3 is a fracture about the stem where the implant is no longer stable and the proximal bone is of very poor quality, generally treated with proximal femoral replacement.

     

    Ko et al. retrospectively reviewed the outcomes of Vancouver B2 periprosthetic femur fractures treated with the placement of a fluted tapered monoblock stem bypassing the fracture site and engaging the diaphysis. There were no cases of neurovascular injury, continued thigh pain, post-operative dislocation, or need for further surgery. The authors concluded that this fluted tapered stem is a satisfactory option for the treatment of this fracture.

     

    Illustration A shows the post-operative x-ray in this case after fracture fixation and revision arthroplasty with a diaphyseal engaging component. Illustration B details the Vancouver classification.

     

    Incorrect answers:

    Answer 1: This would be an appropriate treatment of a Vancouver A fracture. Answer 2: This would be an appropriate treatment of a Vancouver B1 fracture. Answer 3: This would be an appropriate treatment of a Vancouver B1/C fracture.

    Answer 5: This would be an appropriate treatment of a Vancouver B3 fracture.

     

     

     

     

     

     

     

     

    2020

     

  16. A 65-year-old female undergoes the procedure shown in Figure

    A. She is seen in your office for two-year follow-up complaining of a six week history of painful ambulation. She has not had any fevers or chills. A clinical photograph of her knee is shown in Figure B. Synovial fluid analysis reveals a white blood cell (WBC) count of 56,500

    cells/uL. Which of the following terms is the least accurate description for this clinical presentation?

     

     

     

     

     

     

    1. Periprosthetic joint infection

    2. Surgical site infection

    3. Draining sinus tract

    4. Purulence

    5. Chronic infection

     

    Corrent answer: 2

     

    A surgical site infection (SSI) with an implant in place is defined as an infection related to the surgery that occurs within one year of the index procedure.

     

    This patient presents with a chronic periprosthetic joint infection, suggested clinically by acute onset of painful ambulation for the past six weeks, peri-incisional erythema, and a sinus tract with purulent drainage from the surgical incision. Due to the chronicity of her symptoms, the infectious organisms are likely to exist as biofilm rather than in their planktonic state. Because she presents greater than one year post-operatively, this is not considered a true surgical site infection.

     

    Mangram et. al. from the United States Centers for Disease Control (CDC) published guidelines for the prevention of SSI. The CDC defines a surgical site infection as an infection occurring within 30 days after operation if no implant is left in place or within 90 days if an implant is in place and the infection is related to the operation. They are classified as being either incisional or organ-space, and incisional SSIs are further sub-divided into those involving only skin and subcutaneous tissue, and those involving deeper incisional soft-tissues.

     

    Figure A shows a total knee arthroplasty. Figure B demonstrates a suppurative infection with a draining sinus tract in the surgical incision.

     

    Incorrect Answers:

    Answer 1: This patient has a chronic infection as evidenced by her 6 week duration of symptoms.

    Answer 3: This patient has a draining sinus tract with visible purulence. Answer 4: The wound demonstrates purulent drainage

    Answer 5: In chronic infections, bacterial organisms are likely to exist in a biofilm state rather than in their planktonic state, making them adherent to prosthetic surfaces, resistant to antibiosis, and more challenging to eradicate.

     

     

     

    2020

  17. A 65-year-old woman with a history of left total knee arthroplasty 6 years ago presents to the emergency department complaining of knee pain, swelling, and fevers for 5 weeks. Upon examination of her knee, there is a 3+ effusion, warmth, tenderness, and limited range of motion secondary to pain. ESR, CRP, and WBC are elevated. The knee is then aspirated, and frank purulent fluid is obtained. Which of the following are true?

    1. The most likely organism to grow will be beta hemolytic Streptococcus

    2. Aspiration results with a cell count of 700 WBCs and 60% neutrophils fits this patient's clinical picture

    3. The best next step will be removal of all components and placement of antibiotic spacer

    4. Frozen sections are required for definitive diagnosis of prosthetic joint infection

    5. This patient's BMI of 42 kg/m2 does not increase their risk of reinfection after eradication of current infection

    Corrent answer: 3

     

    The patient in the clinical scenario has an obvious periprosthetic joint infection (PJI) of her total knee arthroplasty (TKA). Removal of all components and placement of an antibiotic spacer is the only true statement above.

     

    Infection rates after total hip arthroplasty (THA) and total knee arthroplasty (TKA) are estimated to be 1.7% and 2.5%, respectively. Management is determined by the chronicity of the infection. If the patient has clinical symptoms of an infection within 4 weeks of surgery, it is considered to be acute. A biofilm has not had time to be created and so the treatment is debridement and irrigation, polyethene exchange with prosthetic retention. If the patient begins to have clinical and objective signs of infection >4 weeks after surgery, the infection is now considered to be chronic. There is a very high likelihood of a biofilm on the implants and a two stage revision is required.

     

    Watts et al. performed a 2:1 matched retrospective cohort study investigating the outcomes of 2 stage revision TKAs for PJIs in comparison to non- obese patients receiving the same procedures. They found that patient with morbid obesity (defined as BMI greater than or equal to 40 kg/mg2) had increased risk for revision surgery, reinfection, and reoperation. They also found decreased implant survival at both 5 and 10 years post-operatively, worse pain relief, and decreased overall function after the procedures. They advised that poor outcomes should be anticipated with this population.

    Kuzyk et al. reviewed management of chronic hip and knee PJIs. They reviewed the criteria for diagnosis of a PJI, the technique of two stage revisions and the evidence for use and types of antibiotics in antibiotic cement. They also discussed various cement spacer designs and advantages, options for length of systemic antibiotics, joint fluid analysis after stage 1 procedures and its utility, and parameters for reimplantation for revision arthroplasties.

     

    Illustration A is a table of the AAOS Clinical Practice Guidelines for Diagnosis of PJI. Illustration 2 is an algorithm for treatment of chronic PJIs.

     

    Incorrect Answers:

    Answer 1: The most likely organism to grow in a chronic PJI is coagulase negative Staphylococcus and Proprionibacterium acnes. The most likely organism to cause an acute PJI is Staphylococcus aureus and beta hemolytic Streptococcus.

    Answer 2: An aspiration for PJI of a total knee has a WBC count >1,000 and

    >64% neutrophils.

    Answer 4: Frozen sections are used after appropriate local and intravenous antibiotic administration. They are not utilized in stage 1 revisions.

    Answer 5: A patient with a BMI greater than or equal to 40 kg/mg2 has a significantly increased risk of reinfection after a 2 stage revision has been completed.

     

     

     

     

     

     

     

     

    2020

     

  18. Figure A is a histologic specimen of a joint pseudocapsule surrounding a cobalt-chromium second-generation metal-on-metal (MOM) hip replacement prostheses. The dark areas represent:

     

     

     

    1. Particle-laden histiocytes

    2. Infiltrates of T-lymphocytes

    3. Infiltrates of B-lymphocytes

    4. Infiltrates of T- and B-lymphocytes

    5. Particle-laden macrophages

     

    Corrent answer: 4

     

    Figure A demonstrates the typical perivascular and diffuse infiltrates of T- and B-lymphocytes characteristic of metal-on-metal-related delayed type hypersensitivity (DTH).

     

    The EFFECTOR cell of DTH is the T-CELL. The MOST COMMON cell is the MACROPHAGE, as most DTH-participating cells are macrophages and only 5% of cells present are lymphocytes. Histological examination of the pseudocapsule will reveal BOTH T- and B-lymphocytes. Besides perivascular and diffuse lymphocytic infiltrates, there are also secondary lymphoid follicles suggestive of a hypersensitivity reaction. Released soluble metal (from metal degradation) can activate the immune system by forming complexes with native proteins, creating antigens/allergens. Polymeric wear debris is not known to be a major source of allergic-type immune response.

     

    Jacobs et al. reviewed MOM bearing surfaces. They state that the incidence of metal hypersensitivity (type IV delayed-type hypersensitivity) is 25% in patients with well and poorly functioning implants (twice the general population) and 50-60% in patients with failed implants awaiting revision. The authors note that metal sensitizers include nickel (most common), followed by cobalt and chromium. Metal antigen–sensitized T-cell DTH (T-DTH) lymphocytes release cytokines that recruit and activate macrophages,

    monocytes, neutrophils, and other inflammatory cells. Activated macrophages activate more T cells, which in turn activates more macrophages, which activates more T cells, and so on.

     

    Figures:

    Figure A shows perivascular and diffuse lymphocytic infiltrates in the joint pseudocapsule from a MOM total hip.

     

    Incorrect Answers:

    Answer 1: There is a paucity of particle-laden histiocytes (compared with conventional metal-on-polyethylene articulations) because of low wear rates and lesser wear particle volume.

    Answers 2 and 3: Both T- and B-lymphocytes are seen.

    Answer 5: Particle-laden macrophages are typical of metal-on-polyethylene bearing surfaces.

     

     

     

    2020

     

  19. A patient undergoes a routine total knee arthroplasty for osteoarthritis. The surgeon uses a posterior-stabilized implant with increased posterior femoral offset and a relatively small patellar component. Which of the following complications is most likely to ensue?

    1. Patellar maltracking

    2. Midflexion instability

    3. Early loosening

    4. Patellar clunk syndrome

    5. CAM jump (tibiofemoral prosthetic dislocation) Corrent answer: 4

    Posterior-stabilized implants with increased posterior femoral offset and a small patellar component have an increased risk for patellar clunk syndrome.

     

    Patellar clunk is a complication of posterior-stabilized total knee replacements. It is less common now with newer posterior-stabilized component designs.

    Patients with patellar clunk syndrome (PCS) present with a painful catch or "clunk" when the knee is brought into extension. In some patients a fibrous nodule forms at the proximal pole of the patella. As the knee extends, the nodule is "jerked" out of the notch causing pain and a clunk. These patients tend to present 7-11 months after the operation. Treatment options include non-operative measures, open arthrotomy and debridement, or arthroscopic

    debridement.

     

    Costanzo et al. report the incidence, risk factors, and functional outcomes in a series of patients treated for patellar clunk syndrome with arthroscopic debridement. PCS patients were found to have more valgus preoperative alignment, increased posterior femoral offset, and a small patellar component size.

     

    Dennis et al. reviewed 60 patients who required surgery for painful patellar crepitus. Risk factors included a shortened patellar tendon, use of small patellar components, decreased patellar composite thickness, and increased posterior femoral condylar offset.

     

    Snir et al. review the incidence of PCS in fixed versus high-flex mobile bearing posterior-stabilized knee replacements. Patellar clunk developed in 11.7% knees in the mobile group and 1.8% in the fixed bearing group (p<.001).

     

    Illustration A shows how a fibrous nodule gets stuck in the notch in flexion and is "jerked" out as the knee comes into extension. Illustration B shows a representation of posterior condylar offset and how to measure it on a radiograph.

     

    Incorrect Answers:

    Answer 1: Risk factors for patellar maltracking include internal rotation of the femoral or tibial components, improper patellar or femoral component positioning, and a uneven patellar resection.

    Answer 2: Elevated joint line, multiradii knee replacement designs, and MCL laxity are risk factors for midflexion instability.

    Answer 3: Early aseptic loosening is more likely to occur with poorly positioned implants, poor cementing technique, and/or cementless total knee replacements.

    Answer 5: CAM jump most frequently occurs when there is a large flexion gap in a posterior-stabilized knee replacement.

     

     

     

     

     

     

     

     

    2020

     

  20. Sterilization by gamma irradiation causes rapid in vivo wear of polyethylene when combined with which of the following?

    1. Ram bar extrusion of the polyethylene inserts

    2. An oxygen rich operating room (>20% FiO2)

    3. Irradiation in an inert gas medium

    4. Corrosive metals used adjacent to the insert

    5. Irradiation in air Corrent answer: 5

    Gamma irradiation is an effective way to sterilize implants; however, when the radiation occurs in an oxygen-rich air medium, the polyethylene becomes oxidized which can subject the implant to rapid failure.

     

    Gamma radiation is the most common way to sterilize polyethylene implants. When oxygen is present, the polyethylene becomes oxidized as opposed to cross-linked. Cross-linking helps to improve resistance to both adhesive and abrasive wear, which are the most common mechanisms of failure of current polyethylene implants. Without cross-linking, the polyethylene can delaminate and crack propagation is more common - these can lead to rapid wear of the implants given the uneven articulation surfaces that they create.

     

    Collier et al. retrieved nearly 200 implants from patients with total knee replacements containing polyethylene implants that had been sterilized with gamma radiation in air. They found that one additional years' shelf life correlated with increased wear from 0.11 to 0.14 mm / year, and that younger patients and those with knees that were varus also had increased linear wear rates and higher likelihood of implant failure.

     

    McGovern et al. report on a series of medial unicompartmental knee arthroplasties which utilized polyethylene implants treated with gamma irradiation in air which had long shelf lives. They found that increased shelf-life correlated with shorter time to revision, and that surface delamination and pitting occurred in a greater-than-expected frequency amongst these early arthroplasty failures.

     

    Incorrect Answers:

    Answer 1: Ram bar extrusion is a method of molding polyethylene implants and is separate from the gamma radiation sterilization process

    Answer 2: Without the gamma radiation from the sterilization process present, it is safe to have the polyethylene implant open in an oxygen-rich environment, such as that in an operating room

    Answer 3: Irradiation in an inert gas medium is helpful for cross-linking Answer 4: Corrosion occurs between adjacent metals, and refers to chemical dissolving of metal (e.g. head-neck junction of hip arthroplasty components)

     

     

     

    2020

     

  21. Which of the following methods in the manufacturing process of polyethylene provides the MOST improvement in wear performance?

    1. Ram bar extrusion

    2. Sterlization in gas plasma

    3. Gamma irradiation in argon

    4. Irradiation in an oxygen rich environment

    5. Sterilization in ethylene oxide Corrent answer: 3

    Gamma-irradiation of polyethylene in an inert gas improves the wear performance of polyethylene to a greater degree than the other options listed.

     

    The most common method for sterilization of polyethylene has been by direct exposure to gamma irradiation. The degree of postirradiation degradation can be altered by changing the method of sterilization. When polyethylene is sterilized without radiation, free radicals are not formed, meaning there will be no oxidative degradation. However, this radiation-free process does not provide any cross-linking, which has been shown to decrease wear. By irradiating the polyethylene in an inert gas, cross-linking may occur while minimizing the amount of free radicals. This has been showed to greatly improve its wear characteristics.

     

    Crowninshieldt et al. discuss some of the newer methods for sterilization of polyethylene. They mention that newer second-generation methods of crosslinking polyethylene have included the use of vitamin E, which quenches free radicals, providing low wear and desirable mechanical properties.

     

    Faris et al. compare three different methods of sterilization and the affect they have on total hip replacements. These methods included sterilization by radiation in: 1) inert gas with isostatically molded polyethylene, 2) inert gas and ram-extruded polyethylene, and 3) air with extruded polyethylene. The least wear occurred in gamma inert-molded polyethylene components.

     

    Illustration A shows the wear rate of polyethylene as a function of radiation dose. Note that increasing radiation decreases (improves) wear. However, increasing radiation dose increases free radical formation, thus necessitating strategies to decrease free radical formation (e.g. irradiation in oxygen free environment, vitamin E doping).

     

    Incorrect Answers:

    Answer 1: In ram bar extrusion, calcium stearate is added to the polyethylene which adversely affects wear rates.

    Answer 2: Sterilization in gas plasma does not create cross-linking. Crosslinking improves wear performance.

    Answer 4: Irradiation in an oxygen rich environment decreases the mechanical properties of polyethylene.

    Answer 5: Sterilization of polyethylene in ethylene oxide does not increase the wear performance as much as irradiation in an inert gas.

     

     

     

     

     

     

     

    2020

     

  22. A 76-year-old female presents to the emergency department with left thigh pain after a fall from a curb. She had undergone a successful total hip arthroplasty 8 years prior and was doing well without any antecedent thigh pain prior to this injury. Figure A is her injury radiograph. Which of the following is accurate regarding the most appropriate treatment for this patient?

     

     

     

    1. Locked compression plating is biomechanically superior to non-locked plating for this fracture pattern

    2. The femoral stem should be revised to a long diaphyseal-engaging stem

    3. Allograft struts are indicated due to fracture type and bone quality

    4. Cement augmentation is preferred

    5. Non-locked plate fixation is sufficient for this injury Corrent answer: 5

    The patient has sustained a Vancouver B1 periprosthetic femur fracture after total hip arthroplasty (THA) which requires ORIF with a compression plate with or without cable fixation. Studies show that fracture reduction and plate fixation for this injury generally leads to good outcomes with bony union.

     

    Treatment of periprosthetic femur fractures after hip arthroplasty is based on fracture pattern, location, the stability of the implant and host bone stock around the implant. Vancouver B1 fractures represent an injury at the level of or just below the femoral stem with good proximal bone stock and a well-fixed implant. While implant stability is often verified intra-operatively, there are no indications from either the history or imaging of implant loosening or subsidence. As a result, this fracture pattern may be treated with ORIF with a locking plate construct or conventional plating construct with retention of the well-fixed implant. Cerclage wires may be used for fixation proximal to the implant, though not a requirement, especially with newer periprosthetic

    instrumentation which effectively bypasses the implant stem.

     

    Wood et al. retrospectively investigated the success of using locking compression plate constructs for the treatment of Vancouver B1 and C type patterns in primary and revision THAs. Of the 16 patient cohort, 14 achieved union and 13 were ambulatory by 6 months. There was one patient who went on to implant failure secondary to nonunion resulting in a revision procedure. They recommended at least 10 cortices of fixation both proximal and distal to the fracture site when using a locking compression plate. This fixation was in the form of unicortical screws, bicortical screws, cables, or a combination.

     

    Bryant et al. performed a retrospective case series assessing union after using locked compression plating in Vancouver B1 type periprosthetic femur fractures. In a cohort of 10 fractures, there was 100% union with no hardware failures or complications requiring reoperation. They concluded that using an isolated lateral locked plate (without cerclage wires or strut allograft) that spans the majority of the femur is a successful treatment method.

     

    Fulkerson et al. performed a biomechanical study comparing the stability between locked compression plating and conventional compression plating with cerclage wiring in Vancouver B1 periprosthetic femur fractures. They used 8 pairs of matched embalmed femurs implanted with a cemented total hip prosthesis, created a periprosthetic fracture distal to the implant, then fixed the fracture with either method. After various loading to the cadaveric models, they concluded that though the locked plating was stiffer than conventional plating in axial loading and torsion, there was no difference in loads to failure between the two methods.

     

    Ricci et al. retrospectively assessed the outcomes of indirect plate fixation about Vancouver B1 type periprosthetic femur fractures. With 41 patients included in the final cohort, all went on to union, 3 had evidence of hardware failure without implant loosening, 1 patient had a deep infection requiring a return to OR, and 31 of the 41 patients returned to their preoperative ambulation status. They concluded that using indirect reduction with locking compression plate constructs should be a continued treatment method for B1 type fractures.

     

    Figure A is the AP of a Vancouver B1 periprosthetic femur fracture with a well-fixed stem.

    Illustrations A and B are post-operative images of a Vancouver B1 periprosthetic femur fracture with ORIF using a locking plate

     

    Incorrect Answers:

    Answer 1: Locking plate constructs have not been shown to be biomechanically superior to conventional plating methods

    Answer 2: This is the treatment for Vancouver B2 fracture patterns with a loose stem. The patient has a well-fixed stem (Vancouver B1)

    Answer 3: Union rates of B1 fractures are very high, and therefore strut allograft is not required to aid with union.

    Answer 4: There is no role for cement augmentation in the treatment of Vancouver B1 or B2 periprosthetic fractures.

     

     

     

     

     

     

     

    2020

     

  23. Figures A and B show the AP and lateral of a patient who underwent an uncomplicated total knee replacement 2 years ago. They have complained of persistent pain and recurrent effusions since the second post-operative month. Laboratory analysis was obtained, with an ESR of 20mm/hr (normal 0-12mm/hr) and a CRP of 6 mg/L (normal 1-3mg/L). She has had no fevers or chills. She states that she hears and feels a clunking sensation when ambulating. Physical exam shows moderate flexion instability and significant instability in 90 degrees of flexion. What is the next best step in management

     

     

     

    1. Revision of the femoral component

    2. Revision of the both components

    3. Explantation and placement of antibiotic spacer

    4. Aspiration of the knee joint

    5. Increase the size of the polyethylene Corrent answer: 4

    While the exam is consistent with a diagnosis of instability, laboratory analysis is concerning for infection. Whenever there is a concern for infection the next step in management is always aspiration of the joint for cell count and cultures.

     

    According to the AAOS Guidelines for the diagnosis of hip and knee periprosthetic joint infections (PJI), there is a STRONG recommendation for performing a joint aspiration in patients with an abnormal ESR and/or CRP. This should be sent for culture and cell count with differential. Only after infection has been ruled-out can other diagnoses be considered. The WBC count from the aspiration is a helpful piece of information in the diagnosis of PJI. While the exact values are somewhat controversial, most experts in the field would agree that a WBC of > 1,100 is consistent with possible infection. These results, however, should be taken into context with other data,

    including the CRP, %PMNs, duration from surgery, and the patient's clinical symptoms.

     

    Parvizi et al. reviewed the diagnosis of PJI of the knee. They reviewed the joint aspirations from 453 knee replacements using a cutoff value for infection of 1100 cells/uL for fluid leukocyte count and 64% for neutrophil differential.

    When both tests were negative, the negative predictive value was 99.6%. When both tests were positive, the positive predictive value was 100%.

     

    Rosenthall et al. discuss the importance of radiophosphate bone scanning in the detection of complications following total knee arthroplasty. They show that increased uptake in hip replacements should go away by 1 year in the absence of pathology, however, total knee replacements may take longer than a year.

     

    Figures A and B show an uncomplicated PS total knee replacement with a large joint effusion.

    Illustration A shows a diagnostic algorithm for the diagnosis of PJI. Incorrect Answers:

    Answer 1: Revision of the femoral component is generally only indicated for

    aseptic loosening of the femur.

    Answer 2: Revision of both components for instability is a reasonable option IF the aspiration is not consistent with infection.

    Answer 3: Explantation and placement of an antibiotic spacer is the appropriate procedure if the aspiration is consistent with infection.

    Answer 5: Increasing the size of the polyethylene has mixed results for the treatment on instability. Before any procedures for instability are considered, infection must be ruled-out.

     

     

     

     

     

    2020

     

  24. A 68-year-old male undergoes a left total hip arthroplasty for the treatment of his osteoarthritis. Which of the following is true in regards to his risk of infection?

    1. His body mass index (BMI) of 43 kg/m2 does not put him at increased risk for infection as it is under 45 kg/m2

    2. His albumin level of <3.0 g/dL reduces his risk for wound complications and infection.

    3. Closing the wound with staples instead of sutures will decrease the risk of infection

    4. The use of an indwelling catheter does not have an influence on his infection risk

    5. Receiving an allogenic blood transfusion increases the risk of infection Corrent answer: 5

    Patients receiving an allogenic blood transfusion are at an increased risk of periprosthetic joint postoperative infection.

    Infection rates after total hip arthroplasty (THA) and total knee arthroplasty (TKA) are estimated to be 1.7% and 2.5% respectively. This complication leads to a significant amount of re-operations in patients. Identifying modifiable risk factors for postoperative infection is important, allowing for an active role in further reducing the incidence. Interventions include preoperative antibiotic administration with in 60 minutes of incision, using appropriate skin preparation, frequently changing gloves during the procedure, using antibiotic impregnated cement, laminar flow rooms, limiting operating room traffic, minimizing time patients have indwelling catheters, and providing allogenic blood transfusions only when necessary.

     

    Friedman et al. performed a retrospective cohort study investigating the association of allogenic blood transfusion and postoperative infections after total hip arthroplasty (THA) or total knee arthroplasty (TKA). In a cohort of 6,313 patients, they found that patients who received no transfusion and those who received autologous blood transfusions had similar rates of infections (8.0% and 7.3% respectively). They also found a statistically significant increase in postoperative infection rates in patients who received allogenic blood transfusions (9.9%).

     

    Gomez-Lesmes et al. retrospectively looked at how the length of storage of the units of packed red blood cells used influenced the risk of prosthetic infection in patients who received primary total knee arthroplasties (TKA). A total of 1331 patients underwent TKA and 32 (2.4%) patients experienced deep wound infection. They found that patients who received blood that was stored for greater than 14 days, longer surgical times, and higher BMI had a higher risk of infection. They also found that the first case of the day had a significantly lower risk of infection.

    Illustration A is a chart of the risk factors of a periprosthetic joint infection. Incorrect answers:

    Answer 1: A BMI >40kg/m2 increases risk of postoperative infection.

    Answer 2: An albumin <3.5 g/dL indicates malnutrition and increases risk of postoperative infection.

    Answer 3: There is no difference in infection risk between closing the wound with sutures and staples.

    Answer 4: Leaving an indwelling catheter in for >48 hours puts patients at increased risk of UTI.

     

     

     

     

     

    2020

     

  25. A 25-year-old female presents with complaints of persistent left hip pain. A pelvic radiograph series demonstrates a lateral center edge angle of 16 degrees, a vertical center anterior margin angle of 18 degrees, a Tonnis angle of 15 degrees, a neck-shaft angle of 132 degrees, and a femoral alpha angle of 38 degrees. Magnetic resonance arthrogram demonstrates a degenerative superolateral labral tear with no lesions of the articular cartilage. What is the most appropriate surgical intervention for this patient?

    1. Surgical hip dislocation with femoroacetabular osteoplasty and labral repair

    2. Proximal femoral osteotomy

    3. Salter innominate osteotomy

    4. Bernese periacetabular osteotomy

    5. Total hip arthroplasty Corrent answer: 4

    The patient presents with hip pain and radiographic signs of adult hip dysplasia. She would be best treated with Bernese periacetabular osteotomy (PAO).

     

    Adult hip dysplasia is characterized by undercoverage of the femoral head, which leads to femoroacetabular pathomechanics that increase load to the superolateral acetabulum. These increased contact stresses cause

    degenerative labral disease, cartilage delamination, and progressive degenerative changes that lead ultimately to osteoarthritis. Early diagnosis of adult hip dysplasia in patients with maintained articular cartilage can benefit from reorienting PAO to increase acetabular coverage of the femoral head, which normalizes contact stresses and has been shown to preserve the hip.

     

    In 1988, Ganz et al. first described the reorienting PAO. The procedure was performed through a Smith-Petersen approach, and utilized osteotomies of the ischium, pubic ramus, supra-acetabular ilium, and the quadrilateral plate. They described this osteotomy as advantageous because it could be performed through a single approach and achieve large corrections in morphology, including the option of medializing or lateralizing the acetabulum.

     

    Parvizi et al. performed a retrospective analysis of 36 hips with a diagnosis of degenerative labral tears in the setting of developmental dysplasia of the hip who underwent arthroscopic labral debridement. The authors reported that all patients had improved functional scores at 6 weeks post-operatively, however the scores declined by two years post-operatively, and, in many, they identified accelerated hip osteoarthritis. Sixteen of their patients required further open surgery, including three who underwent total hip arthroplasty.

    The authors concluded that arthroscopic labral debridement in the setting of DDH without correction of the underlying morphological abnormalities is likely to fail.

     

    Steppacher et al. reported 19-year follow-up of the first 63 patients to undergo PAO. They reported that 60% of the hips remained preserved at final follow up. For the 40% who failed, they found six factors that predicted a poor outcome: young age at surgery, pre-operative Merle d’Aubigne and Postel score, positive anterior impingement test, limp, osteoarthrosis grade, and post-operative extrusion index. The Merle d'Aubigne and Postel scores are validated instruments that measure pre- and post-operative pain, gait, and mobility. They concluded that PAO was an effective technique for select patients with good results at least 19 years after surgery.

     

    Illustration A demonstrates the lateral center edge angle (LCEA) of Wiberg. The normal LCEA should measure >25 degrees, with angles <20 degrees consistent with dysplasia. Illustration B demonstrates the vertical center anterior margin angle (VCA), which is measured on the false profile pelvis xray. Normal VCA should measure >25 degrees, with angles < 20 degrees suggestive of dysplasia. Illustration C demonstrates the Tonnis angle. The normal Tonnis angle should measure <13 degrees, with angles greater than 13 degrees suggestive of dysplasia. Illustration D demonstrates the femoral neck-shaft angle (NSA). Normal NSA should measure between 120 and 135

    degrees. The illustration demonstrates valgus femoral necks. Illustration E demonstrates demonstrates the femoral alpha angle. Angles >42 degrees are suggestive of femoral cam lesions.

     

    Incorrect Answers:

    Answer 1: Surgical hip dislocation with femoroacetabular osteoplasty and labral repair would be indicated for patients with femoroacetabular impingement (FAI) that cannot be adequately treated with arthroscopy. This patient has radiographic signs of hip dysplasia, not FAI.

    Answer 2: Proximal femoral osteotomy would be indicated for pediatric patients with hip dysplasia manifesting with abnormal femoral neck-shaft angle or neck anteversion. This patient is skeletally mature and has a normal neck-shaft angle.

    Answer 3: Salter's innominate osteotomy is a redirecting acetabular osteotomy indicated for correction of developmental hip dysplasia in skeletally immature patients with a mobile pubic symphysis.

    Answer 5: Total hip arthroplasty would be indicated if this patient demonstrated advanced degeneration of the articular cartilage, which is not evident on MRI.

     

     

     

     

     

     

     

     

     

     

     

     

     

     

     

     

    2020

     

  26. All of the following promote osteolysis around orthopaedic implants and are paired with a correct inhibitor EXCEPT:

    1. RANK ligand, denosumab

    2. TNF-alpha, etanercept

    3. COX-2, celecoxib

    4. IL-6, tocilizumab

    5. IL-10, anakinra

       

      Corrent answer: 5

       

      Interleukin-10 (IL-10) inhibits proinflammatory cytokine production, osteoclastogenesis, and osteolysis. Anakinra is a receptor antagonist of IL-1, a proinflammatory cytokine.

       

      Periprosthetic osteolysis is a common mode of total hip arthroplasty failure. It

      is initiated by wear debris from prosthetic implants. Debris triggers activation of macrophages, which subsequently secrete many cytokines and inflammatory mediators, including TNF-alpha, IL-1, IL-6 and prostaglandins. The resulting inflammatory cascade ultimately promotes osteoclastic bone resorption and fibrous tissue formation.

       

      Talmo et al. detailed the basic science behind osteolysis and potential pharmacologic agents in the prevention and treatment of osteolysis. In addition to anti-RANK-ligand antibodies, TNF-alpha agents and COX-2 inhibitors, bisphosphonates and growth factors can also target osteolysis. Bisphosphonates incorporate into bone and prevent osteoclastic resorption. Growth factors such as TGF-beta and BMP-2 promote osteoblastic bone ingrowth into implant porosities, enhancing implant stability.

       

      Bukata et al. performed in vitro experiments with fibroblasts derived from wild-type (WT), COX-1 deficient and COX-2 deficient mice. Loss of COX-2 function resulted in decreased PGE2 production, as well as IL-6 production. This was replicated by addition of a COX-2 inhibitor to WT cells. Therefore, COX-2 inhibitors such as celecoxib may decrease osteolysis beyond direct inhibition of prostaglandin synthesis, by inhibiting the downstream proinflammatory cytokine cascade.

       

      Illustration A is a figure from Talmo et al that depicts the pathophysiology of osteolysis and the site of action for potential pharmacologic agents in the prevention and management of osteolysis.

       

      Incorrect Responses:

      Answer 1: RANK ligand stimulates osteoclastogenesis, promoting osteolysis and is inhibited by denosumab, a monoclonal antibody.

      Answer 2: TNF-alpha is a proinflammatory cytokine that promotes osteolysis and is inhibited by etanercept, a decoy receptor.

      Answer 3: COX-2 produces PGE2, which stimulates production of IL-6 and promotes osteolysis. Celecoxib is a selective COX-2 inhibitor.

      Answer 4: IL-6 is a proinflammatory cytokine that promotes osteolysis and is inhibited by tocilizumab, a monoclonal antibody.

       

       

       

       

       

      2020

       

  27. During total knee arthroplasty, an excessive posterior femoral resection will lead to which of the following scenarios?

    1. Loose extension and flexion gaps

    2. Loose extension gap

    3. Loose flexion gap

    4. Tight flexion gap

    5. Tight extension gap Corrent answer: 3

    Illustration A shows the flexion and extension gaps that occur during TKA. Cutting too much posterior femur will lead to a gap mismatch, with a larger flexion gap than extension gap. This creates laxity in knee flexion. If the distal femur is cut too aggressively, extension gap instability would occur. Over-resection of the proximal tibia leads to a balanced gap issue, with need for increased tibial polyethylene thickness to replace the resected bone.

     

    The referenced study by Clarke and Scuderi is a review article of flexion instability in total knee arthroplasty; this article covers diagnosis and treatments of scenarios such as seen in this question.

     

     

     

     

     

    2020

     

  28. A 70-year-old male returns to your clinic having decided to pursue hip replacement for his intractable pain. Which of the following scenarios is associated with the greatest risk of dislocation following primary total hip arthroplasty?

    1. Smith-Peterson approach with failed capsular repair

    2. Watson-Jones approach with direct penetration of iliospoas with retractors

    3. Hardinge approach with acetabular cup placement in 40 degrees of abduction

    4. Direct anterior approach with 15 degrees of anteversion

    5. Moore approach to the hip with capsulectomy Corrent answer: 5

    Of the listed choices, the Moore (or posterior) approach to the hip with capsulectomy, or without capsular repair, has the highest risk of postoperative dislocation.

     

    Hip dislocation after a total hip arthroplasty (THA) is one of the more common complications, with a reported rate ranging from 1 to 3%. There are many variables associated with hip instability including patient-based factors, surgical considerations, and implant positioning. Surgical considerations include the approach to the hip, extent of soft tissue dissection, repair of soft tissues, tensioning, component placement and alignment. Of the possible approaches, the traditional posterior approach to the hip has classically been associated with the highest risk of dislocation. This risk is substantially reduced and approaches that of the other anterior-based approaches with a

    stout capsular repair. Capsulectomy would preclude repair and thus contribute to increased instability. Instability can be further reduced through careful component placement, positioning, and alignment, as well as using appropriately conservative techniques during the procedure to minimize unnecessary soft tissue disruption and restore appropriate tension.

     

    Iorio et al. performed a retrospective review of 390 primary total THAs comparing the rates of dislocation in patients who underwent a posterior approach with a simple capsular repair, an enhanced posterior soft tissue repair (EPSTR), and a posterior minimal incision (10cm) approach using EPSTR. They reported an overall dislocation rate of 2.3%, attributable to 5.5% in patients who underwent a simple repair, 1.3% in patients with EPSTR, and 1.7% with minimal incision EPSTR. The authors concluded that when using a posterior approach to the hip, posterior soft tissue repair, specifically the EPSTR, significantly reduced rates of dislocation regardless of size of the incision.

     

    Pellicci et al. prospectively evaluated the competency of the posterior soft tissue repair with magnetic resonance imaging (MRI) after THA and whether the findings correlated with clinical instability. The authors reviewed the MRIs of 36 patients and found that despite gap formation between the edge of the capsule or external rotators and the greater trochanter, the gap filled in with scar tissue serving as a scaffold for the posterior soft tissue envelope. They also found a correlation between the integrity of the posterior repair and postoperative range of motion of the hip. However, they found no correlation between the integrity of the repair with clinical instability.

     

    Tsai et al. retrospectively reviewed the outcomes of 181 primary THAs where a posterolateral approach was used. One group of patients had their posterior capsule repaired, and the second group had their posterior capsule excised.

    The authors found that capsulectomy resulted in instability in 6.4% of patients while the cohort that underwent repair did not demonstrate any instability (0%). They concluded that the repair of the posterior capsule was absolutely necessary to restoring post-operative stability.

     

    Illustrations:

    Illustrations A is a postoperative radiograph after bilateral THAs. Illustration B is a series of intraoperative images from Tsai et al. A demonstrates the U-shaped capsulotomy, B shows the capsule tagged, C shows the capsular repair, and D demonstrates the repaired external rotators.

     

    Incorrect answers:

    Answers 1: The Smith-Peterson, or direct anterior, approach to the hip has a

    reliably low dislocation rate (0.5%) regardless of capsular repair or integrity. Answer 2: A Watson-Jones, or anterolateral, approach is similarly stable.

    Penetration of the iliopsoas would lead to increased risk of femoral nerve injury but would not compromise stability.

    Answer 3: The Hardinge, or direct lateral, approach to the hip has the lowest risk of dislocation. Furthermore, the cup is well within the optimal range of abduction to minimize instability.

    Answer 4: The direct anterior approach is again among the most stable approaches, and the version of the acetabular cup is within the appropriate safe range.

     

     

     

     

     

     

     

     

     

     

    2020

  29. A 63-year-old female undergoes a total hip arthroplasty for osteoarthritis. In order to decrease the joint reactive forces across the hip, which of the following labels in Figure A should be adjusted to have the most impact?

     

     

     

     

     

    1. A

    2. B

    3. C

    4. D

    5. E

     

    Corrent answer: 2

     

    In Figure A, label B represents horizontal offset. Increasing the horizontal offset will decrease joint reactive forces across the hip as this allows for a larger abductor moment arm and subsequent decreased abductor force necessary to stabilize the hip. Label A represents neck length, which when increased would indeed lead to an increase in horizontal offset, but would also increase leg length, so it is not the best answer.

     

    Blount wrote a landmark article describing optimal hip biomechanics which can be summarized by the following equation:

    Force from abductors x Horizontal offset (Distance to center of femoral head)

    = Force from body weight x Distance to center of femoral head

     

    The relationship between the distance from the center of the femoral head to the abductors or to the body weight can be altered by changing the femoral component horizontal offset or the acetabular component medial-lateral position, respectively.

     

    McAuley et al. review the importance of horizontal offset in the restoration of native soft tissue and abductor muscle tension, which correlates with hip stability post-operatively.

     

    Similarly, Incavo et al. highlight the impact of horizontal offset on THA outcomes. Too little offset can be associated with hip impingement or instability. Too much offset can lead to an inability to perform an adequate capsular closure, trochanteric bursitis, or a lateral cosmetic deformity.

     

    Della Valle et al. review the technique and importance of preoperative planning for THA in order to provide an estimate of the necessary hip center of rotation, horizontal offset, neck angle, and leg length in order to optimize implant fixation and hip biomechanics.

     

    Figure A is a diagram of the modifiable variables of a femoral component. Illustration A is the same diagram with the variables explicitly defined.

    Illustration B is a free body diagram demonstrating the forces across the hip joint. Illustration C demonstrates changes in offset in a femoral component.

     

    Incorrect Answers:

    Answer 1: Label A represents neck length, which when increased would indeed lead to an increase in horizontal offset and an alteration in joint reactive forces. However, this would also increase leg length, so it is not the best answer.

    Answer 3: Label C represents vertical height, which would increase leg length but have no impact on joint reactive forces.

    Answer 4: Label D represents stem length, which has no impact on joint reactive forces.

    Answer 5: Label E represents distal stem diameter, which has no impact on joint reactive forces.

     

     

     

     

     

     

     

     

     

     

    2020

     

  30. According to the AAOS Guidelines for the treatment of symptomatic osteoarthritis, which of the following treatments was given a strong recommendation in favor of its use?

    1. Self-management programs and low impact aerobic activity

    2. Weight loss for patient with a BMI greater than or equal to 25

    3. Intra-articular injection with corticosteroid

    4. Manual therapy (joint mobilization, joint manipulation, myofascial release)

    5. Acetaminophen

     

    Corrent answer: 1

     

    According to the AAOS guidelines for the treatment of symptomatic osteoarthritis of the knee, a STRONG recommendation was given for self-management programs, strengthening, low-impact aerobic exercises, and physical activity consistent with national guidelines.

     

    The AAOS guidelines for the treatment of osteoarthritis is a list of evidenced based treatment recommendations. Other STRONG recommendations include:

    1. We (AAOS) cannot recommend using acupuncture

    2. We (AAOS) cannot recommend using glucosamine and chondroitin

    3. We (AAOS) recommend NSAIDs (oral or topical) or Tramadol

    4. We (AAOS) cannot recommend using hyaluronic acid

    5. We (AAOS) cannot recommend performing arthroscopy with lavage and/or debridement

     

    Arden et al. present a randomized controlled trial comparing tidal irrigation to

    corticosteroid injections. At 26 weeks, 29% of corticosteroid injections and 64% of the tidal irrigation group reported improvement. They conclude that both provide short-term pain relief, but tidal irrigation offers a more sustained duration of pain relief.

     

    Coleman et al. present a randomized controlled trial comparing self-management education programs to a control group for the treatment of knee osteoarthritis. The self-management education program group had statistically significant improvements in pain, quality of life, and function at 8 weeks and 6 months when compared to the control group.

     

    Kirkley et al. present a randomized controlled trial comparing a medial unloader brace, a neoprene sleeve, and medical treatment only for the treatment of symptomatic, varus osteoarthritis. While a neoprene sleeve did offer some advantage compared to the control group, the medial unloader brace was superior to both in terms of pain and a thirty-second stair-climbing test.

    Reference 4 is a link to the actual guidelines. Incorrect Answers:

    Answer 2: Weight-loss was given a MODERATE recommendation by the AAOS.

    Answer 3: Corticosteroid injections were given an INCONCLUSIVE recommendation.

    Answer 4: Manual therapy (joint mobilization, joint manipulation, patellar mobilization, myofascial release) was given an INCONCLUSIVE recommendation.

    Answer 5: Acetaminophen was given an INCONCLUSIVE recommendation.

     

     

     

     

    2020

     

  31. A 75-year-old male requires revision total knee arthroplasty, 15 years after the index procedure. The operative report states the surgeon used standard-sized, cemented, posterior cruciate sacrificing components with size 13mm highly crosslinked polyethylene liner. What would be the MOST LIKELY etiology for revision TKA in this patient?

    1. Infection

    2. Aseptic loosening

    3. Instability

    4. Periprosthetic fracture

    5. Arthrofibrosis

     

    Corrent answer: 2

     

    Aseptic loosing is considered the most common etiology for late revision TKA (>10 years).

     

    The most common causes for failed primary TKA are aseptic loosening, infection, instability, periprosthetic fracture, and arthrofibrosis. Infection is considered the most common failure mechanism for early revision (< 2 years from index procedure) and aseptic loosening was the most common reason for late revision (>10 years). The use of highly crosslinked polyethylene inserts has shown to significantly reduce the complications from polyethylene (PE) wear.

     

    Sharkey et al. reviewed the etiology of TKA failure. In >700 revision surgeries, they found the most common aetiology for TKA failure was loosening (39.9%), infection (27.4%), instability (7.5%), periprosthetic fracture (4.7%), and arthrofibrosis (4.5%). Between 2-10 years from the index arthroplasty, loosening of the prosthesis accounted for 51.4% of patients undergoing revision.

     

    Bozic et al. reviewed the aetiology of revision TKA in the USA. Using national statistic data, 60,355 revision TKA procedures were performed in the United States between October 1, 2005 and December 31, 2006. Overall, the most common causes of revision TKA were infection (25.2%) and implant loosening (16.1%).

     

    Incorrect Answers:

    Answer 1: Infection is considered the most common failure mechanism for early revision (< 2 years from index procedure). Late infection (> 10 years) is less common.

    Answer 3: Stiffness, weakness and pain are the most common COMPLAINTS by patients in the immediate post-operative period after TKA. Rarely is there late onset instability with posterior cruciate sacrificing TKA.

    Answer 4: Periprosthetic fractures occur at the prevalence of 3-5% after TKA. Answer 5: Arthrofibrosis is a rare complication with TKA, with an incidence of 0-3%.

     

     

     

    2020

  32. A patient comes to the office with a flexion contracture following a total knee arthroplasty that has resulted in an unsatisfactory outcome. Intraoperative examination also reveals the knee is loose in flexion. What steps should be included in the revision surgery?

    1. Increase the polyethylene liner thickness

    2. Resect additional tibia

    3. Anteriorly translate the femoral component and decrease polyethylene thickness

    4. Resect additional distal femur and tibia

    5. Resect additional distal femur and upsize the femoral component Corrent answer: 5

    This patient has two problems: 1) loose in flexion (flexion laxity); 2) tight in extension (flexion contracture). Both of these should be addressed in surgery. In order to address the flexion instability, the surgeon can either upsize the femoral component or posteriorly translate the femoral component. Increasing the poly thickness will help the flexion instability, but worsen the tightness in extension. In order to address the extension tightness, the surgeon can downsize the poly insert, resect additional tibia, or resect additional distal femur. Downsizing the poly and resecting additional tibia, however, will both result in increased flexion instability. Resecting additional distal femur only affects the extension gap. Thus, the correct answer is upsizing the femoral component and resecting additional femur.

     

     

     

    2020

     

  33. Which of the following fractures in Figures A-E would best be treated by ORIF with a locking plate with cerclage wires?

     

     

     

     

     

     

     

     

    1. Figure A

    2. Figure B

    3. Figure C

    4. Figure D

    5. Figure E

     

    Corrent answer: 1

     

    Figure A shows an interprosthetic femur fracture in between a total hip and total knee prosthesis. This is an example of a Vancouver C fracture best treated with a locking plate and cerclage wires with retention of components.

     

    Periprosthetic fractures are an increasingly common problem with our increasing geriatric population and amount of joint replacement being performed on an annual basis. The Vancouver classification system has become widely accepted, mostly because of the ability to determine treatment based on fracture classification. The most important decision in pre-operative planning involves the determination of stem stability. While radiographs can give the correct diagnosis pre-operatively, intra-operative stability of the stem remains the gold-standard. For fractures below a well-fixed femoral component (Vancouver C), the best treatment is ORIF with a locking plate and cerclage wires/screws. Pre-operative history should include whether there was

    pain prior to the fall, which may indicate a loose stem.

     

    Masri et al. review the evaluation and management of periprosthetic femur fractures. They stress the importance of the site of the fracture, implant stability, and remaining bone stock when determining the ultimate operative treatment. They also state that in Vancouver C interprosthetic fractures where a stemmed total knee replacement was used, the plate must span the entirety of the stem.

     

    Figure B shows a Vancouver B3 fracture, with poor proximal bone stock. Figure C shows what appears to be a Vancouver B2 fracture based on its location and spiral pattern. This is commonly associated with a loose femoral stem. Figure D shows a Vancouver B3 fracture with massive lysis and likely poor proximal bone stock. Figure E shows a Vancouver B2 fracture.

     

    Illustration A shows the Vancouver classification with treatment considerations for each type of fracture.

     

    Incorrect Answers:

    Answer 2: Vancouver B3 fractures may require the use of a proximal femoral replacement or proximal femoral allograft in younger patients.

    Answer 3: Vancouver B2 fractures are best treated with femoral component revision and application of a plate that bypasses the distal aspect of the fracture by 2 cortical diameters.

    Answer 4: Vancouver B3 fractures may require the use of a proximal femoral replacement or proximal femoral allograft in younger patients.

    Answer 5: Vancouver B2 fractures are best treated with femoral stem revision that bypasses the distal aspect of the fracture by 2 cortical diameters as well as a plate to span the fracture and femoral stem.

     

     

     

     

     

     

    2020

     

  34. Two football players present with knee pain and instability after being tackled in a game. Figures A and B are injury radiographs for Player A and B, respectively. Which of the following is correct regarding the radiographic findings of these 2 players?

     

     

     

     

     

     

     

     

    1. Player A sustained an avulsion fracture of the anterolateral ligament. Player B sustained an avulsion fracture of the arcuate complex.

    2. Player A sustained an avulsion fracture of the arcuate complex. Player B sustained an avulsion fracture of the anterolateral ligament

    3. Both players sustained avulsion fractures of anterolateral ligament.

    4. Player A sustained an avulsion fracture of the iliotibial band. Player B sustained an avulsion fracture of the biceps femoris.

    5. Player A sustained a Segond fracture. Player B sustained a reverse Segond fracture.

    Corrent answer: 1

     

    Player A sustained an avulsion fracture of the anterolateral ligament (ALL). Its tibial attachment is midway bewteen Gerdy's tubercle and the fibular head.

    Player B sustained an avulsion fracture of the arcuate complex.

     

    A Segond fracture is commonly associated with ACL tear (75%). It is caused

    by internal rotation and varus load (unlike the more common cause of ACL tear, which is valgus stress). The arcuate sign is an avulsion fracture of the proximal fibula at the site of insertion of the arcuate ligament complex, and is usually associated with cruciate ligament injury (~90% of cases). The fracture fragment is attached to the LCL or biceps femoris tendon.

     

    Dodds et al. reviewed the anatomy of the ALL in 4 cadaveric knees. They found that the ligament was isometric from 0 to 60° of flexion and slackened with the knee flexed to 90°. It passed superficial to the LCL and knee capsule. Mean length was 59 mm, mean width was 6 mm. The femoral attachment was slightly posterior to both the lateral epicondyle and the femoral attachment of the LCL. The tibial attachment just distal to the anterolateral rim of the plateau, mid-way between the head of the fibula and Gerdy’s tubercle. An arcuate complex avulsion fracture is an avulsion of the fibular styloid attachment of the arcuate complex.

     

    Claes et al. (2014) compared the distance between the tibial ALL footprint to the center of Gerdy's tubercle (GT, in cadavers) vs the distance between GY and a Segond fracture (on CT). They found the former to be 22mm, and the latter to be 22.4mm, and this was not statistically significant. They concluded that the Segond fracture is a bony avulsion of the ALL.

     

    Claes et al. (2013) investigated the ALL in 41 cadaveric knees. They found that the ALL ran obliquely to the anterolateral proximal tibia with attachments to the lateral meniscus. The insertion on the tibia was midway between Gerdy's tubercle and the tip of the fibula head

     

    Figure A shows a Segond fracture (ALL avulsion fracture). Figure B shows the arcuate sign (arcuate complex avulsion fracture). Illustration A shows the ALL (arrowhead) overlying the LCL (star). Illustration B is a diagrammatic representation of of the ALL (59mm long, femoral attachment 8mm behind lateral epicondyle, tibial attachment 11mm below tibial joint line, 18mm behind Gerdy's tubercle, 17mm anterior to fibular head. Illustrations C and D are CT and MRI imaging reconciliation with anatomical specimens showing that a Segond fracture corresponds to the ALL attachment. Illustration E highlights the arcuate sign from Figure B.

     

    Incorrect Answers:

    Answers 2, 3: Figure A shows a Segond fracture (ALL avulsion fracture). Figure B shows the arcuate sign (arcuate complex avulsion fracture).

    Answer 4: The ITB attaches at Gerdy's tubercle.

    Answer 5: A reverse Segond fracture is an avulsion of the deep capsular component of the medial collateral ligament.

     

     

     

     

     

     

     

     

     

     

     

     

     

     

     

     

    2020

     

  35. A patient is scheduled to undergo total knee arthroplasty (TKA) following failure of nonsurgical management. History reveals that she underwent a patellectomy as a teenager as the result of a motor vehicle accident. Examination reveals normal ligamentous stability. For the most predictable outcome, which of the following implants should be used?

    1. Mobile-bearing knee

    2. Posterior cruciate ligament-sparing knee

    3. Posterior cruciate ligament-substituting knee

    4. Semiconstrained-style knee

    5. Triaxial hinged knee Corrent answer: 3

    Paletta and Laskins performed a retrospective study of the results of TKA with cement in 22 patients who had a previous patellectomy. Nine of the patients had insertion of a posterior cruciate ligament-substituting implant. Thirteen patients had insertion of a posterior cruciate ligament-sparing implant. The 5-year postoperative knee scores were 89 for the posterior cruciate ligament-substituting knee versus 67 for the posterior cruciate ligament-sparing knee (P

    < 0.01). The patella functions to increase the lever arm of the extensor mechanism and to position the quadriceps tendon and the patellar ligament roughly parallel to the anterior cruciate ligament and posterior cruciate ligament, respectively. The patellar ligament thereby provides a strong reinforcing structure that functions to prevent excessive anterior translation of the femur during flexion of the knee. The absence of the patella results in the patellar ligament and the quadriceps tendon being relatively in line with one another. After a patellectomy, the resultant quadriceps force is no longer parallel to the posterior cruciate ligament. This results in loss of the reinforcing function of the patellar ligament. The authors believe this loss of reinforcing function may place increased stresses on the posterior cruciate ligament and posterior aspect of the capsule, which may result in stretching of these structures over time. They found a high rate of anteroposterior instability, a high prevalence of recurvatum, and a high rate of loss of full active extension compared with passive extension in the posterior cruciate ligament-sparing group, which supports their theory.

     

     

     

    2020

     

  36. Figure 1 shows the radiograph of a patient who underwent a total knee revision with a posterior stabilized mobile-bearing prosthesis and now has recurrent knee dislocations. What is the most likely cause?

     

     

     

    1. Loose extension gap

    2. Loose flexion gap

    3. Malrotation of the tibial component

    4. Malrotation of the femoral component

    5. Poor prosthetic design Corrent answer: 2

    The patient has a posterior stabilized total knee revision, and the femoral component has dislocated over the tibial polyethylene cam/post. This usually indicates a loose flexion gap, or “flexion instability.” A loose flexion gap can occur due to undersizing of the femoral component, anteriorization of the femoral component, excessive distal augmentation of the distal femur, or collateral ligament insufficiency, especially if combined with posterior capsular insufficiency. Isolated laxity of the extension gap (with a well-balanced flexion gap) causes varus/valgus instability, but it rarely causes the femoral component to “jump” the tibial cam of a posterior stabilized tibial insert.

    Malrotation of the components may cause patellar instability or a rotational instability of the tibiofemoral joint but should not cause a frank posterior dislocation of the tibia, unless combined with other errors of balancing.

    Although a mobile-bearing total knee arthroplasty may be more sensitive to errors in balancing than a fixed-bearing total knee arthroplasty, this complication does not reflect a faulty prosthetic design.

     

     

     

    2020

  37. A metal-on-metal bearing used for total hip arthroplasty shows which of the following properties?

    1. Baseline serum ion levels increase with increasing activity levels.

    2. The risk of cancer is substantially increased.

    3. Linear ion production increases over time.

    4. Ions produced are excreted primarily through the kidney.

    5. Nickel is the most prevalent ion released into circulation.

     

    Corrent answer: 4

     

    Activity levels do not affect cobalt and chromium ion levels, which are the bulk of serum ion levels. The majority of ions are produced in the run-in period in the first several years. A gradual reduction in ion levels occurs thereafter. The kidneys are responsible for the bulk of clearance from the serum, and to date there is no relationship of cancer to ion levels in the serum.

     

     

     

    2020

     

  38. Figure 3 shows the AP radiograph of a patient with diabetes mellitus who has knee pain. A semiconstrained knee prosthesis was used in this patient to prevent which of the following complications?

     

     

     

     

     

    1. Infection

    2. Instability

    3. Stiffness

    4. Bone loss

    5. Malalignment

     

    Corrent answer: 2

     

    The radiographic appearance of the joint is highly suspicious for neuropathic joint (Charcot’s joint). Evidence of bone loss on both the tibial and the femoral sides may necessitate the use of metal and/or bone augments. Patients with a neuropathic joint often have excellent range of motion, and postoperative stiffness is not a problem. The main problem with these patients is instability that occurs secondary to ligamentous laxity. Use of a semiconstrained prosthesis prevents the latter complication.

     

     

     

    2020

     

  39. Based on the radiograph shown in Figure 4, the innervation of what muscle is most at risk with total hip arthroplasty?

     

     

     

     

    1. Quadriceps

    2. Extensor hallucis longus

    3. Lateral gastrocnemius

    4. Adductor magnus

    5. Semitendinosus

     

    Corrent answer: 2

     

    The radiograph reveals a Crowe IV deformity in a patient with developmental dysplasia of the hip. If hip arthroplasty is performed, then some degree of limb lengthening is anticipated. Excessive limb lengthening can result in sciatic nerve palsy in these patients. The peroneal branch of the sciatic nerve is most

    often affected. Of the muscles listed, only the extensor hallucis longus is innervated by the peroneal branch of the sciatic nerve.

     

     

     

    2020

     

  40. A 75-year-old woman who fell on her right knee now reports pain and is unable to bear weight. History reveals that she underwent total knee arthroplasty on the right knee 6 years ago. Radiographs are shown in Figure 5. Management should now consist of

     

     

     

     

     

    1. closed reduction and casting for 6 weeks.

    2. open reduction and internal fixation, using a locked intramedullary rod.

    3. open reduction and internal fixation, using two cancellous screws.

    4. open reduction and internal fixation, using a locked plate and screws.

    5. open reduction and internal fixation and revision of the femoral component.

     

    Corrent answer: 5

     

    The radiographs show a loose femoral component with an associated medial condyle distal femoral fracture. The treatment of choice is open reduction and internal fixation with revision of the femoral component because of the femoral component loosening.

     

     

     

    2020

     

  41. A 64-year-old man undergoes a primary total knee arthroplasty. Three months after surgery he reports persistent pain, weakness, and difficulty ambulating. Postoperative radiographs are shown in Figures 6a through 6c. What is the best course of action at this time?

     

     

     

     

     

     

    1. Hinged knee brace

    2. Patellar component revision with a tantalum implant and lateralization of the patella

    3. Revision knee arthroplasty with greater internal rotation of the tibial component

    4. Revision total knee arthroplasty with a lateral release and external rotation of the femoral component

    5. Revision total knee arthroplasty with a lateral release and internal rotation of the femoral component

    Corrent answer: 4

     

    The Merchant view reveals subluxation of the patellar component. The etiology of maltracking of the patella includes internal rotation of the femoral component, internal rotation of the tibial component, excessive patellar height, and lateralization of the patella component. The treatment of choice in this patient is revision total knee arthroplasty with external rotation of the femoral component. Preoperatively the patient also may require a lateral release,

    revision of the tibial component if it is internally rotated, and possibly a soft-tissue realignment. Component malalignment needs to be addressed first.

     

     

     

    2020

     

  42. Compared to metal-on-polyethylene total hip bearing surfaces, the debris particles generated by metal-on-metal articulations are

    1. larger and less numerous.

    2. larger and more numerous.

    3. smaller and less numerous.

    4. smaller and more numerous.

    5. not detectable.

     

    Corrent answer: 4

     

    Retrieval studies have shown that the debris particles produced by metal-on-metal articulations in total hip arthroplasty are several orders of magnitude smaller and may be up to 100 times more numerous than those found with metal-on-polyethylene articulations.

     

     

     

    2020

     

  43. A 60-year-old patient had the procedure shown in Figure 7 performed 5 years ago. When converting this patient to a total knee arthroplasty (TKA), what patellar problem is commonly encountered intraoperatively?

     

     

     

    1. Fracture

    2. Patella baja

    3. Patella alta

    4. Osteonecrosis

    5. Maltracking

     

    Corrent answer: 2

     

    Patella baja is commonly encountered when converting a high tibial osteotomy (HTO) to a TKA. Patella baja most likely occurs because of scarring. Meding and associates’ study did not show an increased rate of lateral release when converting a knee that had undergone a previous HTO.

     

     

     

    2020

     

  44. Antibiotic-loaded bone cement prostheses, such as that shown in Figure 8, are best created by using which of the following methods?

     

     

     

    1. Using commercially available antibiotic-loaded bone cement

    2. Adding 0.5 g vancomycin to commercially available antibiotic-loaded bone cement

    3. Adding 0.5 g tobramycin and 0.5 g vancomycin/unit of standard bone cement

    4. Adding either 1.0 g vancomycin or 1.2 g tobramycin per 40 g of standard bone cement

    5. Adding a minimum of 3.6 g tobramycin and 1.0 g vancomycin per 40 g of bone cement

    Corrent answer: 5

     

    In a review of the practical applications of antibiotic-loaded bone cement for the treatment of the infected total joint arthroplasties, Hanssen and Spangehl described commercially available antibiotic-loaded bone cement as low-dose antibiotic cements. These cements generally contained 0.5 g of either tobramycin or gentamicin per 40 g of cement. They are indicated for use in prophylaxis and not for treatment of infected total joint arthroplasties. High-dose antibiotic-loaded bone cements are described as those containing greater than 1.0 g of antibiotic per 40 g of cement. Effective elution levels have been documented with 3.6 g tobramycin and 1.0 g vancomycin per 40 g of bone cement. This was documented by Penner and associates. Furthermore, it was shown that the combination of the two antibiotics in the bone cement improved the elution of both antibiotics.

     

     

    2020

     

  45. Figures 9a and 9b show the radiographs of a 75-year-old man who underwent a revision total knee arthroplasty with a long-stemmed tibial component. In rehabilitation, he reports fullness and tenderness in the proximal medial leg (at the knee). The strategy that would best limit this postoperative problem is use of

     

     

     

     

     

    1. a base plate with an offset tibial stem attachment.

    2. a bone ingrowth surface on the augment.

    3. a nonstemmed tibial base plate.

    4. allograft bone instead of metal augments.

    5. bone cement to smooth the outline of the proximal medial tibia.

     

    Corrent answer: 1

     

    The problem with this reconstruction is the medial protrusion of the base plate. The use of a base plate with an offset stem can prevent the protrusion and thus the impingement and pain. Allograft bone or smoothing the outline with cement would be just as prominent and likely to cause pain. An ingrowth surface may improve soft-tissue attachment but would still leave the implant protruding medially and likely to cause pain. A nonstemmed tibial base plate would lead to less medial protrusion but at the expense of a smaller area for load carriage on the proximal tibia.

     

     

     

    2020

  46. Figure 10 shows the AP radiograph of an ambulatory 76-year-old patient. What is the most appropriate surgical treatment option for this patient?

     

     

     

     

     

    1. Revision arthroplasty using a cemented femoral component

    2. Impaction allografting of the femoral component

    3. Proximal femoral replacement arthroplasty

    4. Resection arthroplasty

    5. Hip arthrodesis

     

    Corrent answer: 3

     

    The patient has a periprosthetic fracture around a loose cemented femoral component. The proximal bone stock is poor; therefore, this fracture may be categorized as Vancouver 3-B. Hip arthrodesis and resection arthroplasty provide suboptimal results, particularly for ambulatory patients. Although impaction allografting may be an option to restore the bone stock in a younger patient, the latter procedure will be very difficult to perform when the proximal bone is poor in quality and fractured. Cementing another component into this wide femur is not an option. The best option for revision of the femoral component in this elderly patient is proximal femoral replacement arthroplasty.

     

     

     

    2020

  47. Patients with patellar clunk syndrome are best managed by which of the following methods?

    1. Rest and nonsteroidal anti-inflammatory drugs

    2. Surgical debridement

    3. Patellectomy

    4. Patellar revision

    5. Lateral release/patellar realignment Corrent answer: 2

    Patellar clunk syndrome is usually the result of a fibrous nodule that forms on the undersurface of the distal quadriceps tendon. It may get entrapped in the intercondylar notch of the femoral component during flexion, and lead to a sudden snap as the nodule is pulled out of the notch during active extension. Nonsurgical management is rarely successful. Surgical debridement is usually curative, with only rare recurrence. More aggressive procedures such as realignment, revision, or patellectomy are usually not necessary, and are reserved for cases resistant to soft-tissue debridement.

     

     

     

    2020

     

  48. Increasing articular conformity of the tibial polyethylene insert of a fixed-bearing total knee arthroplasty (TKA) prosthesis will have which of the following biomechanical effects?

    1. Decreased contact stress within the polyethylene

    2. Decreased risk of patellofemoral instability

    3. Decreased risk of mechanical loosening

    4. Increased risk of subsurface polyethylene cracking

    5. Increased femoral rollback during flexion Corrent answer: 1

    Increasing articular conformity increases the surface area for contact between the polyethylene and the femoral component. Advantages of this include lower peak contact stress within the polyethylene and less risk of polyethylene fatigue failure. Patellofemoral tracking is unchanged by increasing conformity unless gross component apposition is present. A potential disadvantage of increasing conformity includes some restriction in femoral rollback. Modest changes in conformity have not been shown to alter the rate of mechanical loosening. If conformity was increased to the extent of significant constraint, a potential increased risk of loosening would be expected, not a decrease.

    Design of modern TKAs includes a compromise in achieving enough constraint

    to lower polyethylene stress, without providing so much constraint as to limit kinematics and stress the fixation interfaces.

     

     

     

    2020

     

  49. A 63-year-old woman reports giving way of the knee and pain after undergoing primary total knee arthroplasty (TKA) 1 year ago. Examination reveals that the knee is stable in full extension but has gross anteroposterior instability at 90 degrees of flexion. The patient can fully extend her knee with normal quadriceps strength. Studies for infection are negative. AP and lateral radiographs are shown in Figures 12a and 12b, respectively. What is the appropriate management?

     

     

     

     

     

     

     

    1. Anti-inflammatory drugs

    2. Knee brace

    3. Physical therapy for quadriceps strengthening

    4. Revision to a thicker polyethylene insert

    5. Revision to a larger, posterior stabilized implant Corrent answer: 5

    The radiographs show posterior flexion instability that is the result of a flexion-extension gap imbalance and posterior cruciate ligament incompetence after a posterior cruciate ligament-retaining TKA. The femur is anteriorly displaced on the tibia, with lift-off of the femoral component from the tibial polyethylene.

    Revision to a larger femoral component will address the larger flexion gap relative to the extension gap, and a posterior stabilized implant will address the posterior cruciate ligament insufficiency. Pagnano and associates, reporting on a series of painful TKAs previously diagnosed as pain of unknown etiology, showed that the pain was secondary to flexion instability. Pain relief was achieved by revision to a posterior stabilized implant.

     

     

     

    2020

     

  50. A 72-year-old woman who underwent right total hip arthroplasty 7 years ago now reports right hip pain and limb shortening. Studies for infection are negative. AP and lateral radiographs are shown in Figures 13a and 13b. What is the most appropriate management?

     

     

     

     

     

     

    1. Observation only

    2. Nonsteroidal anti-inflammatory drugs and protected weight bearing

    3. Revision of the acetabular component with a jumbo cup with screws

    4. Revision of the acetabular component with a reinforcement cage and bone grafting

    5. Resection arthroplasty

     

    Corrent answer: 4

     

    Current literature supports the use of reinforcement cages for the reconstruction of failed, loosened acetabular components associated with major bone loss as seen in this patient. Although results of revision using the so-called jumbo cup with screws generally have been good, the amount of

    bone loss and medial wall penetration shown here and the likelihood of pelvic discontinuity precludes the use of that technique. With either technique, bone grafting of remaining defects is recommended.

     

     

     

    2020

     

  51. At the time of the revision surgery shown in Figure 14, the acetabular component was found to be stable. Polyethylene exchange with a standard ultra-high molecular weight polyethylene liner and grafting was performed. The patient is at significantly increased risk for

     

     

     

     

     

    1. loosening of the femoral component.

    2. loosening of the acetabular component.

    3. prosthetic hip dislocation.

    4. rapid wear of the polyethylene.

    5. continued expansion of the lytic defects.

     

    Corrent answer: 3

     

    Maloney and associates reported a 35% increased risk of pelvic osteolysis after total hip arthroplasty with a porous-coated acetabular component without cement. All components were stable at the time of revision. Only liners were exchanged and debridement of the granuloma with or without bone graft was performed. No defects progressed and one third of the lesions were no longer visible on radiographs, regardless of bone grafting. Unfortunately, despite the technical ease of many of these types of revisions, the dislocation rate for

    these cases is significant. Precautions should be taken postoperatively, and patients should be educated about this risk preoperatively.

     

     

     

    2020

     

  52. What is the most frequent complication of both lateral closing wedge high tibial osteotomy and medial opening wedge osteotomy?

    1. Patella baja

    2. Fracture

    3. Peroneal nerve palsy

    4. Compartment syndrome

    5. Infection

     

    Corrent answer: 1

     

    Scuderi and associates reported on patellar height after a high tibial osteotomy. Eighty-nine percent of the patellae, as measured by the Insall-Salvati index, and 76.3 percent, as measured by the Blackburne-Peel index, were observed to be lowered. More recently, Wright and associates reported a 64% incidence of patella baja in patients undergoing a medial opening wedge osteotomy. The incidence of intra-articular fracture during medial opening wedge osteotomy has been reported to be as high as 11% by Hernigou and associates, whereas the incidence of intra-articular fracture during lateral closing wedge high tibial osteotomy has been reported to be 10% to 20% by Matthews and associates. The incidence of peroneal nerve palsy with a lateral closing wedge high tibial osteotomy ranges from 0% to 20%, according to Marti and associates, whereas the incidence of peroneal palsy following a medial opening wedge osteotomy has been reported to be 15.7% by Flierl and associates. The exact incidence of compartment syndrome after a high tibial osteotomy is not known; however, it does not reach the level of patella baja. The incidence of deep infection after a lateral closing wedge high tibial osteotomy ranges from 0% to 4% according to Billings and associates.

     

     

     

    2020

     

  53. Stiffness can occur following total knee arthroplasty. What is the most appropriate management for a patient who has deteriorating arc of motion after undergoing a revision knee arthroplasty 9 months ago?

    1. Aggressive physical therapy

    2. Manipulation under anesthesia

    3. Investigation for periprosthetic infection

    4. Revision knee arthroplasty

    5. Resection arthroplasty

     

    Corrent answer: 3

     

    Stiffness following total knee arthroplasty can be a disabling condition. There are many reasons for loss of knee motion following total knee arthroplasty.

    Technical errors, such as overstuffing of the patella, malpositioning of the components, and ligamentous imbalance, are all known to result in stiffness following total knee arthroplasty. In some patients with a possible genetic predisposition, aggressive arthrofibrosis may develop and result in loss of knee motion. In any patient who has deteriorating knee motion, particularly after revision arthroplasty, deep infection should be ruled out. Although on occasion surgical intervention may be required to address knee stiffness, the outcome of revision surgery is poor if no reason for stiffness can be determined.

     

     

     

    2020

     

  54. A 59-year-old woman who underwent a total hip arthroplasty 5 years ago now has recurrent dislocation following bariatric surgery and a weight loss of 200 lb. An attempt at converting to a larger head size and trochanteric advancement has failed. Her components are well aligned. What is the best course of action?

    1. Resection arthroplasty

    2. Hip abduction brace

    3. Constrained acetabular liner

    4. Thermal ablation of the posterior capsule

    5. Conversion to a bipolar prosthesis Corrent answer: 3

    When a patient has well-aligned components and soft-tissue tensioning with a larger femoral head and trochanteric advancement has failed, options are limited. The use of a constrained acetabular liner is the best option in this situation. Goetz and associates and Shrader and associates have demonstrated good results with these implants. Shrader used this device on 109 patients with recurrent instability with a successful outcome in all but 2 patients. Resection arthroplasty is a salvage situation and is not the best option at the present time. A hip abduction brace does not address the soft-tissue laxity. Conversion to a bipolar arthroplasty, although possibly minimizing the incidence of dislocation, will lead to groin pain and migration of the component with diminished functional results.

     

     

    2020

     

  55. Figure 16 shows the radiograph of an otherwise healthy 62-year-old woman who fell. Management should consist of

     

     

     

     

    1. revision total hip arthroplasty with a cemented femoral component and adjuvant fracture fixation.

    2. revision total hip arthroplasty with a cementless femoral component and adjuvant fracture fixation.

    3. open reduction and internal fixation of the fracture and retention of the original components.

    4. removal of the components, open reduction and internal fixation of the fracture, and delayed replantation of the components when the fracture is healed.

    5. resection arthroplasty and internal fixation of the fracture.

     

    Corrent answer: 2

     

    The radiograph reveals that the femoral component is grossly loose as evidenced by disruption of the cement column; therefore, retention of the original components will not yield a successful outcome. A cementless revision is the procedure of choice. A strut graft and/or plate may be added at the surgeon’s discretion. A resection arthroplasty would only be considered in a nonambulatory patient. Cemented fixation of the revision component would be problematic given the numerous fracture fragments and the inability to contain the cement.

     

     

    2020

     

  56. A 67-year-old female has elected to undergo total knee arthroplasty for degenerative arthritis. A pre-operative radiograph is provided in Figure A. Exposure to place the distal femoral cutting guide is difficult due to poor knee flexion following a standard medial parapatellar arthrotomy. Which of the following techniques will enhance the exposure without altering post-operative rehabilitation or clinical outcomes?

     

     

     

     

     

    1. Lateral arthrotomy

    2. Complete release of the superficial and deep MCL

    3. Extending the arthrotomy to an extensile rectus snip exposure

    4. Patellectomy

    5. Converting to a mobile-bearing TKA design Corrent answer: 3

    The extensile rectus snip exposure is an extension of the standard medial parapatellar arthrotomy. The exposure is carried superiorly and laterally across the rectus femoris tendon. Illustration A demonstrates the exposure with the rectus snip portion outlined in red. It allows greater exposure of the knee to enhance bone cuts and component insertion without compromising clinical results. A lateral arthrotomy in addition to a medial arthrotomy puts the patella at risk for devascularization. Complete release of the MCL and patellectomy all necessitate post-operative activity precautions. Converting to a mobile-bearing TKA would have no impact on surgical exposure.

     

    Meek et al reviewed 50 total knees utilizing a rectus snip exposure and

    compared them to 57 total knees with a standard medial parapatellar arthrotomy. There were no differences between the groups with regard to function, pain, or patient satisfaction.

     

     

     

     

     

     

     

    2020

     

  57. A 42-year-old man undergoes right total hip arthroplasty for hip dysplasia. Postoperatively, he has a significant limb-length increase with a foot drop. A preoperative radiograph is shown in Figure 19. Which of the following should have been considered preoperatively to avoid this complication?

     

     

     

     

     

    1. Medialization of the acetabular component

    2. Use of a modular femoral implant

    3. Anterolateral approach to the hip

    4. Femoral shortening

    5. Electromyography

     

    Corrent answer: 4

     

    In a patient with bilateral hip dysplasia, there are significant technical challenges that need to be addressed to ensure a successful total hip arthroplasty. Restoring the center of the hip may cause significant lengthening and require femoral shortening. Lengthening of greater than 4 cm can lead to sciatic nerve palsy that will present clinically as a foot drop. A high hip center can be used when there is inadequate bone stock in the acetabulum to achieve adequate host bone coverage. A modular femoral implant may be used for a dysplastic hip with significant rotational deformity. Although an anterolateral approach to the hip may decrease the incidence of sciatic nerve palsy during the exposure, it will not be helpful when there is more than 4 cm of limb lengthening.

     

     

     

    2020

     

  58. A 58-year-old man reports a 2-month onset of groin pain with no history of trauma. Examination reveals that range of motion of the hip is mildly restricted, and he has pain with both weight bearing and at rest. An MRI scan is shown in Figure 20. Treatment should consist of

     

     

     

     

     

    1. protected weight bearing and anti-inflammatory drugs.

    2. core decompression of the femoral head.

    3. vascularized free fibular grafting to the femoral head.

    4. bipolar hemiarthroplasty of the hip.

    5. total hip arthroplasty.

    Corrent answer: 1

     

    The MRI findings show highly increased signal through the entire femoral head and neck on STIR imaging, diagnostic of transient osteoporosis of the femoral head. This disease entity can be seen in middle-aged men, and should be treated nonsurgically. The natural history is that of self-resolution.

     

     

     

    2020

     

  59. Figure 21 shows the radiograph of a 32-year-old patient with right hip pain that has failed to respond to nonsurgical management. What is the most appropriate surgical treatment at this time?

     

     

     

     

     

    1. Femoral derotational osteotomy

    2. Total hip arthroplasty

    3. Arthrodesis

    4. Surgical dislocation of the hip

    5. Periacetabular osteotomy

     

    Corrent answer: 5

     

    The radiograph reveals developmental dysplasia of both hips. The patient has classic anterolateral undercoverage of the femoral head on the right side as demonstrated by a high acetabular index (measured at 27 degrees). Anterior undercoverage can be determined by drawing the marking for the anterior wall that fails to overlap the femoral head in this patient. Currently in North America, the most accepted surgical management for symptomatic dysplasia of the hip with good joint space is a Bernese (Ganz) periacetabular osteotomy. Surgical dislocation of the hip and femoroacetabular osteoplasty may be

    considered for patients with symptomatic femoroacetabular impingement of the hip.

     

     

     

    2020

     

  60. A patient who underwent a total knee arthroplasty for osteoarthritis 18 months ago now reports the sudden development of pain in the ipsilateral knee. Radiographs and examination of the knee are unremarkable. Aspiration of the synovial fluid 3 days later reveals a WBC count of 1,500/mm3. The cells consist of 30% neutrophils and 70% monocytes. Culture results will not be available for several days. The patient has not been on antibiotics prior to this point. Based on these findings, what is the most appropriate management?

    1. Arthrotomy, debridement, and polyethylene exchange

    2. One-stage exchange arthroplasty

    3. Two-stage exchange arthroplasty

    4. Parenteral antibiotics

    5. Nonsurgical management without antibiotics Corrent answer: 5

    Synovial fluid analysis is a very sensitive tool for detecting infection in total knee arthroplasties. Several studies have demonstrated that an absolute leukocyte count in the synovial fluid of less than 1,700 to 2,500/mm3 is an accurate predictor of absence of infection. Similarly, a differential cell count of the WBCs demonstrating less than 50% to 60% neutrophils is an accurate predictor of absence of infection. If both parameters are normal, it is unlikely that the patient has an infection. The three surgical options are contraindicated based on the normal examination findings and laboratory parameters. Similarly, antibiotics should be avoided. The work-up should include tests to evaluate noninfectious sources of knee pain and sources of referred knee pain.

     

     

     

    2020

     

  61. A 38-year-old man who is an avid tennis player has had persistent pain over the medial aspect of his knee for the past 6 years. He notes that the pain occurs on a daily basis with any significant activity. Nonsteroidal anti-inflammatory drugs have failed to provide relief. Radiographs are shown in Figures 22a and 22b. What is the best course of action?

     

     

     

    1. Total knee arthroplasty

    2. Unicompartmental arthroplasty

    3. Insertion of a unispacer

    4. Tibial osteotomy

    5. Knee arthroscopy

     

    Corrent answer: 4

     

    In a relatively young patient who is an avid tennis player, the treatment of choice is a joint preserving procedure. The radiographs reveal varus alignment with loading of the medial compartment. After all nonsurgical management options have been used, the best treatment option is a medial opening wedge osteotomy. A lateral closing wedge osteotomy of the proximal tibia is also a reasonable option, but it is not one of the choices. A unicompartmental arthroplasty or a total knee arthroplasty would place significant restrictions in this patient. A unispacer may be a temporizing procedure but is controversial and without substantial data in the literature. The knee arthroscopy will not address the medial compartment osteoarthritis.

     

     

     

    2020

     

  62. Which of the following statements best describes the outcome of the routine use of continuous passive motion (CPM) machines after total knee arthroplasty (TKA)?

    1. CPM is likely to improve early range of motion and final range of motion.

    2. CPM may improve early range of motion but is unlikely to improve final range of motion.

    3. CPM is likely to decrease postoperative pain.

    4. CPM is likely to improve extension but not flexion.

    5. CPM is likely to restore quicker ambulatory ability.

     

    Corrent answer: 2

     

    Although CPM machines are used widely in the United States for patients undergoing TKA, the benefit seems to be marginal, if any. Numerous randomized trials have shown that final outcomes after total knee arthroplasty are unaffected by the use of CPM machines postoperatively. Some studies have suggested that use of CPM may improve flexion in the first few weeks, but any short-term benefit from the machine was lost by intermediate-term follow-up. Aside from potential improvement in flexion within the first few postoperative weeks, there does not appear to be any benefit from the machines. There is no improvement in pain, ambulation, or extension. The cost-effectiveness of these machines has been questioned by many authors.

     

     

     

    2020

     

  63. When performing knee arthroplasty, which of the following procedures provides the most consistent fixation for the tibial component?

    1. Cementless fixation of the tibial component

    2. Augmenting cementless fixation of the tibial component with pegs or screws

    3. Cementing the metaphyseal portion and press fitting the keel of the tibial component

    4. Cementing the metaphyseal and keel portions of the tibial component

    5. Cemented fixation of the tibial component with screws Corrent answer: 4

    All of the options, except cementing the metaphyseal portion and press fitting the keel of the tibial component, have been shown to create strong and long-lasting constructs; however, cementing of both the platform and the keel offers the most predictable solution. Cementing the platform and not the keel has been shown to have a higher loosening rate than the more traditional methods of fully cementing or using screws to augment fixation.

     

     

     

    2020

  64. Sterilization of ultra-high molecular weight polyethylene by gamma irradiation in air will degrade its wear performance because of

    1. oxidation.

    2. melting.

    3. cross-linking.

    4. corrosion.

    5. creep.

     

    Corrent answer: 1

     

    Gamma irradiation has long been used as a sterilization method for polyethylene. Exposure to gamma irradiation causes breakage of the chemical bonds in the polyethylene, and oxidation will occur if the material is subsequently exposed to air. The amount of oxidation and decrease in wear performance is also related to the length of time that the gamma-irradiated polyethylene is exposed to oxygen.

     

     

     

    2020

     

  65. Figure 23 shows failure of the femoral stem in a patient. What is the most likely reason for the failure?

     

     

     

     

    1. Torsional loading

    2. Cantilever bending

    3. Pistoning

    4. Subsidence

    5. Torque

     

    Corrent answer: 2

     

    A two-dimensional stress analysis has been used to study the effects of some of the factors leading to early fatigue failure of the femoral stem in total hip arthroplasty. It has been demonstrated that loss of proximal stem support at the level of the calcar femorale and subsequent stem stress can lead to fatigue failure. In addition, the role of body weight and range of cyclic stress fluctuation play an important role in fatigue life under conditions where the stem has lost proximal support. These results indicate that stem design could be improved by incorporating some means of adequate support at the calcar femorale where maximum tensile stresses are found to occur. Femoral component fracture is a rare but well-documented complication after total hip arthroplasty. Historically, most stem fractures occur at the middle third of the implant where proximal stem loosening and solid distal stem fixation result in cantilever bending and eventual fatigue failure. The component shown is a modular fluted cementless stem that occasionally fractures at the modular junction in patients with poor proximal bone support.

     

     

     

    2020

     

  66. Which of the following aids in correction of patellar tracking after total knee arthroplasty (TKA)?

    1. Internal rotation of the femoral component

    2. Internal rotation of the tibial component

    3. Increasing size of the tibial component

    4. Medialization of the patellar component

    5. Joint line elevation Corrent answer: 4

    Correct patellofemoral tracking has proven to be a crucial aspect in TKA because a large percent of problems after TKA are related to the patellofemoral articulation. External rotation of the femoral and tibial components has been shown to aid in tracking. Likewise, medialization of the patellar button aids in patellar tracking and prevention of lateral subluxations and dislocations. Attention to the distal femoral cut is critical in maintaining the joint line and preventing patella baja or alta. Tibial sizing, however, is not directly related to patellar tracking after TKA.

     

     

    2020

     

  67. Figure 25 shows the radiograph of an 84-year-old woman who has pain and is unable to extend her knee. History reveals that she underwent total knee arthroplasty 8 years ago. Aspiration and studies for infection are negative. During revision surgery, management of the tibial bone loss should consist of

     

     

     

     

     

    1. reconstruction with a metal augmented revision tibial implant.

    2. reconstruction with a hinged prosthesis.

    3. reconstruction with a structural allograft.

    4. reconstruction with iliac crest bone graft.

    5. filling the defect with cement.

     

    Corrent answer: 1

     

    Massive bone loss encountered in revision total knee arthroplasty remains a significant challenge. Recent reports have shown high success rates using structural allograft to reconstruct large structural bone defects. A hinged prosthesis is not required in this setting. In this patient, a large amount of posterior cortex has been lost, making the area too large to fill with cement or iliac crest bone graft. Because of her age, the treatment of choice is a revision tibial implant and metal augments. Structural allograft would be suitable in a younger patient.

     

     

     

    2020

     

  68. A 58-year-old patient who underwent bilateral hip arthroplasty 12 years ago now reports pain in his hips and difficulty with ambulation to the point where he now uses crutches. A radiograph of

    the hip and pelvis is shown in Figure 26. What is the best treatment option for this patient?

     

     

     

     

     

    1. Revision hip arthroplasty with a bipolar implant

    2. Revision hip arthroplasty with impaction grafting on the femoral and acetabular side

    3. Revision hip arthroplasty with a cemented jumbo acetabular component

    4. Revision hip arthroplasty with a cementless acetabular component

    5. Acetabular component revision with a tri-flange protrusio ring Corrent answer: 4

    The radiographs reveal acetabular component failure with bone loss. There are several treatment options available. The best option for survivorship is a cementless porous-coated acetabular component. This patient may or may not require structural bone graft, which may need to be determined at the time of surgery. Bipolar implants and cemented acetabular components for revision surgery have not demonstrated long-term success. The use of a protrusio ring is reserved primarily for massive bone loss such as a Paprosky type III bone loss with significant superior migration of the acetabular component. The best clinical results for acetabular component revision have been achieved with cementless porous-coated implants.

     

     

     

    2020

     

  69. Figure 27 shows the AP radiograph of a patient who has late instability. The problem most likely occurred as a result of

     

     

     

    1. greater trochanter detachment.

    2. femoral stem loosening.

    3. wear.

    4. osteolysis.

    5. infection.

     

    Corrent answer: 3

     

    Although dislocation can occur anytime after hip arthroplasty, the highest incidence is observed within the first few months. Dislocation occurring many years after arthroplasty has also been described. In contrast to early dislocation, it appears that late dislocation frequently requires surgical intervention. Recent studies suggest that the incidence of late dislocation may be greater than initially appreciated and that the cumulative rate of dislocation rises with increasing follow-up. The presumed etiologic factors for late instability include long-standing problems with the prosthesis (such as malpositioning of the components) with late manifestation, trauma, deterioration in the neurologic status of the patient, and polyethylene wear.

    The eccentric position of the femoral head in this patient confirms polyethylene wear. The femoral stem is well-fixed, and the greater trochanter osteotomy has united well. The minor osteolysis observed around the proximal femur is also the consequence of wear and is not the cause of instability. Infection, without component loosening and massive soft-tissue destruction, is not otherwise known to result in late instability.

     

     

    2020

     

  70. Figure 28 shows the postoperative radiograph of a 36-year-old patient. The cerclage cable was placed for a minimal medial calcar fracture seen during femoral preparation. In the immediate postoperative period, what is the highest level of activity that would be safely permitted?

     

     

     

     

     

    1. Immediate full weight bearing

    2. Protected weight bearing

    3. Toe touch weight bearing

    4. Non-weight-bearing

    5. 50% weight bearing

     

    Corrent answer: 1

     

    The incidence of femoral fracture in primary cementless total hip arthroplasty ranges from 1.5% to 27.8%. It is imperative that the implant and fracture are stable both intraoperatively and postoperatively. Cerclage wiring or cerclage cabling is the current recommended treatment for nondisplaced calcar fractures and minimally displaced proximal fractures. Berend and associates reviewed the results of 58 total hips in 55 patients with intraoperative calcar fracture managed with single or multiple cerclage wires or cables and immediate full weight bearing. Follow-up averaged 7.5 years, and there were no revisions of the femoral component. No patients had severe thigh pain.

     

     

    2020

     

  71. Figure 30 shows the MRI scan of a 68-year-old woman who has left hip pain. What is the most appropriate treatment?

     

     

     

     

    1. Open reduction and internal fixation

    2. Total hip arthroplasty

    3. Incisional biopsy

    4. Proximal femoral resection and reconstruction

    5. Arthrodesis

     

    Corrent answer: 2

     

    The patient has a large zone of osteonecrosis of the left femoral head. The wedge-shaped zone of decreased signal intensity on the T1 image in the subchondral region of the femoral head is typical. Based on these findings, total hip arthroplasty is the most appropriate treatment. Open reduction and internal fixation will not help this condition. Incisional biopsy is indicated only if the MRI scan shows a probable neoplasm. Resection of the proximal femur is indicated only for aggressive malignancy. Arthrodesis may be considered in a younger patient but not in a 68-year-old individual. Other treatments, not listed, such as core decompression, vascularized fibular transplant, and osteotomy may be options in selected patients.

     

     

     

    2020

     

  72. A 30-year-old patient has had severe left hip pain and difficulty ambulating, necessitating the use of a cane, for the past 6 months. A

    photomicrograph of the femoral head sectioned at the time of surgery is shown in Figure 31. What is the most likely diagnosis?

     

     

     

     

     

    1. Renal osteodystrophy

    2. Pyogenic osteomyelitis

    3. Osteoarthritis

    4. Osteonecrosis

    5. Tuberculosis osteomyelitis

     

    Corrent answer: 4

     

    The photomicrograph demonstrates a wedge-shaped infarct with femoral head collapse; therefore, the diagnosis is osteonecrosis of the femoral head. Perthes disease and osteoarthritis do not involve a wedge-shaped defect. Tuberculosis of the hip joint results in greater destruction of the articular cartilage.

     

     

     

    2020

     

  73. When comparing mobile-bearing total knee arthroplasty (TKA) to fixed-bearing total condylar arthroplasty, the mobile-bearing procedure provides

    1. no improvement in survivorship.

    2. approximately 15 degrees greater flexion.

    3. appreciable reduction in wear rates.

    4. a faster recovery profile.

    5. better quadriceps strength.

     

    Corrent answer: 1

     

    Survivorship is similar in the two groups. In a recent study, mobile-bearing TKAs showed a slightly higher maximum flexion than the total condylar fixed-bearing-type designs (112 degrees versus 108 degrees with no difference in recovery rate). Using a fixed-bearing or a mobile-bearing design did not seem to influence the recovery rate in early results after knee arthroplasty. Mobile-bearing arthroplasties are suggested, in theory, to offer a reduction in polyethylene wear; however, clinical studies have not yet proven this.

    Recovery rates have yet to be statistically seen as improved with either method. Differences in strength have not been shown.

     

     

     

    2020

     

  74. Based on the type of articulation shown in Figure 32, wear is not affected by which of the following factors?

     

     

     

     

    1. Radial mismatch of the femoral head to the acetabular component

    2. Sphericity of the bearings

    3. Surface finish of the articulation

    4. Carbon content of the metal-on-metal bearing

    5. Head-to-neck ratio

    Corrent answer: 5

     

    Wear in total hip arthroplasty is a very complex phenomenon. The radial mismatch of the femoral head to the acetabular component has been shown in multiple studies to be a significant factor in wear. The mismatch can neither be too small nor too large. When the mismatch is too small, seizing of the implants can occur. When the mismatch is too large, contact stresses increase and produce exceptionally high wear. The ideal radial mismatch should be approximately 50 microns. Surface roughness and ball sphericity are two items that are extremely important with respect to wear. High carbon content has been shown to decrease wear. This device has a very large head-to-neck ratio, so impingement-related wear is unlikely.

     

     

     

    2020

     

  75. A 78-year-old patient undergoing revision total knee arthroplasty has bone loss throughout the knee at the time of revision. A distal femoral augment is used to restore the joint line. One month after surgery, the patient reports pain and is unable to ambulate. A lateral radiograph is shown in Figure 34. What is the most likely etiology of this problem?

     

     

     

     

     

    1. Inadequate restoration of the joint line

    2. Patellar tendon rupture

    3. Excessive internal rotation of the tibial component

    4. Flexion gap instability

    5. Hyperextension of the femoral component Corrent answer: 4

    Instability is a leading cause of failure following total knee arthroplasty. Instability can present as global instability, extension gap (varus/valgus) instability, or flexion gap (anterior/posterior) instability. Treatment options are numerous based on the exact pathology. The radiograph reveals anterior/posterior instability with dislocation consistent with flexion gap instability. A loose flexion gap can allow the femoral component to ride above the tibial cam post mechanism, resulting in dislocation. Distal femoral augments treat extension gap instability, whereas tibial augments can treat both flexion and extension gap instability. Posterior condyle augments at the distal femur can also be used to treat flexion gap instability. Flexion gap instability is further aggravated by extension mechanism incompetence. Note the excessively thin patella on the lateral radiograph.

     

     

     

    2020

     

  76. Figure 35 shows the AP radiograph of a patient who underwent a previous upper tibial osteotomy (UTO). The patient may be at risk for which of the following during total knee arthroplasty (TKA)?

     

     

     

     

     

    1. Bone loss

    2. Patella alta

    3. Myositis ossificans

    4. Fracture

    5. Instability

     

    Corrent answer: 5

     

    The results of TKA for patients with a prior UTO are reported to be slightly suboptimal. The major problems are patella baja, difficulty in exposure, and instability. Most of the patients exhibit some degree of instability prior to TKA, and ligamentous balancing may be difficult. Ligamentous structures are at risk of rupture during the difficult exposure. The problem of ligamentous balancing is exacerbated by the change in the joint slope that can occur after UTO.

     

     

     

    2020

     

  77. Figure 36 shows the radiograph of a patient who has hip pain and is unable to ambulate. What is the most appropriate management for this patient?

     

     

     

     

     

    1. Bisphosphonates

    2. Protected weight bearing

    3. Open reduction and internal fixation

    4. Revision total hip arthroplasty

    5. Resection arthroplasty

     

    Corrent answer: 4

    The patient has a periprosthetic fracture of the greater trochanter - Vancouver

    A. The reason for the fracture of the greater trochanter is the extensive periarticular osteolysis that has occurred as a result of polyethylene wear. The latter is demonstrated by eccentric seating of the large femoral head in the acetabulum. The most appropriate management is to reverse the osteolysis process, which involves exchange of the acetabular liner with or without revision of the other components depending on their fixation and position. The greater trochanter can also be fixed during revision surgery.

     

     

     

    2020

     

  78. When polyethylene is exposed to radiation and subsequently heated, certain chemical changes occur in the material. Which of the following statements best describes these changes?

    1. The process converts an otherwise interpenetrating networking structure of polymer chains into a linear, high molecular weight polyethylene macromolecule.

    2. The process increases the ductility of the material.

    3. The process leads to fewer particles that are larger in size than the untreated material.

    4. The process improves (lowers) the wear rate but may increase the risk of fracture.

    5. The process decreases the wear rate of the material, compared to untreated polyethylene, when tested against a rough counterface.

    Corrent answer: 4

     

    Exposure of polyethylene to radiation and then heating it to quench the free radicals leads to a cross-linked material. It converts a high molecular weight polyethylene macromolecule to an interpenetrating network structure of polymer chains. The ductility of the material is decreased, hence the greater risk of fracture. While the wear rate (measured as fewer and smaller particles) against a smooth counterface is markedly reduced, cross-linked polyethylene has shown a larger increase in wear rate when a rougher counterface is used compared to noncross-linked material. Due to reduced mechanical strength, highly cross-linked polyethylene is less resistant to abrasive wear.

     

     

     

    2020

     

  79. Figure 37 reveals a periprosthetic fracture around a cemented femoral stem in an 81-year-old patient with Paget’s disease and mild

    coagulopathy. What is the most appropriate reconstructive management on the femoral side?

     

     

     

     

     

    1. Open reduction and internal fixation

    2. Impaction allografting

    3. Proximally coated femoral stem

    4. Allograft prosthetic composite (APC)

    5. Proximal femoral replacement (PFR) Corrent answer: 5

    This is an example of a Vancouver B3 periprosthetic fracture that consists of a fracture around a loose femoral stem with poor proximal bone support.

    Therefore, open reduction and internal fixation is not an option. PFR is an excellent choice for elderly inactive patients with poor femoral bone stock. The surgery can be performed in an expeditious manner, which is very important in a patient with mild coagulopathy. Impaction allografting and APC are both options for younger patients who have bone stock that needs to be restored.

    The results of revision arthroplasty using proximally coated stems, especially under these circumstances, are poor.

     

     

     

    2020

     

  80. A patient with a documented allergy to nickel requires a total knee arthroplasty. Which of the following prostheses is most likely to provide long-term success in this individual?

    1. All-polyethylene tibial component and pure titanium femoral component

    2. All-polyethylene tibial component and cobalt-chromium alloy femoral component

    3. Cobalt-chromium alloy tibial component and cobalt-chromium alloy femoral component

    4. Modular titanium tibial component and pure titanium femoral component

    5. Modular titanium tibial component and oxidized zirconium femoral component

    Corrent answer: 5

     

    Nickel allergy is not an infrequent preoperative finding. The ramifications of such allergies in arthroplasty patients are poorly understood at this time.

    Stainless steel and cobalt-chromium alloys contain relatively high concentrations of nickel. Titanium, oxidized zirconium, and polyethylene do not contain significant amounts of nickel. Titanium is not a good surface for the articulating portion of the femoral component because of its propensity for metallosis. Oxidized zirconium is the only suitable femoral component for patients allergic to nickel. A modular titanium tibial component or an all-polyethylene tibial component would be satisfactory for these patients.

     

     

     

    2020

     

  81. A 42-year-old man reports the recent onset of right hip pain. A radiograph and MRI scan are shown in Figures 38a and 38b. A WBC count, erythrocyte sedimentation rate, and hip aspiration are within normal limits. Management should now consist of

     

     

     

     

     

     

    1. core decompression.

    2. biopsy of the femoral head.

    3. protected weight bearing and observation.

    4. total hip arthroplasty.

    5. percutaneous cannulated pin fixation of the femoral neck.

     

    Corrent answer: 3

     

    Transient osteoporosis of the hip is an uncommon problem, usually affecting women in the last trimester of pregnancy and middle-aged men. Symptoms include pain in the involved hip with temporary osteopenia; however, there is no joint space involvement. In this patient, the imaging findings are consistent with transient osteoporosis. Short TR/TE (repetition time/echo time) images reveal diffusely decreased signal intensity in the femoral head and intracapsular region of the femoral neck. Increased signal intensity is seen with increased T2-weighting. Within a few months, the pain, as well as the

    imaging findings, will completely resolve without intervention. Distinguishing the diffuse features of transient osteoporosis of the hip from the segmental findings of osteonecrosis is essential. Unlike transient osteoporosis of the hip, osteonecrosis will have a double-density signal on MRI and may progress radiographically. Surgical intervention and oral corticosteriods are not indicated for treatment. Protected weight bearing until the pain resolves may decrease symptoms while the transient osteoporosis resolves.

     

     

     

    2020

     

  82. What are the optimal conditions for leaving the acetabular shell in place, replacing the acetabular liner, and grafting the osteolytic defect shown in Figure 39?

     

     

     

     

     

    1. Nonmodular implant

    2. Instability

    3. Well-designed, well-fixed modular implant

    4. Complete radiolucency of the acetabular component

    5. Migration of the acetabular component Corrent answer: 3

    Dense pods of ingrowth into the porous coating of cementless ingrowth sockets are seen. Channels through the non-ingrown portion allow access to the trabecular bone of the ilium. Polyethylene wear debris can enter these areas through screw holes. Expansile, lytic lesions can result, which can become large without compromising implant fixation. Loosening is late and results from catastrophic loss of bone. A well-fixed acetabular component with a modular design, a well-designed locking mechanism, and a good

    survivorship history is a candidate for exchange of the liner and grafting of the osteolytic lesion.

     

     

     

    2020

     

  83. A 53-year-old patient is seen in the emergency department after sustaining a fall onto her left hip. A current radiograph is shown in Figure 40. What is the best treatment option?

     

     

     

     

     

    1. Bed rest and non-weight-bearing for 6 to 8 weeks

    2. Component retention and open reduction and internal fixation

    3. Proximal femoral replacement prosthesis

    4. Revision arthroplasty with a long cemented stem

    5. Revision arthroplasty with a long porous-coated cylindrical stem Corrent answer: 5

    The patient has sustained a Vancouver B2 periprosthetic femoral fracture (a femoral fracture that occurs around or just distal to a loose stem, with adequate proximal bone stock). The stem is no longer fixed to proximal bone; therefore, retention of the femoral component is not recommended.

    Nonsurgical management is contraindicated because of the high risk of nonunion and malunion with significant component settling in the distal fragment and leg shortening. Revision femoral arthroplasty must attain distal fixation in adequate host bone, which is usually successful with a porous-coated cylindrical stem.

     

     

     

    2020

  84. With the increasing availability of total hip arthroplasty (THA) to younger patients with hip osteoarthritis, there has been increased use of alternative bearing surfaces. Compared to a ceramic-on-ceramic articulation, which of the following is a specific advantage of a metal-on-metal bearing surface?

    1. Increased wettability

    2. Increased hardness

    3. Increased fracture toughness

    4. Decreased surface roughness

    5. Lower coefficient of friction Corrent answer: 3

    Alternative bearing surfaces in THA have received much attention in recent years as more and more hip arthroplasties are being performed on younger patients with hip arthritis. The two most popular nonmetal-on-polyethylene bearing surfaces are metal-on-metal and ceramic-on-ceramic. There are arguments supporting the use of either, but ceramic bearings have been shown to have a theoretic increased risk of fracture compared with cobalt-chromium. This has been shown to be clinically relevant with zirconium ceramics. Newer alumina ceramics are being produced with lower porosity and grain size and with higher density and purity, resulting in lower fracture risk but still greater than that of cobalt-chromium.

     

     

     

    2020

     

  85. In the radiograph shown in Figure 42, the fracture pattern around this well-fixed stem is classified as Vancouver type

     

     

     

    1. A.

    2. B1.

    3. B2.

    4. B3.

    5. C.

     

    Corrent answer: 2

     

    The Vancouver classifications describes periprosthetic hip fractures in the following way. Type A fractures are in the trochanteric region. Type B1 fractures occur around the stem or at the tip in the face of a well-fixed stem. These are usually treated with open reduction and internal fixation, usually including struts, cable, and/or cable plates. Type B2 fractures occur in the same region with a loose stem. Type B3 fractures occur with a loose stem where the proximal bone is of poor quality and/or severely comminuted. Type C fractures occur well below the stem.

     

     

     

    2020

     

  86. Figure 44 shows the radiograph of a 65-year-old man who underwent a revision arthroplasty to remove a loose, cemented femoral stem. When planning the postoperative restrictions, the surgeon should be aware that

     

     

     

    1. the approach used reduces the torque-to-failure (fracture) of the construct to less than 50% of the intact femur.

    2. the technique of repair can return the reconstructed prosthesis/bone composite to nearly the strength of the intact femur.

    3. there is no relationship between the density of the native bone and the strength of the prosthesis/bone composite.

    4. the addition of bone graft substitute or autograft has been shown to lessen the time to complete healing.

    5. there is a one in five chance of fracture with this technique; therefore, the surgeon must carefully weigh the potential benefits versus this risk.

    Corrent answer: 1

     

    The transfemoral approach, also known as the extended trochanteric osteotomy, is an important technique to master for revision hip surgery. When performed correctly, it allows excellent exposure of the femoral canal and aids in exposure of the acetabulum. As demonstrated in the study cited, however, it markedly reduces the torque that the composite can withstand without failure. This type of basic science study is important to guide postoperative rehabilitation.

     

     

     

    2020

     

  87. A 37-year-old man who works in a factory has isolated, lateral unicompartmental pain about his knee with activities. Nonsurgical management has failed to provide relief. The radiograph shown in Figure 45 reveals a tibiofemoral angle of approximately 15 degrees which is clinically correctable to neutral. What is the best surgical option in this patient?

     

     

     

    1. Unicompartmental arthroplasty

    2. Total knee arthroplasty

    3. Lateral closing wedge proximal tibial osteotomy

    4. Medial opening wedge proximal tibial osteotomy

    5. Medial closing wedge supracondylar femoral osteotomy Corrent answer: 5

    Patients with a valgus alignment about the knee can have lateral compartment arthritis. Similar to a high tibial osteotomy, a supracondylar femoral osteotomy is indicated in younger patients who have a more active lifestyle and isolated unicompartmental disease. In this young patient who works in a factory and has a valgus knee, a medial closing wedge supracondylar femoral osteotomy is the treatment of choice. The role of arthroplasty is limited in younger patients.

     

     

     

    2020

     

  88. Figure 48a shows the full-leg standing radiograph of a patient with a prior femoral fracture. Figure 48b shows the lateral view of the same joint. The patient is scheduled to undergo total knee arthroplasty. Because the mechanical axis of the lower extremity in patients with a prior femoral fracture may be disrupted, which of the following should be used during surgery to restore the mechanical axis of the lower extremity in this patient?

     

     

     

    1. Customized components

    2. Specialized intramedullary jigs

    3. Hinged prosthesis

    4. Extra-articular osteotomy

    5. Routine knee prosthesis Corrent answer: 5

    The radiograph shows hardware that was used for fixation of a prior femoral fracture. The mechanical axis of the lower extremity in this patient is nearly normal (3 degrees valgus), and the deformity at the healed fracture site (14 degrees) does not appear to affect the joint alignment and is acceptable. Use of a routine knee prosthesis will be possible in this patient. To avoid hardware removal, extramedullary jigs and/or computerized navigation may be used to measure and restore the long axis of the femur. The use of a hinged prosthesis does not influence the mechanical axis directly. Extra-articular osteotomy is occasionally needed to reverse severe deformities.

     

     

     

    2020

     

  89. A 67-year-old female with history rheumatoid arthritis presents with acute onset severe left hip pain eight years status-post total hip arthroplasty. She is unable to weight bear on the left leg, but denies any other pain or systemic symptoms. A current radiograph of the

    pelvis is shown in Figure A. What is the most likely cause of the patient's current hip pain symptoms?

     

     

     

     

     

    1. Poor surgical technique

    2. Rheumatoid arthritis flare

    3. Catastrophic implant failure

    4. Development of pelvic discontinuity

    5. Acute sepsis

     

    Corrent answer: 4

     

    Figure A demonstrates pelvic discontinuity. The radiographic findings associated with this diagnosis include a visible fracture line through the anterior and posterior columns, and medial translation and rotation of the inferior aspect of the hemipelvis relative to the superior aspect on an AP pelvic radiograph.

     

    Berry et al defined patients at risk for pelvic discontinuity, the typical radiographic findings, and success with revision surgical treatment. They found that patients with rheumatoid arthritis were at greatest risk for developing this condition. Also, those with minimal bone loss in which a socket can be satisfactorily supported by native bone without cement, a posterior column plate to stabilize the pelvis and a porous-coated socket is the preferred revision construct .

     

    Della Valle et al describe a comprehensive classification of periprosthetic fractures associated with THA a a guide for management in their ICL.

    Illustration A shows an example of a posterior column plate and a porous-coated socket.

     

     

     

     

     

     

     

    2020

     

  90. Figure 49 shows a histologic section of the lung in a patient who died during total hip arthroplasty. What unexpected finding is seen in the pulmonary capillaries?

     

     

     

     

     

    1. Pulmonary embolism

    2. Methylmethacrylate cement

    3. Hemorrhagic infarct

    4. Granuloma formation

    5. Amyloid

     

    Corrent answer: 2

    Sudden death during total hip arthroplasty has been reported. In a report from the Mayo Clinic, intraoperative death occurred during cemented total hip arthroplasty in 23 patients. Fat and marrow embolization during preparation of the femur or cementing of the femoral component was believed to be responsible for the cardiopulmonary collapse that occurred during arthroplasty. Although fat and marrow emboli were found in the pulmonary capillaries of most of the patients on autopsy, this histologic section shows two particles of cement in the pulmonary capillaries.

     

     

     

    2020

     

  91. After trial placement of components in a primary total knee arthroplasty, the knee is unable to come to full extension, but the flexion gap is appropriately balanced. After adequate soft-tissue releases have been performed, what is the next most appropriate action to balance the reconstruction?

    1. Use a larger femoral component

    2. Use a thinner polyethylene insert

    3. Add posterior femoral augments

    4. Resect more proximal tibia

    5. Resect additional distal femur Corrent answer: 5

    The reconstruction requires additional resection of the distal femur to allow increased extension while maintaining the current flexion gap tension.

    Resecting more proximal tibia or decreasing the tibial polyethylene thickness will decrease flexion tension as well as extension tension. Adding posterior femoral augments and using a larger femoral component will increase flexion tension.

     

     

     

    2020

     

  92. During total knee arthroplasty, the patella is noted to subluxate laterally despite a lateral retinacular release. Which of the following methods is most likely to improve patellar stability?

    1. Slight external rotation of the tibial component

    2. Slight internal rotation of the femoral component

    3. Slight anterior translation of the tibial component

    4. Use of a fixed-bearing knee as opposed to a mobile-bearing knee

    5. Use of a thicker patellar component

    Corrent answer: 1

     

    Slight external rotation of the tibial component will cause a net medialization of the tibial tubercle when the knee is articulated. This will help centralize the extensor mechanism over the trochlear groove and minimize the tendency for lateral subluxation. Internal rotation of the femoral component increases the risk of patellar instability. Anterior translation of the tibial component moves the patellar tendon insertion posteriorly, and may increase force on the patella but should not substantially alter patellar tracking. Clinical studies have shown no patellofemoral benefits to the use of fixed- or mobile-bearing designs.

    Thicker patellar components will not improve tracking, and may compound the problem.

     

     

     

    2020

     

  93. A 73-year-old man has stiffness after undergoing primary posterior cruciate ligament-retaining total knee arthroplasty 18 months ago. Extensive physiotherapy, dynamic splinting, and manipulations under anesthesia have failed to result in improvement. Examination reveals range of motion from 30 degrees to 60 degrees of flexion. The components are well fixed, and the evaluation for infection is negative. In discussing the possibility of revision arthroplasty, the patient should be advised that

    1. the success of improving range of motion to a functional range of 0 degrees to 90 degrees in the literature is between 75% to 80%.

    2. the preoperative arc of motion will not influence the ultimate range of motion after formal component revision.

    3. change from a posterior cruciate ligament-retaining to a posterior cruciate ligament-substituting design has a much greater chance of success.

    4. manipulation under anesthesia will effectively improve range of motion if postoperative stiffness develops following revision.

    5. the major postoperative focus will be to regain near full extension.

     

    Corrent answer: 5

     

    Stiffness following primary total knee arthroplasty remains a vexing problem. Treatment options have included extensive physical therapy, dynamic splinting, manipulation under anesthesia, arthroscopic arthrolysis, open arthrolysis with polyethylene exchange, and ultimately revision arthroplasty. Results are not as gratifying as would be expected. Babis and associates performed an open arthrolysis and polyethylene exchange on seven patients who were followed for a mean of 4.2 months. The results were poor. The mean improvement in arc of motion was only 20 degrees. Nicholls and Dorr treated

    13 patients for stiffness. Only 40% of those patients obtained good to excellent results. Four patients (30%) required manipulation because of recurrent stiffness postoperatively. They noted they could not predictably improve the arc of motion with a revision operation. Haidukewych and associates reported on 15 patients who underwent revision of well-fixed components after total knee arthroplasty for stiffness. Of the 15 patients, 10 (66%) were satisfied with the outcome revision. Interestingly, they noted that in patients for whom the total arc of motion did not improve but who regained near full extension, there was a greater amount of satisfaction with the procedure than for those who did not regain full extension.

     

     

     

    2020

     

  94. A 62-year-old patient is seen for routine follow-up after undergoing cementless total hip arthroplasty 2 years ago. The patient reports limited range of motion that severely affects daily activities. A radiograph is shown in Figure 51. Management should now consist of

     

     

     

     

     

    1. observation only.

    2. nonsteroidal anti-inflammatory drugs and protected weight bearing.

    3. irradiation to the affected area.

    4. surgical excision.

    5. surgical excision and postoperative irradiation.

     

    Corrent answer: 5

    The patient has symptomatic postoperative heterotopic ossification after total hip arthroplasty. Postoperative prophylactic treatments include nonsteroidal anti-inflammatory drugs (usually indomethacin) or low-dose irradiation. The heterotopic ossification shown here is quite mature; therefore, nonsurgical management will not be successful. Surgical excision of grade III or IV heterotopic ossification should be followed with postoperative irradiation to minimize the chances of recurrence.

     

     

     

    2020

     

  95. During impaction of a cementless acetabular component, the posterior column was fractured and found to be displaced. Which of the following is considered the most appropriate surgical option?

    1. Exchange of the cementless cup to a larger component

    2. Retention of the component and bone grafting of the fracture

    3. Retention of the component and postoperative weight protection until the posterior column heals

    4. Removal of the cup, fixation of the posterior column, and application of an antiprotrusio cage

    5. Removal of the cup and cementing of an all-polyethylene liner Corrent answer: 4

    Acetabular bone loss presents a challenge during reconstruction. A cementless hemispherical cup can be used in most patients provided that the acetabular rim, particularly the posterior column, is intact. When the posterior column is disrupted, fixation with a reconstruction plate and/or the use of an antiprotrusio cage is recommended. The latter is particularly important when the posterior column is fractured and displaced, such as in this patient. Under these circumstances, reduction of the fracture and application of an antiprotrusio cage is recommended. In this particular type of case, some surgeons may elect to retain the hemispherical cup and apply an antiprotrusio cage over the cup (“cage over cup” technique).

     

     

     

    2020

     

  96. Which of the following factors increases the risk of sciatic nerve injury in primary total hip arthroplasty (THA)?

    1. Male gender

    2. Anterolateral approach

    3. Posterior superior quadrant acetabular screw placement

    4. Osteonecrosis

    5. Developmental dysplasia of the hip Corrent answer: 5

    Injury to the sciatic nerve is a relatively rare but serious complication of THA. Dissection of the sciatic nerve is not typically done during primary THA, although the nerve can be identified during the surgical approach. An anterolateral approach to THA would not necessarily be associated with any greater incidence of sciatic nerve injury than other approaches. Screw fixation for the acetabular component is often a matter of surgeon preference.

    Provided that the anatomic safe zones for screw fixation (posterior inferior and posterior superior) are recognized, injury to the sciatic nerve from acetabular screws can be minimized. Restoration of anatomic length is important in primary THA. Overlengthening can result in sciatic nerve palsy. Developmental dysplasia of the hip can lead to a congenitally shortened extremity with concomitant congenital shortening of the associated neurovascular structures. Overlengthening of the extremity during THA for developmental dysplasia of the hip can lead to sciatic palsy. Osteonecrosis is not an associated risk factor for sciatic nerve palsy.

     

     

     

    2020

     

  97. A 68-year-old woman who underwent a right total hip arthroplasty 1 year ago has dislocated her hip five times since surgery. Radiographs show a retroverted acetabular component. What is the best treatment for this patient?

    1. Use a constrained acetabular liner

    2. Revise the femoral component to provide greater femoral offset

    3. Revise the femoral head from a 28-mm head size to a 36-mm head size

    4. Revise the acetabular component to 15 degrees of anteversion and 45 degrees of abduction

    5. Perform a greater trochanteric osteotomy to improve soft-tissue tension Corrent answer: 4

    The most common cause of recurrent dislocation following total hip arthroplasty continues to be component malposition. Component malposition should be addressed prior to any other treatment options, such as increasing soft-tissue tension with increased femoral offset or greater trochanteric advancement. A larger femoral head size may help, but correcting the component malposition should give more predictable results. A retroverted acetabular component should be revised to 15 degrees to 20 degrees of

    anteversion, matching the patient’s anatomy with an abduction angle close to 45 degrees.

     

     

     

    2020

     

  98. Figure 54 shows the preoperative radiograph of a 45-year-old woman who is considering total hip arthroplasty with her orthopaedic surgeon. What femoral characteristic is a typical concern in this patient?

     

     

     

     

     

    1. Osteopenia

    2. Excessive anteversion

    3. Excessive varus

    4. Excessive bowing

    5. Stove-pipe femur

     

    Corrent answer: 2

     

    Developmental dysplasia of the hip (DDH) leads to early arthritis of the hip as seen in this patient. Although DDH is believed to mostly affect the acetabulum, most patients with DDH also have anatomic aberrations of the femur. Using three-dimensional computer models generated by reconstruction of CT scans, dysplastic femurs were shown to have shorter necks and smaller, straighter canals than the controls. The shape of the canal became more abnormal with increasing subluxation. The studies also have shown that the primary deformity of the dysplastic femur is rotational, with an increase in anteversion of 5 degrees to 16 degrees, depending on the degree of subluxation of the hip. The rotational deformity of the dysplastic femur arises within the diaphysis

    between the lesser trochanter and the isthmus and is not attributable to a torsional deformity of the metaphysis. Osteopenia is not a concern in a patient with an excellent cortical index (thick cortices and narrow canal). Femoral varus or bowing of the femur is not a typical finding in patients with DDH.

     

     

     

    2020

     

  99. A 68-year-old man with no significant medical history underwent a total knee arthroplasty 4 years ago. A radiograph is shown in Figure

    55. He reports that he had no problems with the knee until 6 weeks ago when he noted the gradual onset of pain following a colonoscopy. Examination reveals a painful, swollen knee. Knee aspiration reveals a WBC count of 40,000/mm3. Management should consist of

     

     

     

     

     

    1. suppressive antibiotics.

    2. open irrigation and debridement with polyethylene exchange.

    3. one-stage resection arthroplasty and reimplantation.

    4. two-stage resection arthroplasty and reimplantation.

    5. arthroscopic irrigation and debridement.

     

    Corrent answer: 4

     

    The treatment of choice for a late hematogenous infection is two-stage resection arthroplasty and reimplantation, with parenteral antibiotics prior to reimplantation. This is particularly true when septic loosening has occurred as in this patient. Open irrigation and debridement with polyethylene exchange

    has been used successfully when the duration of symptoms is 3 weeks or less. Long-term suppressive antibiotics are most commonly used when the patient’s medical condition precludes further surgery. Delayed reimplantation has been shown to be superior to immediate reimplantation in multiple studies. Little data support the use of arthroscopic irrigation and debridement.

     

     

     

    2020

     

  100. A 58-year-old woman is seen in the emergency department after falling at home. History reveals that she underwent right total knee arthroplasty 2 years ago. Radiographs are shown in Figures 56a and 56b. What is the most appropriate treatment?

     

     

     

     

     

    1. Closed reduction and casting

    2. Bed rest and skeletal traction

    3. Open reduction and internal fixation

    4. Retrograde intramedullary nailing

    5. Revision of the femoral component with a stemmed component Corrent answer: 3

    The radiographs show an oblique periprosthetic distal femoral fracture. Of the options listed, open reduction and internal fixation is the most appropriate surgical option because a well-fixed, posterior stabilized closed box femoral component is present. Nonsurgical methods are not favored because of the highly displaced, unstable fracture pattern and prolonged immobility. Revision

    with a stemmed component is an option but would sacrifice more bone stock in this younger patient.

     

     

     

    2020

     

  101. Figures 57a through 57c show the radiographs of a patient who has pain, discomfort, and a popping sensation localized to the posterior aspect of the knee after undergoing primary left total knee arthroplasty 6 months ago. Examination reveals that the patient is able to ambulate without a limp. There is no significant swelling, erythema, or effusion. Range of motion is 0 degrees to 115 degrees, and a palpable crepitation or snapping is detected at the posterior lateral joint line. What is the most likely diagnosis?

     

     

     

     

     

     

    1. Popliteal snapping syndrome

    2. Patellar clunk syndrome

    3. Subluxation secondary to a tight posterior cruciate ligament

    4. Soft-tissue irritation secondary to retained polymethylmethacrylate

    5. Patellar subluxation secondary to a tight lateral retinaculum Corrent answer: 1

    Popliteal snapping syndrome represents the most likely diagnosis. Barnes and Scott noted that the popliteus tendon can be a potential source of internal derangement after total knee arthroplasty. They noted that it can be subluxated anteriorly and posteriorly over a retained lateral femoral condyle osteophyte. Allardyce and associates described the condition as a popliteus condition, snapping as it rolls over a retained lateral femoral condylar osteophyte. Patellar clunk syndrome is a distinct syndrome associated with the patella and has been reported in posterior stabilized knees. In addition to crepitation with range of motion, the patella literally snaps or jumps as the knee is taken from flexion to extension.

     

     

     

    2020

     

  102. Which of the following is the primary mechanism of polyethylene wear in the hip?

    1. Fatigue cracking and delamination

    2. Oscillatory fretting

    3. Crevice corrosion

    4. Oxidative degradation

    5. Adhesion and abrasion Corrent answer: 5

    Although previous theories on acetabular wear implicated fatigue cracking and delamination as primary wear mechanisms, these have actually manifested as major modes of polyethylene wear in knees. The primary mechanism of wear in polyethylene acetabular components appears to be adhesion and abrasion. In an analysis of 128 components retrieved at autopsy or revision surgery, wear appeared to occur mostly at the surface of the components and was the result of large strain plastic deformation and orientation of the surface layers into fibrils that subsequently ruptured during multidirectional motion. It was also shown conclusively that 32-mm heads displayed significantly more wear (volumetric wear) than either 22-mm or 26-/28-mm heads (1-mm increase in size increased volumetric wear by 10%). The wear at the articulating surface was characterized by highly worn polished areas superiorly and less worn areas inferiorly separated by a ridge. Abrasion was very common, occurring after adhesion and plastic deformation of polyethylene fibrils, and abrasion secondary to third-body wear. Wear rates decreased with longer survival of components, indicating a “bedding in” phenomenon, arguing against oxidative and fatigue wear. Crevice corrosion occurs in fatigue cracks with low oxygen tension (under screw heads, etc). Oscillatory fretting consists of cyclical abrading of the outer surface from small movements. Fatigue and delamination is predominant in total knee arthroplasty where stresses are

    maximum just below the surface of the polyethylene component, causing fatigue over time with subsequent delamination. In contrast, hip wear occurs primarily at the surface of the polyethylene component.

     

     

     

    2020

     

  103. Which of the following statements best describes results that have been reported with roentgen stereophotogrammetric analysis (RSA)?

    1. Cemented total hip stems do not migrate.

    2. Well-fixed total hip stems (cemented or cementless) migrate approximately 3 degrees and 5 mm in the first year.

    3. Any early migration (ie, greater than 0 mm less than 6 months after surgery) portends failure of the component.

    4. Migration greater than 1 mm to 2 mm in the first year is associated with a higher risk of loosening.

    5. The system has been proven to not be as accurate as claimed and has been abandoned.

    Corrent answer: 4

     

    Migration of total hip femoral components has been measured by RSA, a technique that affords accuracy of 2 degrees and 0.5 mm. Several published studies on total hip arthroplasty femoral components have established the importance of this technique. Both cemented and cementless components migrate, with the rate of migration suggesting the adequacy of fixation of a component. Migration of 1 mm to 2 mm (occurring in either the varus-coronal plane and retroversion-transverse plane, or both) has been associated with a higher risk of loosening of the component.

     

     

     

    2020

     

  104. Osteonecrosis of the large joints may develop in patients with which of the following conditions?

    1. Collagen I disease

    2. Antiphospholipid syndrome (APS)

    3. Hemochromatosis

    4. Achondroplasia

    5. Paget’s disease

     

    Corrent answer: 2

    Osteonecrosis of major joints can occur in patients exposed to corticosteroids, alcohol, and antiseizure medications, as well as patients with hemaglobulinopathy, such as sickle cell anemia. In addition, patients with primary APS who had not taken corticosteroids were also found to be at high risk for osteonecrosis of the hip. In one study of 30 patients with primary APS, asymptomatic osteonecrosis was evident in 20%. A recent article has also found a high association between idiopathic osteonecrosis of the hip and collagen II mutation. None of the other conditions has been shown to be associated with a higher risk of osteonecrosis.

     

     

     

    2020

     

  105. A 70-year-old female has persistent anterior knee pain and stiffness 10 months status-post total knee arthroplasty with associated lateral patellar release. Radiographs before and after surgery are shown in Figures A and B respectively. Pre-operatively, her Insall-Savati ratio is 0.95, compared to 0.76 post-operatively. Which of the following is the most likely cause of her radiographic abnormality and pain?

     

     

     

     

     

    1. Notching of the femur

    2. Excessive resection of the distal femur and lateral release of the patella

    3. Preoperative patella baja

    4. Excessive release of the patellar ligament from the tibial tubercle

    5. Excessive resection of the proximal tibia Corrent answer: 2

    Figure B represents iatrogenic patella baja and an elevated joint line caused by excessive resection of the distal femur and contracture of the patellar tendon likely as a result of lateral patellar release. Figure A does not demonstrate preoperative patellar baja, and answer choices 4 and 5 would lead to patella alta. The Insall-Savati ratio, shown in Illustration A, is the ratio of the patella tendon length to the length of the patella. A value <0.8 is cosistent with patella baja, and a value >1.2 is consistent with patella alta.

     

    Weale et al reviewed serial radiographs of 84 patients who had had either TKR or UKR to determine the change in patellar tendon length up to 5 years postoperatively. They found that tendon shortening was greatest in those knees which underwent TKA and had required a lateral release.

     

    Figgie et al correlated patellofemoral symptoms with the postion of the implant in 116 posterior stabilized condylar knee prostheses. They concluded that keeping the line of the prosthesis 8mm or less from the natural joint line keeps the flexors and extensors of the knee operating at their optimum length-tension relationship, and provided the best outcomes.

     

    Partington et al performed radiographic review of joint line position before and after revision total knee arthroplasty, and compared it with the joint line position before primary knee arthroplasty. They found that joint line elevation

    >8mm correlated with worse knee scores post-operatively, and distal femoral augments may be necessary in revision situations.

     

     

     

     

     

    2020

     

  106. What percentage of patients with complete peroneal nerve palsy after total hip arthroplasty will never recover full strength?

    1. 90% to 95%

    2. 60% to 65%

    3. 40% to 45%

    4. 20% to 25%

    5. 0% to 5%

     

    Corrent answer: 2

     

    The likelihood of complete functional recovery to preoperative strength levels is 35% to 40%.

     

    Nerve palsy is an uncommon but acknowledged complication of total hip replacement. The overall prevalence is approximately 1%. The sciatic nerve, or the peroneal division of the sciatic nerve, is involved in nearly 80% of cases. The risk of nerve palsy in association with total hip replacement is increased for female compared with male patients, with a diagnosis of developmental dysplasia, and with patients undergoing revision surgery. A preoperative diagnosis of developmental dysplasia of the hip or posttraumatic

    arthritis, the use of a posterior approach, lengthening of the extremity, and use of an uncemented femoral implant increased the odds ratio of sustaining a motor nerve palsy (in Farrell study). In the majority of cases, the origin of the palsy is unknown. Because peripheral nerves are sensitive to compression, unrecognized compression may play a role in these cases.

     

    Farrell performed a retrospective review of 27,004 primary THA’s done at Mayo. There was a 0.17% incidence rate of postoperative motor nerve dysfunction (29 pts with complete palsy, 18 with incomplete palsy). Of the twenty-eight patients with a complete palsy who were available for follow-up, only ten (36%) had complete recovery of motor strength, which took an average of 21.1 months.

     

    Another study by Schmalzried et. al states a prevalence of 1%, with the sciatic nerve involved in 80% of those palsies. Complete, or essentially complete, recovery occurs in approximately 41% and another 44% have only a mild deficit. Patients with some motor function immediately after the operation and those who recover some motor function within approximately 2 weeks of surgery have a good prognosis for recovery.

     

     

     

    2020

     

  107. A 68-year-old male undergoes the procedure shown in Figure A. After this particular procedure, when are nearly all patients' driving reaction times returned to their preoperative level?

     

     

     

     

     

    1. 3 months

    2. 2 months

    3. 4 weeks

    4. 2 weeks

    5. 1 week

     

    Corrent answer: 3

     

    In a patient who undergoes primary total hip arthroplasty, driving reaction time typically returns to preoperative levels in 4-6 weeks.

     

    Several factors can influence return to driving after total hip arthroplasty. In addition to the initial postoperative pain and weaning from narcotic medication, reaction time is also affected by side of surgery, extent of lengthening, and progression with rehabilitation. A small number of patients may convert from one-footed to two-footed drivers after surgery.

     

    Ganz et al. studied the driving reaction time of 55 patients aged 34-85 years old who underwent total hip arthroplasty using a simulation driving assessment. Physical therapy protocols were standardized. They found that reaction time worsened at the 1 week postoperative timepoint but then improved up to 1 year after surgery. Most patients returned to the preoperative reaction time in 4-6 weeks. Three patients in the study became two-footed drivers postoperatively.

     

    Cooper completed a systematic review of return to driving after orthopaedic surgical procedures and suggests that future research be conducted using established time frames as reference.

    Figure A is an AP pelvis radiograph showing a left total hip arthroplasty. Incorrect Answers:

    Answer 1,2,4,5: Best available data suggest that driving reaction time returns

    to preoperative levels in 4-6 weeks after total hip arthroplasty.

     

     

     

    2020

     

  108. Figure 6 shows a sagittal oblique MRI scan. The arrow is pointing to what structure?

     

     

     

    1. Bucket-handle tear of the medial meniscus

    2. Ligament of Humphrey

    3. Ligament of Wrisberg

    4. Posterior intermeniscal ligament

    5. Partial tear of the posterior cruciate ligament Corrent answer: 2

    The meniscofemoral ligaments connect the posterior horn of the lateral meniscus to the intercondylar wall of the medial femoral condyle. The ligament of Humphrey (arrow) passes anterior to the posterior cruciate ligament, whereas the ligament of Wrisberg passes posterior to the posterior cruciate ligament. One or the other has been identified in 71% to 100% of cadaver knees, with the ligament of Wrisberg being more common.

     

     

     

    2020

     

  109. Figure 8 shows the radiograph of a 76-year-old man who has knee pain and swelling. History reveals that he underwent total knee arthroplasty 18 years ago. What is the most likely diagnosis?

     

     

     

    1. Loose femoral component

    2. Loose tibial component

    3. Particle-mediated osteolysis

    4. Polyethylene failure

    5. Infection

     

    Corrent answer: 4

     

    The radiograph reveals complete loss of joint space with particulate metal debris consistent with total polyethylene failure and metal-on-metal articulation. The components appear to be well fixed and minimal osteolysis is evident.

     

     

     

    2020

     

  110. A 57-year-old man reports right hip pain that has been progressive for the past several months. The pain is exacerbated by weight-bearing activities and improves somewhat with rest. A radiograph is shown in Figure 10a and a coronal T1-weighted MRI scan is shown in Figure 10b. What is the most likely diagnosis?

     

     

     

     

     

     

    1. Osteoarthritis of the hip

    2. Osteonecrosis of the hip

    3. Metastatic carcinoma

    4. Femoral head fracture

    5. Rheumatoid arthritis of the hip Corrent answer: 2

    These are classic findings of osteonecrosis of the hip. The radiograph reveals the subchondral sclerotic pattern commonly seen in osteonecrosis and is quite extensive in this patient. The MRI scan reveals the typical serpentine-like region of low signal intensity with a central zone where the signal is similar to fat.

     

     

    2020

     

  111. Which of the following statements best describes the anatomic considerations of the popliteal artery posterior to the knee joint?

    1. It lies posterior to the popliteal vein and 9 mm posterior to the posterior aspect of the tibial plateau in 90 degrees of flexion.

    2. It lies anterior to the popliteal vein and 9 mm posterior to the posterior aspect of the tibial plateau in 90 degrees of flexion.

    3. It lies lateral to the popliteal vein and 15 mm posterior to the posterior aspect of the tibial plateau in 90 degrees of flexion.

    4. It lies medial to the popliteal vein and 9 mm posterior to the posterior aspect of the tibial plateau in 90 degrees of flexion.

    5. It lies anterior to the popliteal vein and 15 mm posterior to the posterior aspect of the tibial plateau in 90 degrees of flexion.

    Corrent answer: 2

     

    Popliteal artery injury during total knee arthroplasty is relatively rare. Knee flexion, the position that occurs during most of the arthroplasty procedure, allows the popliteal vessels to fall posteriorly, further away from harm.

    Anatomically, the popliteal artery lies anterior to the popliteal vein and 9 mm posterior to the posterior aspect of the tibial plateau in 90 degrees of flexion.

     

     

     

    2020

     

  112. At the level of tibial bone resection in total knee arthroplasty, where does the common peroneal nerve lie?

    1. Deep to the arcuate ligament

    2. Closer to bone in larger legs

    3. On the muscle belly of the popliteus

    4. On the bony posterolateral corner of the tibia

    5. Superficial to the lateral head of the gastrocnemius Corrent answer: 5

    At the level of tibial bone resection in total knee arthroplasty, the common peroneal nerve lies superficial to the lateral head of the gastrocnemius and is therefore protected by this structure. In an MRI study of 60 knees, the mean distance from the bony posterolateral corner of the tibia to the nerve was 1.49 cm, with no distance less than 0.9 cm. The distance from the bone to nerve was greater in larger legs.

     

     

    2020

     

  113. An 82-year-old man has had episodic right thigh pain after undergoing a total hip arthroplasty 10 years ago. Initial postoperative radiographs are shown in Figures 26a and 26b, and current radiographs are shown in Figures 26c and 26d. What is the most likely cause of his pain?

     

     

     

     

     

     

     

    1. Acetabular osteolysis

    2. Femoral osteolysis

    3. Acetabular loosening

    4. Femoral loosening

    5. Femoral and acetabular loosening Corrent answer: 4

    These radiographs are dominated by the subsidence of the femoral component. There is also evidence of polyethylene wear and femoral osteolysis in the region of the greater trochanter. There is no evidence of proximal (calcar) stress shielding, and there is a thick distal pedestal. Engh and associates defined two major signs of osseointegration - the absence of radiolucent lines around the porous-surfaced portion of the implant and new bone bridging the gap between the endosteal surface and the porous portion of the implant. Implant migration indicates failure of ingrowth. Osteolysis is a periprosthetic loss of bone secondary to particulate debris and it is often clinically silent unless it is accompanied by pathologic fracture. It is often globular. Acetabular loosening is based on radiolucent lines and implant migration. The current radiographs demonstrate subsidence of the stem with pedestal formation.

     

     

     

    2020

     

  114. Figure 36 shows an AP radiograph of a 65-year-old man who reports activity-related groin pain. History reveals that he underwent total hip arthroplasty 12 years ago. What is the most likely diagnosis?

     

     

     

     

     

    1. Chondrosarcoma

    2. Infection

    3. Wear-induced osteolysis

    4. Corrosive effect due to dissimilar metals

    5. Metastatic tumor

     

    Corrent answer: 3

     

    The AP radiograph demonstrates extensive periacetabular osteolysis. The central hole eliminator has dissociated from the shell and migrated into a lytic defect in the ischium. In a retrieval study, most periacetabular osteolytic lesions had a clear communication pathway with the joint space. Lesions with communication to the joint via several pathways or through a central dome hole (as in this patient) were larger and more likely to be associated with cortical erosion. Although periprosthetic tumors have been described, they are rare and particle-induced inflammation around a prosthesis does not seem to increase the risk for carcinogenesis.

     

     

     

    2020

     

  115. An 82-year-old woman reports activity-related knee pain. History reveals that she underwent total knee arthroplasty 16 years ago. AP and lateral radiographs and a bone scan are shown in Figures 38a through 38c. What is the most likely diagnosis?

     

     

     

     

     

     

    1. Particle-mediated osteolysis

    2. Metastatic carcinoma

    3. Stress shielding

    4. Septic joint

    5. Osteosarcoma

     

    Corrent answer: 1

    The radiographs reveal a large femoral metaphyseal lytic lesion with well-defined borders. Joint space narrowing medially is consistent with polyethylene wear. The most likely diagnosis is particle-mediated osteolysis. Metastatic tumors and primary sarcomas adjacent to an arthroplasty are extremely rare. In addition, malignant tumors and infection would more likely reveal a destructive lesion with poorly defined borders and increased uptake on a bone scan. Stress shielding with massive bone loss has not been described in knee arthroplasty literature, although this entity has been observed in fully porous-coated femoral implants in total hip arthroplasty.

     

     

     

    2020

     

  116. Bleeding is encountered while developing the internervous plane between the tensor fascia lata and the sartorius during the anterior approach to the hip. The most likely cause is injury to what artery?

    1. Ascending branch of the lateral femoral circumflex

    2. Superior gluteal

    3. Femoral

    4. Profunda femoris

    5. Medial femoral circumflex Corrent answer: 1

    The ascending branch of the lateral femoral circumflex artery crosses the gap between the tensor fascia lata and the sartorious and must be identified and ligated or coagulated. The other vessels are out of the field of dissection.

     

     

     

    2020

     

  117. A 60-year-old woman has activity-related hip pain after undergoing arthroplasty 5 years ago. She has severe Parkinsonism and denies fevers or chills. Radiographs are shown in Figures 45a and 45b. What is the most likely cause of her pain?

     

     

     

    1. Chronic deep infection

    2. Heterotopic bone

    3. Femoral loosening

    4. Parkinsonism

    5. Acetabular loosening

     

    Corrent answer: 3

     

    The radiographs reveal both cement debonding at the lateral shoulder of the prosthesis and a cement mantle fracture. Both of these indicate a loose femoral component. The radiographs show a stress fracture with reactive bone on the lateral femoral cortex in conjunction with the cement mantle fracture.

    The acetabular component shows no evidence of loosening. Heterotopic bone usually is not a source of pain when it is Brooker grade I, as in this case.

    Parkinsonism generally is not associated with hip pain.

     

     

     

    2020

     

  118. A 72-year-old man has had persistent pain after undergoing a hemiarthroplasty 18 months ago. Radiographs are shown in Figures 50a and 50b. What is the most likely cause of his problem?

     

     

     

     

     

     

    1. Suboptimal cement technique

    2. Excessive activity level

    3. Oversized bipolar component

    4. Infection

    5. Osteoporosis

     

    Corrent answer: 4

     

    The radiographs demonstrate a rapid erosion of the bipolar component into the acetabulum. Although acetabular erosion is more common with unipolar hip arthroplasties, it can occur with bipolar components. Haidukewych and associates noted a very low erosion rate but none in the first 2 years. The second finding on the radiographs is the linear radiolucency progressing from the joint toward the end of the stem at the cement-bone interface suggesting chronic infection or diffuse loosening. The persistent pain since implantation also suggests chronic infection. High activity levels and osteoporosis do not lead to acetabular erosion in the first 2 years after hemiarthroplasty. While the

    cement technique is suboptimal, loosening and erosion should not be expected from this alone. An oversized bipolar head would extrude and not erode.

     

     

     

    2020

     

  119. In an effort to reduce costs, a limited MRI sequence is planned to detect a possible occult hip fracture. What is the anticipated fracture signal?

    1. Bright on T<sub>1</sub> and T<sub>2</sub>

    2. Dark on T<sub>1</sub> and T<sub>2</sub>

    3. Dark on T<sub>1</sub>, bright on T<sub>2</sub>

    4. Bright on T<sub>1</sub>, dark on T<sub>2</sub>

    5. Enhancement by gadolinium Corrent answer: 3

    At present, radiologists perform multiple MRI images to rule out all possible diagnoses. The ability to specify the anticipated changes on MRI should become more important as a means of reducing costs. MRI is sensitive to changes in free water (or hemorrhage) and thus this will appear dark on Tand bright on T2.

     

     

     

    2020

     

  120. When using the direct lateral (or Hardinge) approach for hip arthroplasty, three muscles are detached from the femur. In addition to the vastus lateralis, they include the

    1. iliopsoas and sartorius.

    2. piriformis and obturator internus.

    3. gluteus maximus and tensor fascia lata.

    4. gluteus minimus and rectus femoris.

    5. gluteus medius and gluteus minimus.

     

    Corrent answer: 5

     

    This approach is criticized for the episodic limp associated with the muscle detachment and reattachment. Classically, two thirds of the gluteus medius is detached as a sleeve with the vastus lateralis. This exposes the gluteus minimus and the ligament of Bigelow. These must also be detached to allow dislocation of the hip and osteotomy of the femoral neck. The rectus femoris lies medially and anteriorly and does not need to be addressed. The piriformis and obturator internus are exposed during the posterior approach. Neither the

    gluteus maximus nor tensor fascia lata attach to the anterior femur. The sartorius and iliopsoas are not exposed during this dissection.

     

     

     

    2020

     

  121. A 40-year-old man complains of increasing groin pain. Radiographs show femoral head avascular necrosis with subchondral lucency but without femoral head collapse. Which of the following medical treatments have been shown to decrease the risk of subsequent femoral head collapse?

    1. Cyclic parathyroid hormone therapy

    2. Bisphosphonate therapy

    3. RANK ligand therapy

    4. RANK therapy

    5. Selective estrogen receptor modulator therapy Corrent answer: 2

    Bisphosphonate therapy is a proven method of preventing femoral head collapse in patients with avascular necrosis and subchondral lucency.

     

    Lai et al evaluated the effect of alendronate on patients with Steinberg stage-II or III osteonecrosis of the femoral head. They found that alendronate prevented early collapse of the femoral head at twenty-four months.

     

    Agarwala et al evaluated the effect of bisphosphonate therapy on patient reported and radiographic outcomes in femoral head avascular necrosis. They found alendronate reduces pain, improves function and may prevent disease progression at 5 year followup.

     

    Nishii et al evaluated the effect of alendronate on 20 hips with osteonecrosis of the femoral head without collapse. They found a lower frequency of collapse and less patient reported pain in patients treated with bisphosphonate therapy compared to controls at 12 month follow up.

     

    Thanks to Dr. Chris Rice at UW Madison:

    "Recent level 1 evidence seems to cast doubt on the efficacy of bisphosphonate treatment in precolapse AVN with medium to large lesions. There is also some thought that the supposed success seen in smaller lesions is due to the natural history of these lesions which often do not progress to collapse even in the absence of any treatment. "

    Lee YK, Ha YC, Cho YJ, Suh KT, Kim SY, Won YY, Min BW, Yoon TR, Kim HJ, Koo KH Does Zoledronate Prevent Femoral Head Collapse from Osteonecrosis?

    A Prospective, Randomized, Open-Label, Multicenter Study. J Bone Joint Surg Am. 2015 Jul 15.

     

    Incorrect Answers:

    Answer 1: Cyclic parathyroid hormone therapy is used in osteoporosis treatment, and not in the treatment of femoral head avascular necrosis. Answer 3 & 4: Neither RANK nor RANK ligand are being used in therapeutic forms currently. Denosumab, an anti-RANK ligand antibody, has shown early success in the treatment of bone lysis in oncologic applications.

    Answer 5: Selective estrogen receptor modulator therapy is used in osteoporosis, and not in the treatment of femoral head avascular necrosis.

     

     

     

    2020

     

  122. Which type of inflammatory cell is thought to be involved in the activation of the biological response to metal-on-metal particulate debris?

    1. Neutrophil

    2. Giant cell

    3. Macrophage

    4. Eosinophil

    5. Lymphocyte

     

    Corrent answer: 5

     

    In metal-on-metal articulations, it is thought that particulate wear debris stimulates lymphocytic invasion due to nanometer sized wear particles combining with native proteins to form haptens. These haptens then stimulate an immune response similar to a delayed type 4 hypersensitivity reaction via stimulation and activation of T-cells (lymphocytes).

     

    Metal-on-metal prostheses generate less volumetric wear than metal-on-poly prostheses, and produce smaller, nanometer-sized wear particles. Two types of adverse reactions have been noted in failed metal-on-metal articulations: 1) perivascular infiltrate of lymphocytes indicative of a delayed-type hypersensitivity response to metal wear products and 2) an aseptic, lymphocyte-dominated, vasculitis-associated lesion (ALVAL). The histopathology of pseudotumors or adverse local tissue reactions has been shown to be consistent with ALVAL.

     

    Heisel et al. reviews the alternative bearing choices for total hip arthroplasty. Ceramic and metal-on-metal bearings have demonstrated reduced wear rates compared to metal-on-poly articulations. However, long term clinical results

    have yet to determine any clinical improvement.

     

    Davies et al. evaluated periprosthetic tissue samples from 25 metal-on-metal total hip replacements and compared them to samples from metal-on-poly total hips and osteoarthritic hips. A unique lymphocytic infiltrate was demonstrated in the metal-on-metal hips. The body's immune response to metallic debris is different than its response to osteoarthritis and polyethylene debris.

     

    Plummer et. al demonstrates that ALVAL lesion severity doesn't necessarily correlate with metal ion levels (what we would assume would be higher wear). There does seem to be a predominance of monocytes in the synovial fluid however in those with ALVAL lesions -- again maybe the downstream effectors of the lymphocytic activation if the type 4 reaction is true.

     

    Illustration A demonstrates the annual linear wear rate of different material combinations used in total hip arthropalsty.

     

    Incorrect Answers:

    Answer 1: Neutrophils are typically stimulated as part of the response to infection not particulate wear debris.

    Answer 2 & 3: Macrophage, fibroblasts and giant cells are released in response to the particulate wear debris created from metal-on-polyethylene articulations not metal-on-metal articulations. They may be subsequently recruited by lymphocytes in delayed hypersensitivity reactions but they are not believed to be the initial activators of the response like lymphocytes are.

    Answer 4: Eosinophilic granuloma formation results from the clonal proliferation of Langerhans-type histiocytes and is not related to wear debris.

     

     

     

     

     

     

     

    2020

  123. When comparing unaged, radiated, cross-linked, ultra-high molecular weight polyethylenes (XPE) treated with remelting or annealing, annealed XPE has

    1. fewer free radicals, better mechanical characteristics, and lower wear than remelted XPE.

    2. fewer free radicals, worse mechanical characteristics, and lower wear than remelted XPE.

    3. more free radicals, better mechanical characteristics, and equivalent wear than remelted XPE.

    4. more free radicals, worse mechanical characteristics, and equivalent wear than remelted XPE.

    5. equivalent free radicals, better mechanical characteristics, and lower wear than remelted XPE.

    Corrent answer: 3

     

    Cross-linking polyethylene with radiation improves the wear properties. Persistent free radical formation within the crystalline areas of polyethylene can become oxidized in vivo. Remelting (heating above the melting point) or annealing (heating below the melting point) are processing techniques that decrease the retained free radicals. The advantage of remelting is more complete removal of persistent free radicals compared with annealing. The advantage of annealing is improved mechanical properties by avoiding disruption of the crystalline areas that occurs with remelting. Other free radical squelching methods (eg, addition of the free radical scavengers such as vitamin E) are being used to achieve both goals (free radical removal and maintenance of improved mechanical properties). To date, no significant difference has been demonstrated in wear rates between the two materials.

     

     

     

    2020

     

  124. Which of the following is considered a contraindication for a constrained polyethylene insert for the management of hip instability?

    1. Well-positioned cup in an elderly patient

    2. Poorly positioned cup

    3. Absent abductor mechanism

    4. Placement of an acetabular cage

    5. Central neurologic decline Corrent answer: 2

    Hip instability is a complication of primary and revision hip arthroplasty. The risk of dislocation ranges from 0.5% to 6% in primaries and 2% to 20% in

    revisions. There are several risk factors for dislocation including patient factors (Parkinsons, dementia, spasticity, alcoholism, previous hip surgery, osteonecrosis, obesity, hip fractures) and surgical factors (head size, restoration of leg length and offset, impingement, surgeon experience, approach). Surgical treatment of instability includes use of larger femoral heads, optimizing implant position, addressing sources of impingement, and increasing offset and/or leg length. The relative indications for constrained liners include an absent abductor mechanism, recurrent dislocation in the presence of well-positioned implants, and failure of nonconstrained surgical solutions. The threshold to choose a constrained liner is lower in elderly, lower demand, or neurologically compromised patients. The risks of constrained liners include implant loosening, limited motion, and constraint mechanism failure. Relative contraindications for a constrained liner include a malpositioned implant or a young patient with a large inherent range of motion.

     

     

     

    2020

     

  125. A 67-year-old man who underwent knee arthroplasty 7 years ago now reports the recent onset of pain and swelling. He denies any recent illness or change in medical status. Examination of the knee reveals a moderate effusion, slight warmth when compared to the contralateral knee, and no tenderness. Pain is increased with weight bearing. Laboratory studies show a WBC count of 6,300/mm3 (normal 3,500-10,500/mm3), a hemoglobin level of 10.1 g/dL, an erythrocyte sedimentation rate of 25 mm/h (normal up to 20 mm/h), C-reactive protein (CRP) level of 0.3 mg/dL (normal less than 0.5 mg/dL), and a uric acid level of 6.2 mg/dL (normal 2.5-7.0 mg/dL). Postoperative and current radiographs are shown in Figures 8a and 8b. What is the most likely diagnosis?

     

     

     

     

     

    1. Infection

    2. Aseptic loosening

    3. Osteolysis

    4. Dislodgement of polyethylene

    5. Crystalline arthropathy

     

    Corrent answer: 3

     

    Osteolysis is being reported in the literature with increasing frequency. The incidence of osteolysis may be related to some variability in polyethylene manufacture, quality, sterilization, or storage. Increased rates have been associated with gamma sterilization in air, prolonged time on the shelf prior to implantation, and certain implant designs. Infection is unlikely given the near normal erythrocyte sedimentation rate and CRP. There are no clear signs of loosening or disruption of the locking mechanism on the radiographs.

    Crystalline arthropathy is possible but does not explain the lytic lesions on the radiographs.

     

     

     

    2020

     

  126. When performing a unicondylar knee replacement, a smaller incision without dislocation of the patella offers what advantage over a standard, patella-everting approach?

    1. the option to convert to a total knee arthroplasty if needed

    2. more anatomic positioning of the components

    3. better ultimate range-of-motion

    4. increased 10-year implant survival rate

    5. improved rate of recovery Corrent answer: 5

    Price et. al. compared 40 unicondylar knee arthroplasty patients with small incisions and no patella dislocation to 20 UKAs (unicondylar knee arthroplasties) with standard incisions and 40 TKAs (total knee arthroplasties)

    . The average rate of recovery (measured by the time taken to achieve straight-leg raising, 70 degrees of flexion, and independent stair climbing) following the small incision UKAs was twice as fast as after standard UKAs and three times as fast as TKAs. Accuracy of implantation was the same for the two UKA groups.

     

     

     

    2020

  127. A medial parapatellar approach is used to perform a total knee arthroplasty, including patellar resurfacing. The patient now reports anterior knee pain with stair climbing and rising from a chair. Radiographs show lateral subluxation of the patella on the Merchant view. Which of the following is a potential cause for the problem?

    1. External rotation of the tibial component

    2. Posterior translation of the femoral component

    3. Decreased composite thickness of the resurfaced patellar component

    4. Lateral placement of the tibial component

    5. Internal rotation of the femoral component Corrent answer: 5

    Lateral subluxation of the patella following total knee arthroplasty has several possible causes, including internal rotation of the femoral component, internal rotation of the tibial component, and increasing the patellofemoral space (either increasing the composite thickness of the patella or anterior translation of the femoral component). Increasing the patellofemoral space leads to increased tension on the lateral patellofemoral retinaculum when a medial parapatellar approach is used. Increased tension in the lateral soft-tissue tether can cause lateral subluxation. Placement of the femoral and tibial components to the most lateral aspect of the cut surfaces helps to ensure the tibial tubercle is medialized appropriately to avoid lateral subluxation of the patellar component.

     

     

     

    2020

     

  128. In patients with a Trendelenburg gait, which of the following gait patterns is exhibited?

    1. The pelvis on the swing side drops, causing increased adduction of the affected hip during the stance phase.

    2. The pelvis on the swing side drops, causing decreased adduction of the affected hip during the stance phase.

    3. The pelvis on the swing side elevates, causing increased adduction of the affected hip during the stance phase.

    4. The pelvis on the stance side elevates, causing decreased adduction of the affected hip during the stance phase.

    5. The pelvis on the swing side remains neutral, but there is circumduction of the affected hip.

    Corrent answer: 1

    At the hip, one of the most common abnormal kinematic patterns is the Trendelenburg gait pattern that occurs in patients with weakened hip abductors. The pelvis on the swing phase of gait drops, and this leads to increased adduction of the affected hip during the stance phase. Patients with weakened hip abductors should hold a cane in their contralateral hand to assist with support of the pelvis during this phase of gait. In the absence of a cane, patients will often lean their torso toward the affected side. This shifts the center of gravity closer to the affected hip, decreasing the moment arm force required by the hip abductors.

     

     

     

    2020

     

  129. A lateral radiograph obtained following a posterior stabilized total knee arthroplasty is shown in Figure 15. The complication seen could be best explained by which of the following?

     

     

     

     

     

    1. Too little posterior tibial slope

    2. Posterior translation of the femoral component

    3. Inadvertent injury to the popliteus while cutting the posterolateral femoral condyle

    4. Overly recessing the posterior cruciate ligament

    5. Elevation of the joint line Corrent answer: 3

    Dislocation of the knee in posterior stabilized total knee arthroplasty occurs when the cam on the femoral component is translated anterior to the central tibial post. Excessive flexion laxity is the most common etiology of this complication. The posterior cruciate ligament is a stabilizer in flexion and is generally removed as part of the technique. The popliteus is also a stabilizer in flexion and should be protected. Inadvertent injury to the popliteus can lead to an imbalance and flexion laxity. Other surgical factors that lead to greater flexion laxity include undersizing the femoral component, anterior translation of the femoral component, and additional posterior tibial slope. Changes in the location of the joint line have an effect on both extension and flexion stability.

     

     

     

    2020

     

  130. A 40-year-old man has moderate lateral compartment arthritis several years after undergoing a partial lateral meniscectomy. He has a correctable 5 degree valgus knee deformity compared to his other limb. His patellofemoral and medial compartments do not show any radiographic signs of degenerative changes. His knee has full range of motion and is stable on exam. After failing nonoperative treatments, which surgical option is most likely to give him the best outcome?

    1. Valgus producing high tibial osteotomy

    2. Varus producing distal femoral osteotomy

    3. Total knee replacement

    4. Arthroscopic debridement and chondroplasty

    5. Tibial tubercle osteotomy with anteromedialization Corrent answer: 2

    A varus producing distal femoral osteotomy would likely give this patient the best outcome correcting his valgus deformity and unloading his lateral arthritic compartment.

     

    Classic contraindications to an osteotomy include inflammatory arthropathy, cruciate deficiency, flexion contracture >15 deg, or <90 deg flexion.

     

    Wang et al performed a retrospective study on 30 patients with a varus producing distal femoral osteotomy for valgus deformities with lateral compartment arthritis and found 83% satisfaction at nearly 8 years follow up. Failures were revised to a total knee arthroplasty.

     

     

     

    2020

  131. Following an uneventful primary total knee arthroplasty, an otherwise healthy 60-year-old woman does well for 3 years. She is now being evaluated for new knee pain that started 5 weeks ago following an upper respiratory tract infection. Examination reveals an effusion in the knee, the C-reactive protein is elevated, and an aspiration reveals a WBC count of 15,000/mm3 and gram-positive cocci. What is the next most appropriate step in management?

    1. Serial aspirations and antibiotics

    2. Evaluation for crystal disease

    3. Arthroscopic irrigation and debridement and antibiotics

    4. Open irrigation and debridement, polyethylene exchange, and antibiotics

    5. Resection of the knee implant and antibiotics Corrent answer: 5

    The clinical scenario is one of a previously well-functioning knee arthroplasty with symptoms starting following an upper respiratory illness. The temporal onset of the symptoms raises the possibility of a hematogenous infection to the knee. Timing of treatment is considered an important factor in the outcome of acute infections around implants. Surgical irrigation and debridement (preferably with a polyethylene exchange) can be successful if performed within the first 4 weeks (preferably within 2 weeks) of symptoms. Success of irrigation and debridement with retention of the implants decreases significantly with durations of greater than 2 to 4 weeks and therefore management generally requires removal of the implants. WBC counts of greater than 2,500/mm3 to 3,000/mm3 in an aspirate from a joint arthroplasty should increase the suspicion for an infection. Crystal disease can be a source for an effusion and pain in total knee arthroplasty; however, it is important to rule out infection since timing directly impacts the success of surgical treatment. In this patient, the aspirate also showed a positive Gram stain, suggesting infection. Serial aspiration and arthroscopic debridement for the management of septic processes involving arthroplasty implants generally are not recommended.

     

     

     

    2020

     

  132. An 84-year-old woman has recurrent dislocation following hip arthroplasty. A radiograph obtained following revision with a cemented constrained liner is shown in Figure 23a. One year later she falls and dislocation occurs again as seen in Figure 23b. All of the following factors are important in avoiding this complication EXCEPT

     

     

     

     

     

    1. scoring the polyethylene.

    2. scoring the inside of a smooth shell.

    3. closely matching the size of the liner and shell.

    4. fully seating the liner.

    5. positioning the liner to limit or avoid impingement.

     

    Corrent answer: 3

     

    The outer diameter of the liner needs to be several sizes smaller than the shell to allow adequate room for cement. Revision for instability in the setting of a well-fixed acetabular component can be accomplished by cementing a new liner into the existing shell and therefore avoiding potential damage to the pelvis during extraction of the shell. This is particularly important when there is significant osteolysis behind the shell that might make reconstruction difficult. Biomechanical studies of liners cemented into well-fixed shells have shown that, when done well, the interface can be stronger than most acetabular locking mechanisms. Failures have occurred when the liner or shell was too smooth to allow inter-digitation of the cement or when there was not room for an adequate cement mantle. A liner that is too large will not be captured by the shell (as in this patient) and can result in failure.

    Impingement of the femoral component on the liner or shell can also lead to early failure of the construct.

     

     

     

    2020

     

  133. A 55-year-old electrician who is 5' 8" and weighs 243 lbs has severe pain in the medial side of his knee. Six months ago he underwent an arthroscopy and partial medial meniscectomy that resulted in very little improvement in symptoms. Radiographs confirmed medial femoral condyle osteonecrosis and he underwent a successful medial unicondylar knee arthroplasty and returned to work in 3 weeks. Radiographs are shown in Figures 29a and 29b. Four months after this surgery, he now reports the gradual onset of pain and swelling for the past 2 weeks. Laboratory studies, including a CBC count, erythrocyte sedimentation rate, CRP, and uric acid level, are all normal. Synovial fluid analysis reveals a WBC count of 6,500/mm3 and a RBC count of 45,000/mm3. He has difficulty weight bearing and is unable to work. What is the most likely diagnosis?

     

     

     

     

     

     

    1. Hematogenous infection

    2. Component loosening

    3. Crystalline arthropathy

    4. Synovial entrapment

    5. Stress fracture

     

    Corrent answer: 5

     

    Unicondylar knee arthroplasty can have a high success rate; however, early complications have been reported. One such complication is a stress fracture of the tibial plateau. This more commonly occurs in patients who weigh more than 200 lb and has been associated with pin placement during surgery and technical error during the sagittal cut in the tibia. Hematogenous infection or infection from the index surgical procedure is a possibility, but would present with an abnormal ESR and CRP and a synovial WBC typically greater than 20,000/mm3. Loosening is unlikely and is not evident on the radiographs.

    Crystalline arthropathy is also possible, but unlikely given the laboratory values and the synovial analysis. Synovial entrapment can occur and cause pain and a bloody effusion, but it is unlikely to cause such a dramatic change in symptoms and the ability to bear weight.

     

     

     

    2020

     

  134. Which of the following neurovascular structures is closest to the posterior capsule of the knee?

    1. Popliteal vein

    2. Popliteal artery

    3. Tibial nerve

    4. Common peroneal nerve

    5. Sural nerve

     

    Corrent answer: 2

     

    The order from superficial to deep (as well as from lateral to medial) is tibial nerve, popliteal vein, and popliteal artery. The sural nerve is subcutaneous in the posterior calf and the peroneal nerve is posterolateral.

     

     

     

    2020

     

  135. A 78-year-old man who is an alcoholic is seen for follow-up. History reveals that he underwent a total hip arthroplasty 12 years ago. He has thigh pain but no functional limitations. Laboratory studies show that the erythrocyte sedimentation rate and CRP are within normal limits. Radiographs are shown in Figures 32a through 32c. What is the most appropriate management?

     

     

     

     

     

     

     

     

    1. Continued regular follow-up radiographs

    2. Recommend the use of a cane

    3. Bisphosphonate therapy

    4. Revision to a long-stem cemented prosthesis

    5. Revision to a long-stem cementless prosthesis Corrent answer: 5

    Circumferential radiolucency of the femoral stem with a cement mantle fracture is diagnostic for osteolysis and loosening. Due to the thinning of the cortex, the patient should undergo revision soon to avoid the inevitable periprosthetic fracture. Follow-up radiographs should be obtained in all patients with total joint arthroplasty. When wear, radiolucencies, or osteolysis become apparent, radiographs should be obtained more frequently.

    Bisphosphonate therapy has been proposed to slow the progression of osteolysis but, so far, this remains unproven and would have no effect in this patient because the stem is already loose. A cane offers inadequate protection, and the patient should be encouraged to use a walker until revision can be performed.

     

     

     

    2020

  136. What is the most common complication after revision of a total hip polyethylene liner in a patient with well-fixed femoral and acetabular shell components?

    1. dislocation

    2. failure of the femoral component

    3. extensive osteolysis

    4. failure of the fixation between the liner and the acetabular shell

    5. fracture of the polyethylene Corrent answer: 1

    Multiple studies have shown the most common complication of revision surgery with polyethylene exchange is dislocation.

     

    Lachiewicz et al reviewed 35 hips with 1st generation components (Harris-Galante porous HGP I and II acetabular components) underwent polyethylene exchange without cement. There were no instances of loosening but 7 hips dislocated.

     

    Boucher et al assessed 24 hips which underwent polyethylene liner exchange and found 25% dislocated at least once.

     

    Beaule et al examined 32 hips which underwent polyethylene exchange by cementing a new liner into the existing fixed cementless acetabular component and found 22% dislocated.

     

    Incorrect Answers: The other complications listed were noted at much lower rates in all 3 studies if at all.

     

     

     

    2020

     

  137. A patient who underwent total hip arthroplasty through a modified Hardinge approach 2 years ago now reports a persistent limp and pain over the trochanter. A CT scan is shown in Figure 59. What is the most likely cause of the problem?

     

     

     

    1. Trochanter fracture

    2. Splitting the gluteus medius fibers more than 5 cm from the tip of the trochanter

    3. Aberrant retractor placement

    4. Inadequate physical therapy

    5. Inadequate restoration of offset Corrent answer: 2

    The superior gluteal nerve courses between the gluteus medius and gluteus minimus approximately 4 cm to 6 cm superior to the tip of the trochanter. The Hardinge approach is an intranervous approach with the superior gluteal nerve at risk in the superior extent of the approach. Splitting of these muscles should be limited to 4 cm to avoid injuring the innervation to the anterior portion of the gluteus medius/minimus and the tensor fascia lata. Avulsion of the abductor repair can occur and be a source of weakness; however, one would not expect the fatty replacement seen in the scan. Aberrant retractor replacement can lead to nerve injury with the Hardinge approach, particularly aberrant placement of the anterior retractor leading to a femoral nerve injury.

     

     

     

    2020

     

  138. Risk factors for a motor nerve palsy following primary total hip arthroplasty include all of the following EXCEPT?

    1. Developmental dysplasia of the hip

    2. Limb lengthening

    3. Posttraumatic arthritis

    4. Obesity

    5. Posterior approach

     

    Corrent answer: 4

     

    Certain pre-operative risk factors may place patients at increased risk for the development of nerve injury during primary total hip arthroplasty (THA).

    These include a history of developmental dysplasia of the hip (DDH), limb lengthening, posttraumatic arthritis, cementless femoral fixation, and the posterior approach.

     

    Farrell et al. reviewed more than 27,000 patients who underwent THA between 1970 and 2000. Forty-seven patients developed a nerve palsy (0.17%) with 29 complete palsies and 18 incomplete palsies. DDH, post-traumatic arthritis, posterior approach, use of a cementless femoral implant, and limb-lengthening were all found to have an increased risk of nerve palsy. Body-mass index (BMI) was not associated with increased risk of nerve palsy.

     

    Of note, the average time to maximal recovery for both incomplete and complete nerve palsies was approximately 2 years. Patient should be counseled regarding this complication especially if one of the above risk factors is present.

     

     

     

    2020

     

  139. A patient with a first-generation cementless acetabular component has polyethylene wear and pelvic osteolysis. Both components appear well-fixed. At surgery it is noticed that two of the locking tines for the liner are broken. You choose to cement a new polyethylene liner and bone graft the osteolytic lesion. What is the most common complication?

    1. Failure of acetabular fixation

    2. Polyethylene wear

    3. Progression of osteolysis

    4. Dislocation

    5. Fixation failure of the polyethylene to the shell Corrent answer: 4

    The clinical success of the Harris-Galante porous hemispherical cups as well as other similar cementless acetabular components has led to a relatively common scenario in revision surgery where the fixation is good and the bone has remodeled. In these situations, with a well ingrown cup and a poor or

    damaged locking mechanism, cementation of a liner into the stable cup has been shown to be an effective alternative to cup removal. In cups without holes, texturing of the surface improves pull-out strength with a cemented liner as does multidirection texturing of the polyethylene, 2-mm cement mantle, and containment of the undersized polyethylene within the shell. This alternative allows the use of newer highly crosslinked polyethylene for improved wear characteristics and arrest of the osteolytic process. The most common complication for isolated polyethylene exchanges (with or without cement fixation) is postoperative instability.

     

     

     

    2020

     

  140. A nondisplaced periprosthetic small posterior wall acetabular fracture is noted intra-operatively during total hip arthroplasty. The acetabular component is stable and well-fixed after implantation of an ingrowth acetabular shell during intraoperative examination. Which of the following treatment options will best maintain motion and clinical function?

    1. ORIF of the posterior column and THA revision

    2. Cage reconstruction of acetabular component

    3. THA revision using a cemented acetabular component

    4. Placement of a hip abductor brace and non-weight bearing in the affected limb

    5. No change in treatment Corrent answer: 5

    Acetabular fracture during total hip arthroplasty is a known complication that typically occurs during acetabular component impaction. If noticed intra-op, the stability of the component should be assessed by the surgeon to determine treatment. If the component is stable, no additional treatment is necessary. If the acetabular component is unstable, then it should be changed and/or supplemented with component screws until stability is obtained.

     

    The referenced article by Haidukewych showed no need for acetabular fixation after fracture as long as the acetabular component was deemed stable. In their 7121 cases studied, they had a fracture rate of 0.4% and a high rate of fracture with elliptical monoblock cups. All fractures healed and the acetabular components achieved ingrowth without migration.

     

     

     

    2020

  141. In assessing the knee after acute injury, increased tibial external rotation at 30 degrees of knee flexion indicates injury to what structure?

    1. Medial collateral ligament

    2. Anterior cruciate ligament

    3. Posterior cruciate ligament

    4. Posterolateral corner

    5. Posteromedial capsule

     

    Corrent answer: 4

     

    Increased external rotation at 30 degrees indicates injury to the posterolateral corner of the knee. Increased external rotation at both 30 degrees and 90 degrees indicates injury to both the posterolateral corner and the posterior cruciate ligament.

     

     

     

    2020

     

  142. Figure A is a photograph of a femoral head removed during revision of a ceramic on ceramic total hip arthroplasty. Which of the following is the most likely indication for this revision total hip arthroplasty?

     

     

     

     

     

    1. Prosthetic infection

    2. Recurrent dislocations

    3. Leg length discrepancy

    4. Ceramic bearing fracture

    5. Aseptic lymphocytic-vasculitis-associated lesion (ALVAL) Corrent answer: 2

    The markings on the femoral head have previously been described as resembling a pencil lead mark. Its presence indicates transfer of metal from the acetabular cup to the ceramic femoral head. This occurs during the

    component on component impingement and excursion that occurs during dislocation.

     

    Kim et al conducted a retrieval analysis of 15 ceramic femoral heads that had metallic smearing as a result of dislocation. They found statistically significantly more femoral head surface roughness and increased polyethylene liner wear rates as the percentage of metallic smearing surface area increased. Scanning Electron Microscopic (SEM) evaluation commonly showed pits and scratches on the femoral head (Image A) and stereoscopic microscopy shows embedded metallic debris acting as a third body in the polyethyelene.

     

     

     

     

     

     

     

    2020

     

  143. All of the following are contraindications for a medial unicompartmental knee replacement EXCEPT?

    1. ACL deficiency

    2. Anterior compartment osteophytes and pain beneath the patella

    3. Medial sided knee pain with medial and lateral compartment osteophytes and 3 degrees of varus deformity

    4. Lateral sided knee pain with degenerative changes isolated to the medial compartment and 7 degrees of varus deformity

    5. Flexion contracture of 3 degrees

     

    Corrent answer: 5

     

    Unicompartmental knees rely on normal knee biomechanics to preserve motion and stability at the knee. The indications for unicondylar knee replacements are evolving. Using the original published criteria, indications for

    unicondylar replacement include: non-inflammatory arthritis, mechanical axis deviation <10 degrees from neutral in varus or 5 degrees in valgus, intact ACL, flexion contracture <15 degrees, no patellofemoral symptoms (although radiographic arthritis alone at the PF joint is not a contraindication), flexion to 90 degrees or greater, age >60, low activity demand, and weight <82 kilograms.

     

    In a study of 10 year follow up, Murray et al, defined the following indications for UKA: fixed flexion deformity <15 degrees, correctable varus deformity (indicating a competent medial collateral ligament), full-thickness cartilage in the lateral compartment, and non-inflammatory arthritis. Patellofemoral arthritis is not a contraindication, nor is moderate obesity.

     

    The second Murray reference is a review of the indications and techniques for mobile bearing unicompartmental knee arthroplasty as mentioned above.

     

    Though not noted in the cited articles, OKU mentions that the presence of patellofemoral arthritis is not necessarily correlated with UKA failure, though pain with arthritis is. Finally, all indications for UKA include a competent ACL. Without an ACL, normal biomechanics cannot be present and the surgery fails.

     

     

     

    2020

     

  144. Figure A displays the preoperative radiographs of a 67-year-old obese, diabetic woman undergoing total knee arthroplasty. Following the bone resections, a PCL-substituting trial implant is placed. At full extension, 30 degrees of flexion, and 90 degrees of flexion, the knee is found to be tight laterally and loose medially. Sequential releases are made until the IT band, popliteus, and LCL have all been released, and a larger polyethylene trial is placed, but the knee continues to demonstrate medial laxity and lateral tightness. Which of the following should be done to achieve a successful outcome?

     

     

     

    1. Surgical release of the lateral retinaculum

    2. Surgical release of the lateral retinaculum and reconstruction of the medial patellofemoral ligament

    3. Imbrication of the MCL and augmentation with allograft

    4. Convert to a constrained TKA design

    5. Recut the tibia in valgus to improve balance Corrent answer: 4

    The radiograph demonstrates a Charcot arthritic knee with severe valgus deformity. The surgeon is unable to achieve balancing through soft tissue release, and thus should convert to a constrained design (either constrained condylar or hinge).

     

    Semi-constrained knees or constrained condylar knees rely on a large tibial post to enhance varus/valgus stability. Posterior-stabilized knees use a smaller post and femoral box design to improve femoral rollback, but are not sufficient to provide varus/valgus stability in a grossly unstable knee.

     

    Sculco et al reviews constraint in TKA. Soft tissue balancing is the first step to achieving a stabilized TKA. Most primary TKA's will become stabilized with appropriate soft tissue procedures and the use of a PCL-retaining or posterior stabilized TKA. Patients with severe deformity and revision may not attain adequate stabilization with soft tissue procedures alone and require a

    constrained TKA design to supplement varus/valgus stability. Finally, in the patient with severe bone loss and gross instability, a rotating hinge knee may be necessary to achieve stability. Changing the mechanical axis of the knee or recuting the tibia in extreme varus or valgus should not be done to balance knee as it may increase risk of aseptic loosening - an already elevated risk for the Charcot knee.

     

    Lu et al reviewed 4 patients (5 knees) who underwent TKA for charcot arthropathy. At 5-year follow-up all 5 knees were doing very well with no complications, no secondary surgery, and satisfactory clinical outcomes.

     

    Illustrations A and B are radiographs and photographs of a constrained knee system demonstrating the tibial post resting in the femoral box.

     

     

     

     

     

     

     

     

    2020

  145. During the course of a revision total knee arthroplasty via a medial parapatellar exposure, the surgeon does a complete intra-articular release and synovectomy but exposure is still inadequate. A quadriceps snip is performed and, at the end of the procedure, the knee is stable throughout a range of motion and the postoperative radiographs show acceptable alignment of the components. The patient's postoperative physical therapy regimen should include which of the following?

    1. No restriction in range of motion or weight bearing after surgery.

    2. Limit flexion to 30 degrees postoperatively, progressing 10 degrees per week

    3. Limit flexion to 90 degrees for the first 6 weeks postoperatively

    4. Limit to active flexion only with no passive flexion or active extension for 6 weeks

    5. Use of a hinged knee brace for 6 weeks postoperatively Corrent answer: 1

    A quadriceps snip is performed by extending a medial parapatellar approach superiorly and laterally across the quadriceps tendon. It is then repaired primarily at the end of the procedure. The primary advantage of this technique over other surgical maneuvers that improve exposure at the time of revision total knee arthroplasty is that the postoperative regimen for physical therapy does not need to be altered.

     

     

     

    2020

     

  146. A healthy 72-year-old woman is seen 14 days after cemented total knee arthroplasty. She reports increasing pain and swelling for the last 4 days accompanied by 4 days of wound drainage. Examination reveals that she is afebrile, and has erythema and moderate serosanguinous drainage from the wound. The knee is moderately swollen. Aspiration of the knee reveals no organisms on Gram stain. Culture results are expected back in 48 hours. Optimal management should consist of

    1. initiation of a first-generation cephalosporin while awaiting culture results.

    2. initiation of broad-spectrum antibiotics while awaiting culture results.

    3. ultrasound to evaluate for fluid collection around the knee.

    4. surgical debridement of the knee before culture results are available.

    5. inpatient observation and no antibiotics until culture results are available.

     

    Corrent answer: 4

    Increased pain, swelling, erythema, and drainage 2 weeks removed from the primary arthroplasty are all signs of a probable infection. Erythrocyte sedimentation rate and C-reactive protein may not be helpful as they are elevated postoperatively even in the absence of infection. Even in the absence of infection, persistent wound drainage is an indication for surgical debridement to prevent subsequent infection. When a postoperative infection is easily recognized by clinical examination, there is no need to wait for a positive culture before proceeding with debridement.

     

     

     

    2020

     

  147. A patient with a history of rheumatoid arthritis reports a painful total hip arthroplasty 3 years after the index procedure. Radiographs reveal loosening of the femoral component. Preoperative blood work shows an erythrocyte sedimentation rate (ESR) of 38 mm/h (normal 0-29 mm/h) and a C-reactive protein (CRP) of 8.9 (0.2-8.0). What is the most appropriate action at this time?

    1. Technetium bone scan

    2. Hip aspiration for culture

    3. FDG-PET scan

    4. Surgery with no further investigations

    5. Revision surgery and obtain an intraoperative frozen section Corrent answer: 2

    The question centers on the appropriate work-up for a failed total hip arthroplasty prior to revision surgery. The preoperative ESR is elevated and the CRP is at the upper end of normal. If either the ESR or CRP is elevated, further investigations are required to exclude infection as a cause of loosening, particularly in a patient only 3 years after the index procedure. A technetium scan alone is nonspecific and will show increased uptake because of the loose femoral component. An intraoperative frozen section is a helpful confirmatory investigation, but whenever possible the diagnosis should be made preoperatively to allow for appropriate surgical planning. Recently, investigators have shown the value of FDG-PET scanning as a useful investigation for diagnosing infection; however, it is no more accurate than the combined use of an ESR and CRP, and does not allow for identification of an infecting organism. At this point, a hip aspiration for culture is the most appropriate investigation.

     

     

     

    2020

  148. Figure 6 shows the radiograph of a 72-year-old woman who underwent a primary total hip arthroplasty 17 years ago. She now reports groin pain. Optimal surgical management should consist of which of the following?

     

     

     

     

     

    1. Synovectomy and polyethylene liner exchange

    2. Synovectomy, polyethylene liner exchange, and femoral stem revision

    3. Synovectomy and complete acetabular revision

    4. Synovectomy, bone grafting of lytic lesions, and retention of the components and polyethylene liner

    5. Revision of all components with synovectomy Corrent answer: 1

    Polyethylene wear is evident due to the superiorly eccentric position of the femoral head within the acetabulum. Despite proximal femoral osteolysis, the component appears well fixed, as does the acetabulum. The acetabular component appears to be well positioned. Therefore, an isolated synovectomy and polyethylene liner exchange is indicated. If the hip is stable, there is no need for more extensive revision work.

     

     

     

    2020

     

  149. Which of the following statements best describes how unicompartmental knee arthroplasty (UKA) differs from total knee arthroplasty (TKA)?

    1. TKA has a higher reoperation rate than UKA during the first 10 postoperative years.

    2. TKA provides for more normal rotational kinematics than UKA.

    3. UKA more closely replicates normal knee kinematics than TKA.

    4. Mobile bearings have been successful in UKA but not TKA.

    5. Mobile bearings have been successful in TKA but not UKA. Corrent answer: 3

    Because UKA does not require cruciate sacrifice, patellofemoral resurfacing, or rotational changes to the femur or tibia, it reliably recreates normal knee kinematics. UKAs have generally demonstrated higher reoperation rates than TKAs at intermediate and long-term follow-up, due in part to progression of arthritis in the nonresurfaced compartments. Mobile bearings have been clinically successful in both UKA and TKA.

     

     

     

    2020

     

  150. Which of the following is associated with the use of large femoral heads in total hip arthroplasty?

    1. Increased risk of total hip dislocation

    2. Decreased range of motion

    3. Fewer options for femoral head bearing material

    4. Acceptable wear with modern bearing surfaces

    5. Increased component impingement Corrent answer: 4

    Larger diameter femoral heads reduce the risk of hip dislocation by allowing greater range of motion before component impingement becomes a risk, and no reports have shown an increase in bearing wear. With modern bearings, volumetric wear should remain low, despite the increase in head diameter.

    Large femoral heads are available in all common bearing materials

     

     

     

    2020

     

  151. A 68-year-old man underwent a primary total hip arthroplasty 2 years ago for a femoral neck fracture. His early postoperative course was unremarkable, but he notes some aching in the thigh since surgery. His symptoms have gotten worse over the last year, such that he now has activity-related thigh pain that limits his walking ability. An AP hip radiograph is shown in Figure 9. What is the most appropriate surgical management?

     

     

     

    1. No surgery is indicated

    2. Revision of the acetabular component

    3. Revision of the femoral component

    4. Psoas tendon tenotomy

    5. Strut grafting of the femur to stiffen the bone near the tip of the implant Corrent answer: 3

    The radiograph shows a loose femoral component with failure of osseous integration. There is a reactive radiolucent line around the fiber mesh portion of the implant, hypertrophy of the bone under the collar, and a pedestal formation at the distal tip of the implant, all of which indicate failure of bone ingrowth. If bone ingrowth had occurred, there would be stress shielding of bone under the collar.

     

     

     

    2020

     

  152. Figure 10 shows patellar radiographs of a 68-year-old woman who underwent bilateral total knee arthroplasty 2 months ago. Following a recent fall onto the right side, she now reports anterior pain in the right knee. A CT scan shows that the femoral and tibial components are appropriately externally rotated and radiographs show acceptable axial alignment and no evidence of loosening. What is the most appropriate treatment option?

     

     

     

    1. Fracture fixation and bracing

    2. Lateral retinacular release with proximal realignment

    3. Tibial component revision

    4. Distal realignment by medialization of the tibial tubercle

    5. Revision of the patellar component Corrent answer: 2

    Treatment of patellofemoral instability after total knee arthroplasty (TKA) is directed by its etiology. In instances of component malpositioning, revision of one or both components is indicated. If the components are determined to be in satisfactory position, soft-tissue procedures can be pursued. Lateral retinacular release is usually the first soft-tissue procedure used to improve patellofemoral mechanics. In this patient, the patellar fracture fragment is so small that it can be excised. Distal realignment is not usually used as the first line of treatment for patellar maltracking following TKA.

     

     

     

    2020

     

  153. Which of the following statements most accurately describes the risk of ileus following total joint arthroplasty?

    1. Older age decreases risk.

    2. Male gender decreases risk.

    3. The risk is roughly 1% in total joint arthroplasty patients.

    4. The risk more commonly occurs in total knee arthroplasty (TKA) patients than in total hip arthroplasty (THA) patients.

    5. A history of abdominal surgery has no effect on risk.

     

    Corrent answer: 3

     

    The risk of postoperative ileus is noted to be higher in patients undergoing THA than patients undergoing TKA. Older age, male gender, and a history of abdominal surgery have been identified as risk factors.

     

     

    2020

     

  154. A 68-year-old woman who underwent right total hip arthroplasty 14 years ago now reports pain in her thigh. A radiograph is shown in Figure 14. What is the most reliable method for reconstructing the femoral component?

     

     

     

     

     

    1. Revision to a standard length cemented femoral component using third generation cementing techniques

    2. Revision to a proximal femoral replacement device ("tumor prosthesis")

    3. Revision to a proximally porous-coated, metaphyseal loading cementless femoral component

    4. Revision to a diaphyseal engaging, extensively coated cementless femoral component

    5. Revision to an allograft-prosthetic composite Corrent answer: 4

    The patient has a loose, cemented femoral component. The proximal femur is deficient and may not have adequate strength to support a proximally porous-coated cementless implant. Cemented hip implants have shown higher rates of failure when used for revision surgery. An extensively coated, cementless stem that engages the diaphysis has been shown to have excellent long-term durability in patients such as this, where the diaphysis is preserved and more than 4 cm of intact diaphysis is available for fixation. Allograft-prosthetic composites and tumor replacement prostheses are used for patients with more

    severe bone loss of the proximal femur and would not be indicated for this patient.

     

     

     

    2020

     

  155. A 71-year-old woman with coronary artery disease underwent an uncomplicated right total hip arthroplasty for osteoarthritis 12 years ago. Her hip has functioned well until approximately 18 months ago when she noted the spontaneous onset of groin, buttock, and proximal thigh pain that is present at rest and made worse with activity. A radiograph is shown in Figure 15. What is the recommended management at this point?

     

     

     

     

     

    1. Immediate admission to the hospital and emergent revision hip arthroplasty

    2. Reassurance and follow-up if symptoms worsen

    3. Repeat radiographs in 1 month

    4. Protected weight bearing with urgent revision hip arthroplasty when the patient is medically cleared

    5. A prescription for alendronate and reevaluation in 1 year Corrent answer: 4

    The radiograph shows significant osteolysis with loosening of the femoral component. The patient is symptomatic and surgery is indicated because of the extent of osteolysis and the loose femoral component. Reassurance and follow-up if symptoms worsen places the patient at risk for further bone loss

    and periprosthetic fracture. Emergent surgery is not required because the symptoms have been present for more than a year; however, urgent revision hip arthroplasty is recommended when the patient is medically cleared. While there is data to suggest that bisphosphonates may slow the progression of osteolysis in animal modes, there is no clear evidence that bisphosphonate treatment prevents the progression of osteolysis in humans. Additionally, this patient has a loose symptomatic femoral component.

     

     

     

    2020

     

  156. A 51-year-old woman who underwent a total knee arthroplasty 14 months ago for severe degenerative arthritis now reports progressive pain, swelling, and buckling of the knee. She must use crutches and is unable to negotiate stairs. Laboratory testing reveals a normal erythrocyte sedimentation rate and C-reactive protein. Radiographs of the patient are shown in Figures 17a through 17c. What is the most important test to further evaluate this problem?

     

     

     

     

     

     

    1. Long standing anterior-posterior radiograph of the hip-knee-ankle

    2. Axial CT views from the supracondylar distal femur to the proximal tibia below the tibial tubercle

    3. Fluoroscopic stress views to demonstrate the position of subluxation

    4. Bone scan

    5. MRI scan

     

    Corrent answer: 2

    The cause of subluxation in this patient is multifactorial, and includes a laterally positioned patellar component, a tibial tray that is internally rotated and translated to the medial side of the proximal tibial surface, and a femoral component that is markedly internally rotated about 10 degrees. All of these findings will be apparent on a CT scan. The long standing radiograph may be helpful but does not show the particular rotational abnormalities of both implants that are causing this problem. Fluoroscopic review may show how unstable the patella is, but the initial Merchant's view shows the basic problem. A bone scan does not provide information about component malposition. An MRI scan is inferior to a CT scan because of image artifact.

     

     

     

    2020

     

  157. Changes to the properties of ultra-high molecular weight polyethylene with increasing irradiation dose include improved

    1. resistance to oxidation.

    2. ultimate tensile strength.

    3. resistance to crack propagation.

    4. fracture toughness.

    5. volumetric wear.

     

    Corrent answer: 5

     

    Increased irradiation doses cause a decrease in the mechanical properties of the polyethylene, resulting in a decrease in ultimate tensile strength, fracture toughness, and resistance to crack propagation. Irradiation leads to the production of free radicals, requiring a step in the manufacturing process (melting, annealing, vitamin E doping) to stabilize the free radicals and reduce the potential for oxidation. Wear resistance is improved with irradiation; however, there is minimal benefit with doses of greater than 10 Mrads.

     

     

     

    2020

     

  158. A 72-year-old man with a history of Parkinson's disease, stable coronary artery disease, and mild renal insufficiency is seen for hip arthroplasty. Which of the following is considered the most appropriate bearing of choice?

    1. Ceramic-on-ceramic

    2. Large diameter metal head on highly cross-linked polyethylene liner

    3. Ceramic head on metal liner

    4. Large diameter metal-on-metal total hip arthroplasty

    5. Metal on conventional polyethylene liner (noncross-linked) Corrent answer: 2

    The most appropriate bearing of those listed would be a large diameter metal head on cross-linked polyethylene. Because of the renal insufficiency, metal-on-metal should be avoided because metal ions are renally excreted. Ceramic-on-ceramic implants are not necessary in this patient because of the patient's limited activity and life expectancy, as well as the downside of increased cost, the small but definite risk of ceramic fracture, and the increased technical demands of inserting ceramic implants. A highly cross-linked liner is favored to allow for maximum head size as the patient may be at increased risk of dislocation because of his neurologic disorder.

     

     

     

    2020

     

  159. What is the most common cause of reoperation following contemporary hip resurfacing for degenerative coxarthrosis?

    1. Limb-length discrepancy

    2. Altered femoral offset

    3. Snapping psoas tendon

    4. Limitation of hip movement

    5. Femoral neck fracture Corrent answer: 5

    Femoral neck fracture, necessitating revision surgery to a total hip arthroplasty, is the most likely common early complication after hip resurfacing. Cortical notching, varus positioning, and other technical variables appear to predispose to this complication. Limb lengths do not change appreciably after hip resurfacing; the same applies to femoral offset. Hip movement is not limited by hip resurfacing, and there is no evidence that psoas tendon impingement is increased by hip resurfacing.

     

     

     

    2020

     

  160. A 77-year-old man with a history of mild renal insufficiency and atrial fibrillation on warfarin therapy is scheduled to undergo a left total hip arthroplasty. He previously underwent a right total hip arthroplasty with development of significant heterotopic bone that resulted in limitation of motion. What is the most appropriate form of

    prophylactic treatment to minimize the formation of heterotopic bone on his left hip?

    1. Postoperative indomethacin for 3 weeks

    2. Postoperative indomethacin for 6 weeks

    3. No treatment indicated; can treat later if heterotopic bone forms

    4. 800 centigrey of radiation given to the periprosthetic soft tissues preoperatively on the morning of surgery

    5. 400 centigrey of radiation given to the periprosthetic soft tissues day 2 postoperatively

    Corrent answer: 4

     

    This question centers on the prophylactic treatment to reduce the risk of heterotopic bone formation. Prophylaxis is indicated because he has already demonstrated bone formation with his prior hip arthroplasty, which places him at increased risk for developing heterotopic bone on the contralateral side. He is on warfarin and has renal insufficiency, which makes the use of NSAIDs contraindicated. The recommended dose is 600 to 800 centigrey of radiation given within 24 hours of surgery preoperatively or 72 hours postoperatively.

     

     

     

    2020

     

  161. A 46-year-old male construction worker has right hip pain that has failed to respond to nonsurgical management. His body mass index (BMI) is 32, he is 6'2" tall, and he has no other medical comorbidities. AP and lateral radiographs of the right hip are shown in Figures 23a and 23b. The patient inquires about his suitability for metal-on-metal hip resurfacing. The patient should be educated that he is at higher risk for failure secondary to which of the following?

     

     

     

    1. BMI > 30

    2. Presence of secondary changes of the acetabulum

    3. Osteonecrosis of the femoral head

    4. Age of younger than 55 years old

    5. Male gender

     

    Corrent answer: 3

     

    This young patient has osteonecrosis of the femoral head with a large area of collapse. The results of hip resurfacing arthroplasty have been reported to be best in young, male patients who are younger than 55 years of age with a diagnosis of osteoarthritis. Although some authors advocate metal-on-metal hip resurfacing as an option for patients with osteonecrosis of the femoral head, in this particular patient, given the size of the necrotic segment, he would be at higher risk for failure and a conventional total hip arthroplasty would be a more conservative option. As the acetabulum is resurfaced in metal-on-metal hip resurfacing, the secondary changes of the acetabulum are not an issue and his BMI is in an acceptable range for the procedure.

     

     

     

    2020

     

  162. A healthy 78-year-old woman falls down a flight of stairs 2 years after undergoing left total hip arthroplasty. Radiographs are shown in Figures 25a through 25c. Optimal management should include which of the following?

     

     

     

     

     

     

    1. Skeletal traction

    2. Long stem femoral revision

    3. Closed reduction and internal fixation with a retrograde femoral nail

    4. Open reduction and internal fixation with a conventional plate and screws

    5. Open reduction and internal fixation with a plate, proximal cables, and distal screws

    Corrent answer: 5

     

    The fracture occurs distal to the component, and does not jeopardize the cemented implant fixation. The cement mantle is intact. The component appears well fixed and without osteolysis. In such cases, the femoral component can be left alone and the fracture fixed with internal fixation. There is no available intramedullary space proximal to the fracture to allow for a retrograde nail. A standard plate cannot obtain adequate fixation of the

    proximal fragment due to the presence of the femoral stem. Therefore, a plate that uses screws distal to the fracture and cables proximal to the fracture is the treatment of choice. Skeletal traction is usually reserved for patients unable to withstand surgery. This fracture is classified as a type C, according to the Vancouver classification of postoperative femoral fractures.

     

     

     

    2020

     

  163. Osteolysis after total hip arthroplasty with polyethylene acetabular bearings is most closely correlated with which of the following risk factors?

    1. Patient weight

    2. Femoral head bearing material

    3. Linear wear rate

    4. Decreased femoral offset

    5. Increased femoral offset Corrent answer: 3

    The development of osteolysis appears to be multifactorial. Patient activity, component positioning, polyethylene oxidation level, and bearing surface all appear to contribute. They contribute, however, by increasing the rate of wear. Therefore, the one variable that correlates closest with the likelihood of osteolysis (and the magnitude of osteolysis) is the wear rate of the bearing couple. Wear can be measured linearly or volumetrically. Both correlate with the development of osteolysis.

     

     

     

    2020

     

  164. A 62-year-old woman undergoes an uncomplicated primary metal-on-metal cementless hip arthroplasty. Her early postoperative course is unremarkable. She returns at 1 year reporting groin pain. Her symptoms are made worse with stair climbing and getting in and out of her vehicle. Examination reveals minimal discomfort with passive range of motion, but straight leg raising reproduces her symptoms. A radiograph is shown in Figure 28. What is the most appropriate surgical management?

     

     

     

    1. Revision of the femoral component

    2. Psoas tendon tenotomy

    3. Exchange of the bearing to metal-on-polyethylene

    4. Exchange of the bearing to ceramic-on-ceramic

    5. Exchange of the bearing to a smaller femoral head and metal liner Corrent answer: 2

    The radiograph shows a cementless cup that is placed low within the native acetabulum and appears large relative to the patient's native acetabulum. The size and inferior position of the cup place it at risk for causing psoas tendon irritation or impingement. Symptoms are typical for irritation of the psoas tendon and not consistent with either a loose implant (more mechanical activity-related pain) or metal hypersensitivity (more constant pain and not just associated with hip flexion activities). Controversy exists regarding tenotomy versus acetabular revision.

     

     

     

    2020

     

  165. Increasing tibial polyethylene conformity can have what effect on fixed bearing total knee arthroplasty?

    1. Increased contact stress within the polyethylene

    2. Increased risk of polyethylene delamination

    3. Decreased mechanical forces transferred to the fixation surfaces

    4. Decreased femoral rollback during flexion

    5. Increased femoral rotation during flexion Corrent answer: 4

    Increasing conformity between the tibial polyethylene bearing and the femoral component has desirable and undesirable consequences. Increased conformity increases the contact area and thereby reduces contact stress within the polyethylene. This can lead to less risk of polyethylene wear, fracture, and delamination. However, increasing conformity also limits the ability of the femur to roll back during flexion, and may transfer increased shear stress to the fixation surfaces of the implants.

     

     

     

    2020

     

  166. The anterior approach to total hip arthroplasty requires dissection between which of the following muscle planes?

    1. Sartorius and gluteus maximus

    2. Gluteus minimus and rectus femoris

    3. Rectus femoris and sartorius

    4. Tensor fascia lata and sartorius

    5. Tensor fascia lata and rectus femoris Corrent answer: 4

    The anterior approach to the hip joint involves identifying the plane between the tensor fascia lata and the sartorius muscles.

     

     

     

    2020

     

  167. A 41-year-old female postal worker reports a 9-month history of left groin and lateral hip pain. She denies pain in the right hip. Her social history reveals that she smokes and drinks on average five alcoholic beverages per week. Her body mass index (BMI) is 26. Radiographs are shown in Figures 31a through 31c and coronal and axial MRI scans are shown in Figures 31d and 31e, respectively. What is the most important factor that will determine if her right hip will become symptomatic?

     

     

     

     

     

     

     

     

     

     

     

     

     

     

    1. Patient age

    2. Gender of the patient

    3. Patient's BMI and activity level

    4. Size of the lesion

    5. Association with alcohol use Corrent answer: 4

    Several factors have been examined as potentially predictive of progression in the patient with an asymptomatic osteonecrotic lesion of the femoral head.

    While the presence of ongoing risk factors, lesion location, lesion stage, age, gender, and BMI have all been suspected as important, the size of the lesion, particularly when over one third of the size of the femoral head, is a significant risk factor for progression.

     

     

     

    2020

     

  168. A 66-year-old woman reports pain in both of her knees that has been recalcitrant to nonsurgical management. Radiographs, including an AP of both knees, lateral and patellar views of the more symptomatic left knee and a mechanical axis, are shown in Figures 32a through 32d. The lateral radiograph shows maximal knee extension. When counseling the patient preoperatively regarding the risks of total knee arthroplasty, she should be educated that she is at higher risk than the typical patient for which of the following complications?

     

     

     

     

     

     

    1. Deep venous thrombosis

    2. Peroneal nerve injury

    3. Wound healing problems

    4. Femoral component loosening

    5. Quadriceps tendon rupture Corrent answer: 2

    The patient has a large valgus deformity and flexion contracture that places her at increased risk for peroneal nerve injury. The peroneal nerve is tethered at the fibular head and correction of her valgus deformity and flexion contracture at the time of total knee arthroplasty can stretch the nerve, leading to peroneal nerve palsy.

     

     

     

    2020

     

  169. Following total knee arthroplasty, a patient is noted to have asymmetrical absent pulses and poor capillary refill. What is the next most appropriate step in management?

    1. Observation of the limb for 4 hours to see if the arterial spasm resolves

    2. Measurement of lower leg compartment pressures

    3. Magnetic resonance angiogram

    4. Emergent return to the operating room for wound exploration while the patient is still under anesthesia

    5. Return to the operating room, obtain a vascular surgery consultation, and perform an intraoperative arteriogram

    Corrent answer: 5

     

    An assessment of the location of the vascular compromise is necessary prior to surgical exploration. Vascular repair will most likely require a separate surgical exposure. Vascular reperfusion may be accomplished at the time of an arteriogram with the use of a stent in certain situations. Return to the operating room with vascular surgical consultation and intraoperative arteriogram is appropriate. An immediate postoperative compartment syndrome is unlikely. Magnetic resonance angiogram is not appropriate because of the potential for a delay in diagnosis.

     

     

     

    2020

     

  170. A 57-year-old man undergoes total hip arthroplasty, has an uncomplicated early postoperative course, and a normal neurovascular status is documented. However, on postoperative day two he develops a progressive foot drop that increases over the next 24 hours. Postoperative repeat radiographs of the hip arthroplasty are unrevealing. There is no suggestion of swelling of the thigh to suggest a subfascial wound hematoma. What is the next most appropriate step?

    1. MRI of the lumber spine

    2. MRI of the knee joint

    3. Electromyography and nerve conduction velocity studies of the sciatic nerve

    4. Lumbar spine anterior/posterior and lateral radiographs

    5. Venous Doppler of the lower extremity Corrent answer: 1

    Spinal stenosis is commonly found in patients with degenerative arthritis of the hip joint and would be a strong possibility for causing a foot drop if the radiographs do not reveal a dislocation or other direct mechanical cause for the sciatic nerve compromise.

     

     

     

    2020

     

  171. A 68-year-old woman undergoes an uncomplicated primary total knee arthroplasty with cement. Twelve days later, the proximal wound appears healed and pain is decreased, but the distal wound continues to drain with slight surrounding redness. What is the most appropriate management?

    1. Prescribe an antibiotic specific for hospital-acquired bacteria.

    2. Recommend antibiotic ointment and peroxide treatment.

    3. Place the knee in an immobilizer and observe.

    4. Place sutures to close up the wound in the clinic.

    5. Irrigate and debride the knee in the operating room.

     

    Corrent answer: 5

     

    Persistent wound drainage after a total knee arthroplasty can lead to infection and other complications. In this situation, antibiotics alone are not indicated, nor are topical wound treatment measures. Prompt, aggressive treatment includes returning the patient to the operating room for irrigation and debridement, with possible polyethylene exchange. Resection of components is not necessarily indicated at this early stage because the arthroplasty can still be salvaged.

     

     

     

    2020

     

  172. A nondisplaced fracture of the proximal medial femoral neck proximal to the lesser trochanter is noted at the time of insertion of a cementless tapered wedge-type femoral component in a total hip arthroplasty. Appropriate perioperative management should include which of the following?

    1. Cerclage cable placed proximal to the lesser trochanter with partial weight bearing for 6 weeks postoperatively

    2. No intraoperative or postoperative modifications are necessary

    3. Non-weight-bearing for 6 weeks, retention of the femoral component, and no cerclage wire

    4. Fracture exploration and repair with multiple cerclage cables, strut allograft and revision of the femoral component with a long-stemmed implant

    5. Revision with a cemented implant Corrent answer: 1

    The fracture should be explored in its entirety. If it remains in the intertrochanteric region, a single cerclage cable passed above the lesser trochanter and tightened around the femoral component is appropriate. A more distal or displaced fracture should be repaired with cerclage cables and consideration for revision of the femoral component with a long-stemmed or cemented implant should be given.

     

     

     

    2020

     

  173. A 68-year-old woman reports pain and sensations of instability following a primary total knee arthroplasty 18 months ago. A preoperative radiograph is shown in Figure 39a and postoperative AP and patellar view radiographs are shown in Figures 39b and 39c. A CT scan shows that the femoral component is internally rotated 8 degrees and the tibial component is internally rotated 4 degrees. Management should include which of the following?

     

     

     

     

     

     

    1. A structured physical therapy program

    2. A custom patellar stabilizing brace

    3. Modular polyethylene liner exchange to a thicker liner

    4. Revision of the femoral and tibial components

    5. An open lateral retinacular release Corrent answer: 4

    The patient had a valgus knee preoperatively and in these patients, care must be taken to avoid internal rotation of the femoral component that can lead to patellar instability. At the present time she has a completely dislocated patella, evidence of coronal instability on her AP radiograph, and internally rotated femoral and tibial components; therefore, management should consist of revision of both of her components to place them in appropriate external rotation.

     

     

    2020

     

  174. The term "paradoxical motion," used to describe knee kinematics, is best described by which of the following definitions?

    1. The patella does not roll forward into the trochlear groove during knee extension.

    2. The tibia rolls back on the femur during knee extension.

    3. The tibiofemoral contact point moves anteriorly during knee flexion.

    4. The posterior cruciate ligament rolls posteriorly with respect to the anterior cruciate ligament during knee extension.

    5. The femur rolls back on the tibia during knee flexion.

     

    Corrent answer: 3

     

    The term "rollback" describes the posterior movement of the tibiofemoral contact point with knee motion from extension to flexion. Therefore, with "paradoxical rollback" this contact point moves anteriorly. "Paradoxical rollback" is a term used to connote the inability of the anterior cruciate-deficient, posterior cruciate-retaining total knee prosthesis to create normal posterior femoral rollback with knee flexion.

     

     

     

    2020

     

  175. Patients with hip dysplasia have a series of anatomic abnormalities that most commonly include which of the following?

    1. Shallow, medialized acetabulum that is deficient anteriorly and superiorly

    2. Large contact area between the femoral head and acetabulum

    3. Large femoral head with long femoral neck

    4. Excessive femoral neck anteversion and a posterior greater trochanter

    5. Decreased neck-shaft angle Corrent answer: 4

    Patients with developmental dysplasia of the hip share a common pattern of anatomic abnormalities including an acetabulum that is shallow, lateralized, anteverted, and deficient anteriorly and superiorly. On the femoral side, the head is usually small, the neck is short and anteverted with a posteriorly placed trochanter, and the femoral canal is small. The neck-shaft angle is typically increased. The contact area of the femoral head and acetabulum is typically decreased.

     

     

     

    2020

  176. An active 72-year-old man underwent a right hybrid total hip arthroplasty for osteoarthritis 4 years ago. His hip has functioned well until approximately 8 months ago. He now reports activity-related proximal thigh pain and groin pain. A current radiograph is shown in Figure 43a. A radiograph obtained prior to the onset of symptoms is shown in Figure 43b. What is the most likely cause of his symptoms?

     

     

     

     

     

    1. Polyethylene wear with polyethylene-induced synovitis

    2. Psoas tendon irritation secondary to psoas impingement over the anterior inferior edge of the acetabular component

    3. Loose femoral component

    4. Osteolysis secondary to polyethylene wear particles

    5. Modulus mismatch of the stem and femur Corrent answer: 3

    The radiograph shows a loose femoral component. The implant has debonded showing a radiolucent line at the lateral shoulder (zone 1), and also increased radiolucency at the bone cement interface medially. The patient's symptoms are mechanical in nature with activity-related pain. Symptoms are not characteristic of psoas tendon irritation. Although osteolysis is present, in the absence of fracture it is generally asymptomatic. Additionally, in this patient the osteolysis involving the proximal femur is much more likely to be secondary to cement particulate debris and less likely related to polyethylene wear particles.

     

     

    2020

     

  177. A 64-year-old woman who underwent a successful total hip arthroplasty (THA) 2 years ago now reports a painful hip. A radiograph is shown in Figure 44. Laboratory evaluation includes an erythrocyte sedimentation rate (ESR) of 65 mm/h (0-30 mm/h) and a C-reactive protein of 5.4 mg/L (< 0.8 mg/L). What is the next step in management?

     

     

     

     

     

    1. Ultrasound examination

    2. Technetium bone scan

    3. Indium-labeled WBC scan

    4. Hip joint aspiration

    5. Two-stage revision surgery Corrent answer: 4

    Hip joint aspiration, with analysis of the cell count, differential, and culture greatly aids in the diagnosis of infection in THA. Aspiration is indicated in this patient because both the ESR and C-reactive protein are elevated, and the radiograph shows well-fixed components without evidence of loosening.

    Technetium and Indium-labeled WBC scans have less specificity and sensitivity respectively, and may not be necessary if the diagnosis is confirmed by joint aspiration. Two-stage revision surgery may be the treatment of choice if infection is confirmed, but it is not the next step in this patient.

     

     

    2020

     

  178. What is the most common cause of early failure for patellofemoral arthroplasty?

    1. Progression of tibiofemoral arthritis

    2. Loosening of the femoral trochlear component

    3. Loosening of the patellar component

    4. Patellar instability/maltracking

    5. Rupture of the quadriceps tendon from trochlear component impingement secondary to excessive anterior placement and flexion of the implant

    Corrent answer: 4

     

    In properly selected patients who have no or minimal tibiofemoral arthritis, the most common cause of early failure is patellofemoral instability secondary to uncorrected patellar malalignment, soft-tissue imbalance, or component malposition. Progression of tibiofemoral arthritis is also a leading cause of failure, but occurs late in about 25% of patients. Loosening of components has occurred in less than 1% of knees with cemented implants. A higher rate of loosening has been reported in cementless implants.

     

     

     

    2020

     

  179. An 88-year-old nursing home resident is seen in the emergency department after a fall. At the time of admission, physical examination of the affected extremity reveals absent pulses and inadequate capillary refill. A radiograph is seen in Figure 48. Appropriate management includes which of the following?

     

     

     

    1. Emergent open reduction and internal fixation of the fracture

    2. Emergent consultation with vascular surgery and a possible arteriogram

    3. Revision of the femoral component followed by vascular surgical consultation

    4. Open reduction and internal fixation of the fracture with intraoperative vascular consultation

    5. Release traction until surgical management is performed Corrent answer: 2

    The patient has a Vancouver type B3 periprosthetic femur fracture (fracture around a total hip prosthesis with a loose stem and poor proximal bone stock). This type of periprosthetic fracture necessitates revision of the femoral component as well as fracture fixation. This patient has diminished blood flow in the extremity that necessitates emergent vascular surgical evaluation.

     

     

     

    2020

     

  180. An otherwise healthy 58-year-old woman reports hip pain after undergoing total hip arthroplasty 8 months ago. She gives a history of prolonged wound drainage after surgery treated with antibiotics. Hip aspiration is positive for methicillin-resistant coagulase-negative staphylococcus. Appropriate management at this point includes which of the following?

    1. Arthroscopic irrigation followed by appropriate antibiotic treatment

    2. Treatment with 6 weeks of IV vancomycin and oral rifampin

    3. Treatment with 6 weeks of IV vancomycin and oral rifampin, followed by indefinite oral antibiotic suppression

    4. Open debridement with exchange of the polyethylene insert, followed by appropriate antibiotic treatment

    5. Open debridement with removal of the implants and insertion of an antibiotic spacer

    Corrent answer: 5

     

    The patient has a late chronic periprosthetic infection. The history points to a missed postoperative infection where the patient was treated with antibiotics for prolonged wound drainage. A low virulent organism, such as coagulase-negative staphylococcus, is often the pathogen. In an otherwise healthy patient with a reasonable life expectancy, the goal of treatment should be cure of the infection. Treatment for late chronic infection requires removal of the implants. Treatment with antibiotics alone, or debridement without removal of the implants is generally unsuccessful. Controversy exists regarding one-stage treatment with immediate reimplantation versus two-stage treatment with delayed reimplantation. Generally, a higher cure rate is achieved with a two-stage reimplantation (approximately 90% to 93%) than with one-stage reimplantation (75% to 80%).

     

     

     

    2020

     

  181. Posterior cruciate-retaining total knee arthroplasty has been studied by numerous methods including kinematic video fluoroscopy. Which of the following is the best description of typical kinematic behavior?

    1. Unpredictable anterior femoral condylar translation from full extension to 90 degrees of flexion

    2. Predictable femoral rollback from full extension to 90 degrees of flexion

    3. Medial femoral condyle pivot with minimal medial femoral condyle translation and lateral femoral condyle posterior translation in flexion

    4. Lateral femoral condyle pivot with minimal medial femoral condyle translation

    5. Anterior tibial contact of both the medial and lateral femoral condyles in full extension

    Corrent answer: 1

    In vivo fluoroscopic studies have enlightened orthopaedic surgeons as to the actual kinematic performance of the total knee implants. Importantly, many technical factors combine to result in abnormal kinematics after total knee arthroplasty. The typical kinematics of the normal knee are negated by osteoarthritis and the various changes created by the surgeon's technique.

    Dennis and associates observed that anterior femoral condylar translation during deep knee flexion was most commonly observed in posterior cruciate-retaining knees.

     

     

     

    2020

     

  182. A 52-year-old woman has a 60-degree extensor lag following a right total knee arthroplasty performed 16 months ago. Since the time of her primary total knee arthroplasty she has undergone primary repair of a patellar tendon rupture that occurred after a fall 8 months ago. A lateral radiograph of the knee is shown in Figure 52. A CT scan obtained to determine component rotation showed that the femoral component is internally rotated 9 degrees and the tibial component is internally rotated 12 degrees. Appropriate management at this time should include

     

     

     

     

     

    1. a structured physical therapy program to increase quadriceps muscle strength.

    2. a hinged knee brace locked in extension while ambulating.

    3. exchange of the modular polyethylene spacer to a thicker insert and

      reconstruction of the patellar tendon using hamstring augmentation.

    4. exchange of the modular polyethylene spacer to a thicker insert and reconstruction of the patellar tendon using an extensor mechanism allograft tensioned tightly in full extension.

    5. revision of the tibial and femoral components and reconstruction of the patellar tendon using an extensor mechanism allograft tensioned tightly in full extension.

    Corrent answer: 5

     

    A chronic patellar tendon rupture is a difficult complication to manage. Patients typically present with both inability to extend their leg and instability of the extremity, oftentimes associated with multiple falls. Attempts at secondary repair have been associated with high failure rates whereas the use of an extensor mechanism allograft has been shown to more effectively restore active extension in a substantial percentage of patients. Important aspects of the technique include fully tensioning the graft in full extension and immobilization of the extremity for 6 to 8 weeks postoperatively to allow for graft healing. Nonsurgical management will not result in an acceptable outcome for a young patient, and attempted secondary repair is associated with a high rate of failure, even when augmented with local tissues. This patient has gross rotational malalignment of the components and the surgeon faced with this problem should consider obtaining a CT scan to determine component rotation preoperatively.

     

     

     

    2020

     

  183. A 66-year-old woman who underwent a right total knee arthroplasty 13 years ago now has pain, knee effusion, squeaking, and grinding in the operated knee. The patellar view radiograph is shown in Figure 54. What is most likely to be encountered during revision arthroplasty?

     

     

     

    1. Loose femoral and/or tibial component

    2. Fractured patella

    3. Extensive metallosis

    4. Patellar clunk

    5. Ruptured extensor mechanism Corrent answer: 3

    Wear of the polyethylene insert of a metal-backed patellar or tibial component is a well-recognized complication following total knee arthroplasty. If the polyethylene insert completely wears out, the underlying metal-backed base plate will be in direct contact with the femoral component, resulting in scratching of the components and release of metal debris. The released metal debris results in black "tattooing" of the tissues or so-called metallosis. In addition, reactive synovitis and osteolysis occur secondary to a host response to both polyethylene debris and metal particles. Patients may report knee pain, swelling, squeaking, and an unstable feeling during walking and standing. Patellar clunk syndrome is characterized by the development of a fibrous nodule on the deep aspect of the quadriceps tendon that falls into the intercondylar notch of the femoral component during knee flexion.

     

     

     

    2020

     

  184. A 56-year-old man who underwent a left total hip arthroplasty 8 years ago is seen following a fall from a standing height. A radiograph obtained at 2 years postoperatively is shown in Figure 55a and a current radiograph obtained in the emergency department is shown in Figure 55b. On further questioning, he reports pain in this thigh for

    the past 3 years that has been increasing in intensity. Appropriate management at this time includes which of the following?

     

     

     

     

     

    1. Nonsurgical management with the use of a cast-brace

    2. Nonsurgical management with skeletal traction

    3. Open treatment with a locked plate with or without strut allograft

    4. Revision of the femoral component to a cemented femoral component that bypasses the fracture site by two cortical diameters

    5. Revision of the femoral component to a cementless femoral component that bypasses the fracture site by at least two cortical diameters

    Corrent answer: 5

     

    The patient has a Vancouver type B2 periprosthetic femoral fracture with a loose, cementless femoral component. Radiographs show subsidence of the femoral component and the patient reports pain in the thigh for several years prior to the fall. When the femoral component is loose, revision is mandated to treat both the loose component and the fracture. Nonsurgical management is associated with a high risk of medical complications related to extended recumbency as well as a high rate of malunion and nonunion. Cementless femoral component revision has been shown to fare better than cemented femoral components, particularly for treating periprosthetic fractures of the femur.

     

     

     

    2020

  185. What mechanical properties are observed in polyethylene used for total knee arthroplasty after the material undergoes oxidation?

    1. Increased elastic modulus and decreased strength

    2. Increased elastic modulus and increased ductility

    3. Decreased elastic modulus and decreased strength

    4. Decreased strength and increased ductility

    5. Decreased elastic modulus and decreased ductility Corrent answer: 1

    When polyethylene undergoes oxidation, the material undergoes a decrease in strength and ductility, and an increase in the elastic modulus. This makes the material more brittle, and leaves it vulnerable to delamination, fracture, and pitting.

     

     

     

    2020

     

  186. Which of the following abnormalities has been observed in a higher than expected frequency in patients with metal-on-metal hip bearings?

    1. Renal cell carcinoma

    2. Leukocyte chromosomal aberrations

    3. Carcinomas of the gastrointestinal tract

    4. Soft-tissue sarcomas

    5. Thyroid carcinoma

     

    Corrent answer: 2

     

    Metal-on-metal hip bearings have been associated with very low rates of wear and are commonly used in North America and Europe. Patients with these bearings have higher levels of metal ions (particularly cobalt and chromium) in the bloodstream than patients with bearings made of other materials.

    Although many researchers have been concerned that these ions may predispose to cancer, there has been no evidence that patients with metal-on-metal bearings are developing sarcomas or carcinomas with higher frequency than the general population. However, there has been mixed data as to whether hematopoietic malignancies are slightly more prevalent in patients with metal-on-metal bearings. Two recent reports have found chromosomal abnormalities, such as translocations and aneuploidy, to be increased in patients with metal-on-metal hip bearings. The clinical consequences of these changes, if any, remain unknown.

     

     

    2020

     

  187. A 73-year-old woman with a history of type II diabetes mellitus undergoes a total hip arthroplasty for osteoarthritis. She continues to have serosanguinous wound drainage from the midportion of the incision 12 days after surgery. What is the most appropriate treatment at this time?

    1. Return to the operating room for debridement and irrigation with removal of all implants and immediate reimplantation

    2. Return to the operating room for open debridement and irrigation, exchange of the polyethylene insert, followed by appropriate antibiotics based on intraoperative culture results

    3. Hip spica pressure dressing and a 2-week course of oral antibiotics

    4. Hip aspiration for culture, followed by a 2-week course of appropriate IV antibiotics based on culture results

    5. Hip aspiration for culture, followed by a 6-week course of appropriate IV antibiotics based on culture results

    Corrent answer: 2

     

    This patient has an acute postoperative deep infection, with prolonged drainage at 12 days postoperatively. Even without other signs or symptoms of infection, the prolonged drainage this far out after surgery is concerning for an acute postoperative infection. The most appropriate treatment at this point is open debridement with retention of the implants. Modular parts, if present, are exchanged. Oral or IV antibiotics alone are generally inadequate to treat the infection, and this form of management alone would result in a missed opportunity to potentially cure the infection. Removal of all implants at this point is not required because early aggressive debridement within the first 2 weeks after surgery offers a reasonable outcome. After 4 to 6 weeks of symptoms, results of debridement are less favorable and removal of the implants is usually required.

     

     

     

    2020

     

  188. A 63-year-old woman reports pain in her groin, particularly when rising from a chair and when taking her first steps out of bed in the morning. History reveals that she underwent a left primary total hip arthroplasty 19 years ago. An AP radiograph is shown in Figure 62 and revision surgery is planned. What is the most reliable method for reconstruction of the acetabulum?

     

     

     

    1. Use of a cemented, all polyethylene acetabular component

    2. Use of a cementless, porous-coated acetabular component with adjunctive screw fixation

    3. Use of an antiprotrusio cage

    4. Use of a bilobed, cementless acetabular component

    5. Use of a bipolar head placed directly into the acetabulum Corrent answer: 2

    The patient has severe wear and loosening of her cemented, all polyethylene acetabular component with a Paprosky type 2 acetabular defect. Optimal longterm results have been achieved with the use of a cementless, porous-coated acetabular component with adjunctive screw fixation. Cemented, all polyethylene components have been shown to have a high rate of failure when used in the revision setting. Bilobed components and antiprotrusio cages are unnecessary for this straightforward defect and have had mixed results reported in the literature. The use of a bipolar head placed directly into the acetabulum is associated with high rates of persistent pain and progressive acetabular erosion.

     

     

     

    2020

     

  189. A 72-year-old man undergoes an uncomplicated cementless total hip arthroplasty for advanced osteoarthritis. At his 6-week postoperative follow-up, he has minimal pain and is progressing well

    with his mobility. Radiographs show early formation of Brooker grade III heterotopic bone around his hip. What is the best treatment of the heterotopic bone at this time?

    1. Observation, repeat radiographs, and reexamination in 6 weeks

    2. A 14-day course of indomethacin

    3. A 4-week course of indomethacin

    4. Plan for a return to the operating room at 10 weeks for excision of the heterotopic bone

    5. Arrange urgently for 800 centigrey of radiation to the soft tissues and areas of heterotopic bone around the hip, with shielding of the implants

    Corrent answer: 1

     

    The development of heterotopic bone occurs early after hip arthroplasty. The process begins within days after surgery; therefore, prophylactic treatment must be in the early postoperative period (preoperative radiation given within 24 hours of surgery, or postoperative radiation given within 72 hours of surgery, or nonsteroidal anti-inflammatory drugs (NSAIDs) given postoperatively for 7 to 21 days - longer duration has not been shown to be of any additional benefit). At 6 weeks, prophylactic treatment with NSAIDs or radiation is no longer effective. Surgery at 10 weeks would be premature because the patient is currently asymptomatic with regards to the heterotopic bone, and surgery prior to full maturation of the bone may increase the risk for more abundant recurrence of bone.

     

     

     

    2020

     

  190. Figure 64 shows the radiograph of a 61-year-old man with ankylosing spondylitis. He is scheduled to undergo left total hip arthroplasty. Which of the following perioperative interventions should be considered?

     

     

     

    1. Intraoperative sciatic nerve monitoring

    2. Preoperative radiation therapy with 700 centigray

    3. Postoperative radiation therapy with 1,500 centigray

    4. Postoperative continuous passive motion

    5. Soft cervical collar during postoperative physical therapy Corrent answer: 2

    Patients with ankylosing spondylitis are at high risk for heterotopic ossification after total hip arthroplasty, and perioperative prophylaxis with nonsteroidal anti-inflammatory drugs or radiation therapy has been effective at reducing the risk. Radiation prophylaxis given within 24 hours preoperatively has been shown to be as effective as radiation given postoperatively. Doses of 700 to 800 centigray have been shown to be as effective as higher doses with less potential risk. Continuous passive motion has not been shown to improve the postoperative flexibility of these patients. Although these patients may have stiff spines and may be at risk for neurologic injury in the event of spinal fracture, a soft cervical collar is not expected to alter the risk should a catastrophic spinal injury occur. Since there has been no shortening of the limb, the sciatic nerve should not be at significantly higher risk for neurapraxia than with a more routine hip arthroplasty.

     

     

     

    2020

     

  191. An active 38-year-old male carpenter reports activity-related medial knee pain. Arthroscopy performed 3 years ago revealed a torn medial meniscus that was debrided and mild condylar changes of the medial femoral condyle and medial tibial plateau. Current standing

    radiographs reveal Ahlback stage II changes with mild medial femoral joint space narrowing and a 5-degree varus deformity. What is the best treatment option?

    1. High tibial osteotomy

    2. Arthroscopic debridement and condylar "microfracture"

    3. Osteoarticular transplant to the medial femoral condyle

    4. Unicondylar arthroplasty

    5. Total knee arthroplasty Corrent answer: 1

    The choice of high tibial osteotomy is appropriate to correct a deformity that has a very high risk of leading to progressive arthritis. It should be more important than either the arthroscopic method or the osteoarticular transplantation, though one may also consider these options. Prosthetic choices are limited for a young active male in favor of less "end stage" options.

     

     

     

    2020

     

  192. A patient with a severe nickel allergy and degenerative joint disease of the hip would be best served by which of the following prosthetic options?

    1. Cemented titanium stem, ceramic (alumina) head, and press-fit titanium cup

    2. Cemented cobalt-chrome stem, ceramic (alumina) head, and press-fit cobalt-chrome cup

    3. Press-fit titanium stem, cobalt-chrome head, and press-fit titanium cup

    4. Press-fit titanium stem, titanium head, and press-fit titanium cup

    5. Press-fit titanium stem, ceramic (alumina) head, and cementless titanium cup

    Corrent answer: 5

     

    Nickel is present in cobalt-chrome and stainless steel alloys used in orthopaedic surgery; therefore, these materials are not well suited for nickel-sensitive patients. Nickel is not present in titanium alloys or in ceramic components. Titanium is therefore the material of choice for the femoral and acetabular component. Titanium is a poor option for the femoral head due to its susceptibility to abrasive wear. Titanium is also poorly suited to cemented applications in hip arthroplasty because it is less stiff than cobalt-chrome (and stainless steel), and therefore transmits greater stresses to the cement column. Titanium's poor abrasion resistance can also leave the component

    susceptible to increased abrasive wear in the event of loosening and micromotion. Therefore, of the options available, a cementless titanium stem and socket with a ceramic head is the best choice. In nickel-sensitive total knee arthroplasty patients, cemented oxidized zirconium femoral components can be used in place of cobalt-chrome, and titanium tibial components (press-fit or cemented) have been used with success.

     

     

     

    2020

     

  193. Implant position at the time of primary total knee arthroplasty to optimize patellar tracking includes which of the following?

    1. External rotation of the femoral and tibial components

    2. Internal rotation of the femoral component and external rotation of the tibial component

    3. Internal rotation of the femoral and tibial components

    4. Medialization of the femoral component

    5. Lateralization of the patellar component Corrent answer: 1

    Component rotation is a critical factor to optimizing patellar tracking at the time of primary and revision total knee arthroplasty. Both the femoral and tibial components should be externally rotated whereas the patellar components should be medialized.

     

     

     

    2020

     

  194. The placement of supplemental screw fixation with acetabular component fixation is a typical adjunct measure but carries the greatest risk of vascular injury if placed in which of the following positions?

    1. Superior

    2. Posterior superior

    3. posterior inferior

    4. Superior lateral

    5. Anterior superior

     

    Corrent answer: 5

     

    Anatomic studies indicate that the safe areas for screws are superior and posterior. The external illiac artery is at risk with anteromedial placement, and the sciatic nerve may be compromised by posterior inferior screw placement.

     

     

    2020

     

  195. A 71-year-old woman has a failed revision hip arthroplasty and is undergoing a re-revision hip arthroplasty. Her last hip surgery was 4 years ago with revision of the acetabular component. Radiographs show a well-fixed extensively porous-coated femoral component and a failed acetabular component with proximal and medial migration through the floor of the acetabulum. Preoperative laboratory studies reveal an erythrocyte sedimentation rate (ESR) of 70 mm/h (normal

    0-29 mm/h), a C-reactive protein (CRP) of 23.3 (normal 0.2-8.0), and a negative hip aspiration. At the time of surgery, tissues look inflamed and a frozen section shows 20 WBC per high power field; however, a Gram stain is negative. What is the most appropriate action at this point?

    1. Proceed with the revision as planned

    2. Obtain cultures and proceed with revision of the acetabulum only

    3. Obtain cultures and proceed with revision of the femur only

    4. Obtain cultures, remove the implants, and insert an antibiotic spacer

    5. Obtain cultures and close Corrent answer: 4

    Despite the negative aspiration preoperatively, intraoperative findings are suspicious for infection. Additionally, the preoperative blood work is also concerning for infection with an elevated CRP and ESR. The frozen section is also positive. Most important is the unreliability of the Gram stain. Numerous investigators have show high false negative rates for Gram stain in chronic periprosthetic infection. The Gram stain should not be relied on for decision-making in revision surgery, particularly when other investigations point to infection. With the information available, the diagnosis is deep infection. The best course of action is to obtain cultures, remove the implants, and insert an antibiotic spacer. Only obtaining cultures and closing would require a second operation to remove the implants if the cultures are positive.

     

     

     

    2020

     

  196. A 79-year-old patient has a history of peripheral vascular disease and reports chronic knee pain. She has had coronary artery disease treated with angiography and stents on two occasions. Peripheral pulses are decreased in both lower extremities, but the patient is disabled by advanced chronic degenerative arthritis in her right knee

    and would like to proceed with a total knee arthroplasty. The next most appropriate evaluation should include which of the following?

    1. Ankle-brachial index of the affected lower extremity

    2. Femoral popliteal angiography

    3. Venous Dopplers of both lower extremities

    4. MRI of the popliteal fossa

    5. Radiographs to identify calcified plaques in the femoral artery Corrent answer: 1

    This question is designed to draw attention to the fact that peripheral vascular disease carries an increased risk of complications for the patient and should be carefully evaluated. The vascular surgeon will make the choice of revascularization or surgical clearance for knee reconstruction based on the initial results of the ankle-brachial index.

     

     

     

    2020

     

  197. An active 18-year-old patient reports severe left hip pain that prevents her from playing lacrosse. An AP radiograph of the pelvis is shown in Figure 73. What is the most appropriate option for this patient?

     

     

     

     

     

    1. Activity modification

    2. Hip fusion

    3. Periacetabular osteotomy

    4. Femoral osteotomy

    5. Total hip arthroplasty Corrent answer: 3

    The patient has developmental dysplasia of the hip (DDH). There is anterolateral deficiency of the acetabulum as is evidenced by the increased acetabular index and the reduced center-edge angle. The patient has some arthritis of the hip with narrowing of the joint space and cyst formation visible on the radiograph. Although all of the mentioned choices may be acceptable treatments for dysplasia of the hip, periacetabular osteotomy is the best and most appropriate option for this young patient. Periacetabular osteotomy allows correction of the problem and can even improve the joint space as the new region of the acetabulum is rotated into the weight-bearing region. Hip fusion is very poorly tolerated by young patients, especially women. Femoral osteotomy alone is unlikely to address the problem because the major problem is on the acetabular side. Although the patient has arthritis, because of the young age of the patient, prosthetic replacement is not an attractive option.

    The outcome of periacetabular osteotomy even for patients with moderate arthritis has been favorable, deferring the need for total hip arthroplasty by a mean of 6.5 years.

     

     

     

    2020

     

  198. Figure 74 shows the radiograph of an 84-year-old woman who reports severe right knee pain. At the time of total knee arthroplasty, she is found to have gross insufficiency and attenuation of the medial collateral ligament (MCL) complex. Optimal management should consist of

     

     

     

     

    1. primary repair of the MCL and use of a posterior stabilized total knee arthroplasty (TKA) prosthesis.

    2. augmentation of the MCL with a collagenous tissue scaffold and use of a posterior stabilized TKA prosthesis.

    3. complete release of the lateral collateral ligament (LCL) and use of a posterior stabilized TKA prosthesis.

    4. lateral unicompartmental arthroplasty.

    5. use of a varus-valgus constrained TKA prosthesis.

     

    Corrent answer: 5

     

    Patients with severe valgus deformity may have near complete attenuation of the MCL. Attempts at ligament repair or reconstruction at the time of TKA can have unpredictable outcomes, leading to an unstable TKA. Although there may be a role for trying to reconstruct the ligament in conjunction with a nonconstrained implant in young patients with long life expectancies, in elderly patients a constrained prosthesis can provide varus-valgus stability with a predictable outcome. In younger patients, there is concern that the extra prosthetic constraint may shorten the longevity of the prosthetic fixation. In older patients, the constrained implant is likely to last a lifetime, with several studies documenting excellent survivorship (96%) at 10 years. Complete release of the LCL will leave the knee grossly unstable medially and laterally, and could necessitate a hinged prosthesis.

     

     

     

    2020

     

  199. Figure 75 shows the radiograph of a healthy 52-year-old woman who has severe right hip pain that has been unresponsive to nonsurgical management. What is the most appropriate surgical procedure at this time?

     

     

     

    1. Total hip arthroplasty

    2. Hemiarthroplasty of the hip

    3. Valgus femoral osteotomy

    4. Periacetabular osteotomy

    5. Varus femoral osteotomy Corrent answer: 1

    The patient has developmental dysplasia of the right hip, as can be noted by the shallow acetabulum and lack of femoral head coverage. She has secondary osteoarthritis, manifested radiographically by joint space narrowing, periarticular sclerosis, and subchondral acetabular cyst formation. Femoral and/or acetabular redirectional osteotomies are most effective when performed before the onset of arthritis. After osteoarthritis has set in, total hip arthroplasty is the most reliable procedure for reducing pain.

    Hemiarthroplasties are not indicated in the presence of arthritic changes of the acetabulum.

     

     

     

    2020

     

  200. Which of the following surgical maneuvers is most likely to enhance proper patellar tracking during total knee arthroplasty and minimize the need for a lateral retinacular release?

    1. Use of a mobile-bearing prosthesis

    2. Anterior placement of the tibial tray

    3. Internal rotation of the femoral component

    4. Internal rotation of the tibial component

    5. External rotation of the tibial component Corrent answer: 5

    Slight external rotation of the tibial component medializes the tibial tubercle and helps reduce the tendency for lateral patellar maltracking. Mobile bearings have not been shown to improve patellar tracking. Internal rotation of the tibial or femoral component will accentuate patellar maltracking. AP positioning of the tibial tray will affect the force across the patella, but it is not likely to affect patellar tracking.

     

     

     

    2020

     

  201. An 83-year-old man with a history of diabetes mellitus reports abdominal pain on postoperative day number three following a total hip arthroplasty. The patient reports having a bowel movement the prior evening. Examination reveals that the abdomen is distended but nontender. What is the next step in management?

    1. Rectal examination for occult blood

    2. Insertion of a nasogastric tube with lavage to identify blood in the gastric contents

    3. Radiograph of the abdomen

    4. Ultrasound of the abdomen

    5. CT of the abdomen Corrent answer: 3

    The patient has risk factors, symptoms, and signs of Ogilvie syndrome of acute colonic pseudo-obstruction. This unusual but potentially catastrophic complication is characterized by functional colonic obstruction without an associated mechanical blockage. This disorder has been associated with advanced age, male gender, the use of narcotic pain medications, and patients who have undergone hip arthroplasty. The first step in management of any complication is diagnosis, and the diagnosis is most rapidly made using radiographs that show dilation of the large intestine.

     

     

     

    2020

     

  202. A 46-year-old man reports occasional squeaking of his hip 2 years after undergoing an uneventful total hip arthroplasty. History reveals no pain, physical examination cannot reproduce audible

    squeaking, and radiographs show appropriate implant position. What is the most appropriate management?

    1. Revise the cup bearing to polyethylene

    2. Revise the cup bearing to polyethylene and replace the femoral head with a metal design

    3. Increase the cup abduction angle

    4. Decrease the cup abduction angle

    5. Continue routine follow-up and observation Corrent answer: 5

    In the absence of component malpositioning, hip pain, or other compelling reasons to reoperate, a squeaking ceramic bearing is not an indication for revision surgery. The patient can be reassured and observed. Hopefully, with a better understanding of acoustic phenomena following ceramic total hip arthroplasty, this complication can be minimized.

     

     

     

    2020

     

  203. A healthy 74-year-old man reports right knee pain and swelling 6 years after undergoing primary total knee arthroplasty. Radiographs are shown in Figures 80a and 80b. He states he had severe bilateral bowlegged deformity for the last 20 years, but the right side got "a little better after the knee arthroplasty." The infection work-up is negative, and a review of a bone scan and serial radiographs reveals no signs of component loosening. Lateral ligaments appear lax on examination. Optimal surgical management should consist of which of the following?

     

     

     

     

     

     

    1. Tibial polyethylene exchange

    2. Tibial polyethylene exchange and lateral collateral ligament reconstruction and medial collateral ligament release

    3. Revision of all components and soft-tissue balancing

    4. Isolated tibial revision

    5. Femoral revision and tibial polyethylene exchange Corrent answer: 3

    The prosthesis has failed 6 years after implantation. In this patient, the tibial component is in 5 degrees of varus, so the varus mechanical axis was never restored to neutral. The polyethylene bearing is significantly narrowed

    medially, a sign of incomplete release of the tight medial structures and residual varus angulation of the limb. A polyethylene exchange alone will not correct the varus malalignment of the tibia, nor will a femoral revision with polyethylene exchange. An isolated tibial revision will correct the varus but will probably render the posterior cruciate ligament incompetent, necessitating a femoral revision to a posterior cruciate-substituting design. The procedure most likely to result in a successful outcome is a full revision. Soft-tissue releases to balance the medial and lateral sides also are required.

     

     

     

    2020

     

  204. Which of the following organisms is most often found in a late (> 3 months) infection of a total hip arthroplasty?

    1. Staphylococcus aureus

    2. Staphylococcus epidermidis

    3. Group B streptococcus

    4. Eschericia coli

    5. Streptococcus viridans

     

    Corrent answer: 2

     

    Staphylococcus epidermidis is the most common organism found in an infected total hip arthroplasty greater than 3 months from the origional surgery.

    Staphylococcus aureus is more common in acute postoperative infections, and

    E. coli is associated with infections of the urinary tract. Streptococcus species are less common.

     

     

     

    2020

     

  205. A patient sustained a periprosthetic femoral fracture. The proximal femur is comminuted and the femoral component is loose. The patient has absent pulses and poor capillary refill. An emergent arteriogram is shown in Figure 82. What is the most appropriate management?

     

     

     

    1. Open reduction and internal fixation of the fracture

    2. Traction, followed by revision total hip arthroplasty and vascular repair

    3. Revision total hip arthroplasty, four compartment fasciotomy, and sequential neurovascular examination

    4. Revision total hip arthroplasty, followed by vascular repair

    5. Traction, followed by vascular shunting, revision total hip arthroplasty, and definitive vascular repair

    Corrent answer: 5

     

    The patient requires emergent revascularization with signs of ischemia of unknown duration. The leg has shortened substantially and initial management requires traction to return the leg to length. This should be followed by revascularization of the leg. A temporary shunt may be helpful until the fracture has been stabilized (revision total hip arthroplasty) with the leg at full length. Definitive vascular bypass should then be performed.

     

     

     

    2020

     

  206. During normal human knee flexion (beginning with the knee fully extended), which of the following statements best describes tibial rotation with respect to the femur?

    1. Rotation is constantly occurring in both directions during the flexion cycle.

    2. The tibia initially externally rotates, then progressively internally rotates.

    3. The tibia initially internally rotates, then progressively externally rotates.

    4. The tibia initially internally rotates, then remains in that rotational position until deep flexion when further internal rotation occurs.

    5. The tibia initially externally rotates, then remains in that rotational position until deep flexion when further external rotation occurs.

    Corrent answer: 4

     

    During knee flexion, the tibia initially rotates internally in approximately the first 20 degrees and generally maintains this rotational position until flexion past 90 degrees when significantly more internal rotation occurs.

     

     

     

    2020

     

  207. Which of the following polyethylene manufacturing processes is expected to generate the greatest degree of polyethylene oxidation?

    1. Sterilization with ethylene oxide

    2. Sterilization with gamma radiation in a vacuum condition

    3. Sterilization with gamma radiation in air

    4. Sterilization with gamma radiation in oxygen-free gasses

    5. Cross-linking the polyethylene followed by melting Corrent answer: 3

    Oxidation of polyethylene has been associated with increased rates of polyethylene wear. Oxidation occurs after polyethylene has been irradiated in the presence of oxygen. Gamma irradiation has been commonly employed to sterilize the polyethylene prior to sterile packaging. Over the last decade, several methods of reducing oxidation of polyethylene have been used. These include irradiation in an inert gas (such as argon or nitrogen), irradiation in vacuum packaging, and avoiding irradiation altogether and sterilizing the polyethylene with ethylene oxide, gas plasma, or vaporized hydrogen peroxide. Cross-linking polyethylene has been done with gamma irradiation and electron beam irradiation. Heating/melting the material after irradiation allows the free radical chains within the polyethylene to cross-link together rather than oxidize.

     

     

     

    2020

     

  208. When compared to total hip arthroplasty, hip resurfacing offers which of the following advantages?

    1. Better patient compliance with precautions

    2. Increased patient activity in sports

    3. Increased mobility of the hip

    4. Improved pain relief

    5. Preservation of proximal femoral bone Corrent answer: 5

    When compared to the outcomes of conventional hip arthroplasty with large diameter femoral heads, hip resurfacing does not result in better outcomes in terms of compliance with hip precautions, increased hip mobility, pain relief, or improved patient activity. The advantage of hip resurfacing is in preserving proximal femoral bone stock that may be advantageous in revision surgery, should that become necessary.

     

     

     

    2020

     

  209. A 78-year-old woman underwent total hip arthroplasty 15 years ago. She reports a recent history of increasing thigh pain prior to a fall and is now unable to ambulate. Radiographs are shown in Figures 87a and 87b. What is the best treatment for this condition?

     

     

     

     

     

     

    1. Surgical traction for 6 weeks followed by application of a cast brace

    2. Application of a femoral cable plate

    3. Femoral revision with a cemented long stem prosthesis

    4. Application of cerclage wired double allograft femoral struts

    5. Femoral revision with a cementless long taper fluted modular stem and proximal allograft strut supplementation

    Corrent answer: 5

     

    Severe periprosthetic fractures after total hip arthroplasty with a loose implant and progressive bone loss are difficult problems for orthopaedic surgeons, with a high complication rate. Recent literature favors the use of long fluted

    tapered stems that have a long distal taper that may optimally engage the remaining femoral shaft isthmus. Plating options are problematic because the ability to use screws with the plate is limited by the intramedullary stem.

    Although not the only solution to this problem (such as allograft-prosthetic composites, impaction grafting, tumor prostheses), long distally fixed stems circumvent this problem by enhancing fracture healing and create a long-term prosthetic solution in these most difficult cases.

     

     

     

    2020

     

  210. Which of the following has been associated with an increased likelihood of stress shielding after cementless total hip arthroplasty?

    1. Use of titanium alloy femoral components

    2. Use of proximally fixed femoral components

    3. Use of distally fixed femoral components

    4. Use of press-fit stems in patients with narrow intramedullary canals

    5. Use of plasma sprayed components Corrent answer: 3

    Although stress shielding has not been associated with adverse clinical outcomes to date, it is a commonly observed process after cementless total hip arthroplasty. Factors that increase the magnitude of stress shielding include the use of distally fixed cobalt-chrome stems, particularly in patients with osteoporosis. Large diameter femoral components (larger than 18 mm in diameter) have also been associated with an increased prevalence of stress shielding.

     

     

     

    2020

     

  211. During total hip arthroplasty, neurologic injury most commonly occurs in which of the following structures?

    1. Inferior gluteal nerve

    2. Obturator nerve

    3. Peroneal branch of the sciatic nerve

    4. Tibial branch of the sciatic nerve

    5. Femoral nerve

     

    Corrent answer: 3

     

    The incidence of nerve injury with total hip arthroplasty is approximately 1%. The sciatic nerve is involved roughly 80% of the time, with the peroneal

    branch being almost always involved. Isolated tibial branch involvement is reported to occur in only 1% of neurologic injuries related to hip arthroplasty. The superior gluteal nerve may be injured in direct lateral approaches.

     

     

     

    2020

     

  212. A 71-year-old man underwent an uncomplicated hybrid ceramic-on-conventional polyethylene hip arthroplasty 8 years ago. He now has minimal hip symptoms, but radiographs show massive acetabular osteolysis. An AP pelvis radiograph is shown in Figure 91a and a sagittal CT scan is shown in Figure 91b. What is the most appropriate treatment?

     

     

     

     

     

     

    1. Symptomatic treatment with analgesics and a follow-up radiograph in 2 years

    2. Revision of the acetabular component and exchange of the ceramic head

    3. Revision of both the acetabular and femoral components

    4. Revision of the acetabular liner, bone grafting, and exchange of the femoral head to metal

    5. Revision of the acetabulum and exchange of the femoral head to metal Corrent answer: 4

    The patient has massive osteolysis for which surgery is indicated. Follow-up in 2 years risks further progression of osteolysis and the potential for

    catastrophic failure. The acetabular component appears well-fixed (CT scan shows a pillar of bone extending down to the dome of the acetabulum) and well positioned and removal should be avoided because the resulting defect may be extremely large and difficult to reconstruct. A liner exchange is favored, provided that the cup is well positioned and of sufficient size to accept a new liner with adequate thickness. In this case, bone grafting behind the socket (without destabilizing it) would be indicated, although routine bone grafting of osteolytic defects is controversial. Revising a ceramic head to a new ceramic head should also be avoided because ceramic head fractures have occurred with placement on a used taper. Metal adapter sleeves are available for some implants if a ceramic head is desired.

     

     

     

    2020

     

  213. In total knee arthroplasty, what would be a difference with use of the tibial component shown in Figure A compared to Figure B?

     

     

     

     

     

     

    1. Increased ductility of the polyethylene

    2. Increased wear on the articular side of the polyethylene

    3. Less wear on the articular side of the polyethylene

    4. More wear at the backside of the polyethylene

    5. Less wear at the backside of the polyethylene Corrent answer: 4

    Adding a polyethylene-metal interface by making it modular leads to more wear on the backside, not the articular surface.

     

    Modularity with metal tibial base plates has the advantage of being able to customize implants intraoperatively. However, a disadvantage of modularity is increased rates of osteolysis and backside polyethylene wear due to micromotion between tibial baseplate and undersurface of polyethylene that occurs during loading. A monoblock tibial component creates intraoperative challenges as determination of tibial component size is done prior to

    cementing.

     

    Weber et al. reviewed 1000 TKAs performed with 5- to 11-year follow-up. Of tibial components, 698 were monoblock and 353 were modular assembly design. Higher rates of osteolysis, radiolucent lines, and revision were found with the modular design. The TKAs done with a monoblock design gave better clinical and radiologic results.

     

    Collier et al. reviewed 365 TKA cases followed for 5-10 years. They concluded that polishing the tibial baseplate counterface and avoiding polyethylene gamma radiation in air led to decreased rates of osteolysis.

     

    Figure A shows an example of a modular polyethylene-metal interface design. Figure B shows an example of an all-polyethylene tibial component.

     

    Incorrect Answers:

    Answer 1: Tibial component modularity does not affect the ductility of the polymer.

    Answers 2 and 3: Polyethylene wear does not change with modularity on the articular side.

    Answer 5: Polyethylene wear increases on the backside of modular polyethylene tray.

     

     

     

    2020

     

  214. Advantages of a resurfacing metal-on-metal hip arthroplasty over a large diameter metal-on-metal total hip arthroplasty include which of the following?

    1. Lower risk of femoral component loosening

    2. Acetabular bone preservation

    3. Lower reoperation rate

    4. Femoral bone preservation

    5. Lower wear rate Corrent answer: 4

    A resurfacing hip arthroplasty preserves bone stock in the proximal femur, at the expense of a higher reoperation rate because of component loosening and femoral neck fracture. Wear rate is the same as both types of hip arthroplasty use a large head metal-on-metal bearing surface.

     

     

     

    2020

  215. Figure 94 shows the lateral radiograph of an 80-year-old woman who is an independent ambulator and has a supracondylar periprosthetic fracture around the knee. What is the most appropriate management for this patient?

     

     

     

     

     

    1. Closed reduction and nonsurgical management

    2. Open reduction and internal fixation using a distal femoral locking plate

    3. Open reduction and internal fixation using an intramedullary rod

    4. Revision total arthroplasty using distal femoral allograft

    5. Revision total knee arthroplasty using distal femoral replacement Corrent answer: 5

    A supracondylar femoral periprosthetic fracture that is a reducible or an irreducible fracture with poor distal bone stock and in the vicinity of loose or malpositioned components (type III) is very difficult to treat. One of the treatment options for patients with a loose femoral component and poor bone stock is the use of distal femoral replacement. The distal femoral replacement can be performed with relative ease, expediency, and is best suited for elderly or sedentary patients.

     

     

     

    2020

     

  216. Which of the following statements best describes the kinematic behavior of the knee during motion from full extension to flexion?

    1. Both the medial and lateral knee tibiofemoral contact points rotate and translate equally with increasing knee flexion.

    2. The medial femoral condyle translates much less than the lateral femoral condyle with knee flexion.

    3. The lateral femoral condyle translates much less than the medial femoral condyle with knee flexion.

    4. The medial compartment rotates internally whereas the lateral compartment rotates externally.

    5. The lateral compartment rotates internally whereas the medial compartment rotates externally.

    Corrent answer: 2

     

    During normal knee flexion, knee kinematic analysis reveals that the medial tibiofemoral contact point moves very little (translates) in the anterior-posterior direction, whereas the lateral contact point moves much greater in the anterior-posterior direction (translates), resulting in more lateral translation, rollback, and medial pivoting.

     

     

     

    2020

     

  217. A 71-year-old businessman reports medial knee pain recalcitrant to nonsurgical management. Examination reveals that his body mass index (BMI) is 28 and he has a mild varus deformity with a range of motion from 5 degrees to 130 degrees of flexion. Anterior drawer and Lachman's test are negative. Radiographs are shown in Figures 98a through 98c. For cultural and religious reasons, he is concerned about maintaining his range of motion and kneeling ability. Which of the following options is best?

     

     

     

     

     

     

     

     

    1. High tibial osteotomy

    2. Unicompartmental knee arthroplasty

    3. Cruciate-retaining total knee arthroplasty with a fixed bearing design

    4. Posterior stabilized total knee arthroplasty with a fixed bearing design

    5. Cruciate-sacrificing total knee arthroplasty with a rotating platform design Corrent answer: 2

    Unicompartmental arthroplasty of the knee is associated with better range of motion than either total knee arthroplasty or high tibial osteotomy. In a prospective randomized trial of unicompartmental and total knee arthroplasty for patients with medial compartment osteoarthritis, patients with the unicompartmental prosthesis had better range of motion. The literature that compares range of motion in cruciate-retaining as opposed to posterior stabilized and fixed bearing as opposed to mobile-bearing total knees suggests relatively equivalent range of motion between these designs.

     

     

     

    2020

     

  218. At the time of revision total knee arthroplasty, the surgeon is trialing the knee and finds that it extends fully and is stable in flexion with a 23-mm trial spacer; however, the patella is impinging on the polyethylene spacer. No augments were used on the femur or the tibia because the components fit well without them. What is the most appropriate action at this time?

    1. Proceed with implantation of the final components.

    2. Perform a Z-lengthening of the patellar tendon.

    3. Increase the size of the femoral component and use posterior femoral

      augments to decrease the size of the flexion gap.

    4. Increase the size of the femoral component and use augments both distally and posteriorly to lower the joint line and decrease the size of the flexion gap.

    5. Place distal femoral augments on the femoral component to lower the joint line.

    Corrent answer: 4

     

    The surgeon in this case is faced with a common scenario at the time of revision total knee arthroplasty and the tendency is to elevate the joint line. Elevation of the joint line is associated with deleterious effects including anterior knee pain, restricted knee flexion, and instability. The error that has been made is resting the femoral component on the bone that is left behind after removal of the prior component; this typically leads to a femoral component that is too small (leading to an enlarged flexion gap) and proximal to where it should be (enlarging the extension gap). Although the flexion and extension gaps are equivalent, joint line elevation has occurred. To correct this problem, the femoral component size should be increased or offset posteriorly (to decrease the size of the flexion gap) and distal femoral augments should be used to decrease the size of the extension gap and restore the joint line to the appropriate level.

     

     

     

    2020

     

  219. A 68-year-old woman is undergoing a cementless medial/lateral tapered femoral placement during a total hip arthroplasty and the surgeon notices a small crack forming in the anteromedial femoral neck with final implant insertion. The most appropriate management should include which of the following?

    1. Placement of a cerclage cable around the femoral neck above the lesser trochanter

    2. Removal of the implant, placement of a cable around the femoral neck above the lesser trochanter, and reinsertion of the implant

    3. Removal of the press-fit implant and cementing of the same femoral stem

    4. Final seating of the cementless femoral component without additional measures

    5. Removal of the cementless femoral component and placement of a revision modular taper-fluted femoral stem

    Corrent answer: 2

     

    The recognized treatment of the proximal periprosthetic fracture is first to identify its extent and then to optimize the correction. Removing the implant

    seems logical to accomplish the identification. Several studies indicate that proximal cerclage wiring is adequate to create a "barrel hoop" stability of the proximal femur. The postoperative management may also include protected weight bearing and periodic radiographs.

     

     

     

    2020

     

  220. Glucosamine supplementation is used in the treatment of osteoarthritis due to its properties of

    1. COX-2 inhibitory effects.

    2. increasing synovial fluid viscosity.

    3. direct spinal pain mediator.

    4. substrate for proteoglycan formation.

    5. inhibiting chondrocyte apoptosis.

     

    Corrent answer: 4

     

    Glucosamine serves as a substrate for the formation of chondroitin sulfate and hyaluronate biosynthesis. Chondroitin sulfate is the major component of the aggrecan molecule, contributing to the hydrophilic properties of cartilage and formation of the viscous elastic layer of cartilage. One meta-analysis of glucosamine efficacy showed an objective slowing of cartilage degeneration measured by radiographic joint space narrowing and WOMAC index scores.

    Glucosamine does not have anti-COX-2 inhibitory effects, does not increase synovial fluid viscosity, is not a pain mediator, and does not inhibit chondrocyte apoptosis.

     

     

     

    2020

     

  221. An obese patient underwent a posterior cruciate ligament-retaining total knee arthroplasty that has failed and the patient is being considered for revision total knee arthroplasty. The patient also has medial collateral ligament deficiency. In selecting a revision knee prosthesis, which of the following features is considered to be essential?

    1. The ability to retain the posterior cruciate ligament

    2. A constraint mechanism that primarily resists varus-valgus loads

    3. A constraint mechanism that primarily resists rotational loads

    4. Femoral rollback

    5. High conformity between the femoral and tibial articulating surfaces Corrent answer: 2

    Obtaining stability and proper alignment of the components is the most important aspect of knee arthroplasty. An obese patient with a deficient medial collateral ligament is at risk of exerting excessively high valgus loads on the prosthesis, and may develop a valgus deformity. Therefore, the most important consideration is to use a constrained implant that incorporates inherent resistance to varus-valgus loads, such as a rotating hinge design or a constrained condylar design. Constrained designs generally have conforming tibiofemoral surfaces. In such patients, retaining the posterior cruciate ligament or allowing femoral rollback are secondary concerns compared to achieving stability and maintaining alignment.

     

     

     

    2020

     

  222. The hip joint allows motion in different anatomic planes. How many degrees of freedom does the hip joint provide and in which anatomic planes?

    1. 2 degrees of freedom in compression and tension

    2. 2 degrees of freedom in flexion and extension

    3. 3 degrees of freedom in flexion-extension, abduction-adduction, and axial rotation

    4. 3 degrees of freedom in anterior-posterior translation, medial-lateral translation, and axial motion

    5. 6 degrees of freedom in flexion-extension, abduction-adduction, axial rotation, anterior-posterior translation, medial-lateral translation, and axial motion

    Corrent answer: 3

     

    Degrees of freedom of a joint are the set of independent rotations and displacements that the joint allows. The hip is a ball-on-socket joint and, as such, allows all rotations but completely resists or "constrains" all translations. In anatomic terms, the hip joint provides three degrees of freedom: flexion-extension (rotation in the sagittal plane), abduction-adduction (rotation in the frontal plane), and axial rotation (rotation in the transverse plane).

     

     

     

    2020

     

  223. A polyethylene acetabular cup that was packaged in air and sterilized using gamma radiation has been stored unopened for 5 years. Why should this component not be implanted?

    1. It is not crosslinked.

    2. It may not be sterile.

    3. The design may be obsolete.

    4. It may have undergone creep distortion.

    5. It may be substantially oxidized.

     

    Corrent answer: 5

     

    Sterilization using gamma radiation induces free radicals (uncombined electrons) in the polyethylene. During storage in air, oxygen will diffuse into the material and react with the free radicals, causing breakage of the molecular chains. This oxidation can markedly weaken the material, making it much more susceptible to wear and fatigue fracture. The gamma dose used for sterilization, in the range of 2.5 to 4 Mrads, induces a moderate amount of crosslinking, which (in the absence of oxidation) improves the wear resistance compared to noncrosslinked polyethylene. If the packaging has not been damaged, the component should still be sterile, even after 5 years of storage. Creep distortion should not be a problem because the component has not been under load, and the design may still be appropriate for clinical use.

     

     

     

    2020

     

  224. In a total hip prosthesis, using a larger diameter ball (for example, 36 mm instead of 28 mm) can improve stability. All other factors being equal, the volumetric rate of wear of the bearing surfaces would be expected to be greater with a larger diameter, due to the longer sliding distance per step. However, in laboratory tests of metal-metal bearings, larger diameter metal-metal hips, such as surface replacements, have shown wear rates comparable to conventional, smaller diameter metal-metal hips. What is the most likely explanation for this?

    1. Different diameters have comparable amounts of boundary lubrication.

    2. Larger bearings are more flexible.

    3. Larger bearings have greater sliding speed.

    4. Larger bearings have more surface carbides.

    5. Larger bearings have smaller diametral clearance.

     

    Corrent answer: 3

     

    Metal-metal bearings operate in a "mixed lubrication" mode, ie, with part of the surface in direct contact and other parts separated by fluid film. As the diameter of the bearing increases, the total sliding distance per step increases in proportion. If all other factors were unchanged, the volumetric wear per

    step also would increase in proportion. However, the sliding speed of the bearing surfaces also increases in proportion, which tends to draw in more fluid. This increases the separation of the bearing surfaces and tends to offset the negative effect of the increased sliding distance.

     

     

     

    2020

     

  225. Crosslinking of ultra-high molecular weight (UHMW) polyethylene has been shown to increase its resistance to wear, both in laboratory studies and in up to 10 years of clinical use. As the crosslinking dose is increased above the traditional range for gamma radiation sterilization (ie, 2.5 to 4 Mrads), wear resistance progressively increases. What is the principal negative effect of increasingly high crosslinking?

    1. The polyethylene becomes yellowish in color.

    2. The polyethylene becomes dimensionally unstable.

    3. The polyethylene oxidizes.

    4. The polyethylene loses fatigue strength.

    5. The polyethylene absorbs joint fluid.

     

    Corrent answer: 4

     

    All other factors equal, the fatigue strength of UHMW polyethylene decreases somewhat as the level of crosslinking increases. Some types of crosslinking may impart a slight yellowish or grey color, but this has no clinical consequence. Crosslinking also has no substantial effect on dimensional stability. Although radiation crosslinking induces free radicals in the polyethylene that could lead to oxidation, these can be neutralized using suitable post-crosslinking thermal treatments, or by the addition of antioxidants, such as vitamin E.

     

     

     

    2020

     

  226. The radiographs of a cemented total knee arthroplasty show substantial amounts of osteolysis underneath the tibial baseplate. There is no evidence of excessive wear of the superior surface of the polyethylene. The patient has had no previous implants, and the prosthesis has no intramedullary stem. What is the most probable cause of the osteolysis?

    1. Stress shielding caused by the implant, leading to bone resorption

    2. Polyethylene wear debris from the tibiofemoral surface articulation

    3. Metal wear debris from the tibiofemoral surface articulation

    4. Cement debris from the cement-bone fixation surfaces

    5. Polyethylene fretting wear debris from the backside of the insert Corrent answer: 5

    A number of studies have shown that in some designs of total knee implants, micromotion and fretting between the backside of the polyethylene insert and the tibial tray can generate significant amounts of wear debris, sufficient to cause substantial osteolysis. Stress shielding is not a likely cause in this case because there is no intramedullary stem. In general, articulation of metal against polyethylene generates insignificant amounts of metal wear debris.

     

     

     

    2020

     

  227. All of the following are absolute or relative contraindication to hip resurfacing arthroplasty EXCEPT?

    1. 1.5 cm femoral head bone cysts

    2. Acetabular dysplasia

    3. Coxa vara

    4. Femoral neck bone stock deficiency

    5. Age less than 50-years-old Corrent answer: 5

    Absolute contraindications for resurfacing include deficiency of femoral head or neck bone stock or a small or bone-deficient acetabulum.

     

    As described by Schmalzried et al., relative contraindications including coxa vara, female sex, large bone cysts greater than 1cm, or major osteophytic changes in the head neck junction. Coxa vara specifically has been associated with early failure of hip resurfacing.

     

    Gupta et al. performed 3D finite element analysis to predict long term bone density changes after hip resurfacing. They showed concerning long-term regions of strain at the head-neck junction which may predispose patients to late femoral neck fracture after resurfacing.

     

    In a retrospective review, Eastaugh-Waring et al. evaluated what percentage of young adult patients undergoing a total hip arthroplasty would have been canidates for hip resurfacing and found that in approximately 50%, a hip resurfacing may have been indicated.

     

     

    2020

     

  228. A 45-year-old distance runner has a hyaluronic acid injection to his knee because of degenerative arthritis. He immediately develops a severe rash and a systemic hypersensitivity reaction. This patient most likely is also allergic to which of the following?

    1. Penicillin

    2. Sulfur

    3. Shellfish

    4. Chicken or eggs

    5. Lidocaine

     

    Corrent answer: 4

     

    Preparations of hyaluronic acid can be divided into low and high molecular weight compounds. Contraindications to intra-articular hyaluronic acid include joint or skin infection, overlying skin disease, and allergies to chicken or egg products if using a preparation derived from rooster comb.

     

     

     

    2020

     

  229. Figure 21 shows a coronal T1-weighted MRI scan of the knee. The arrow indicates what anatomic structure?

     

     

     

     

    1. Posterior cruciate ligament: anterolateral bundle

    2. Posterior cruciate ligament: posteromedial bundle

    3. Meniscofemoral ligament

    4. Popliteus

    5. Oblique popliteal ligament Corrent answer: 3

    The arrow is pointing to the meniscofemoral ligament. The meniscofemoral ligament connects the posterior horn of the lateral meniscus to the intercondylar wall of the medial femoral condyle. The ligament of Humphrey passes anterior to the posterior cruciate ligament (PCL), whereas the ligament of Wrisberg passes posterior to the PCL. One or the other has been identified in 71% to 100% of cadaver knees, with the ligament of Wrisberg being more common. The PCL is shown inferior to the indicated structure. The popliteus and oblique popliteal ligament are not visualized in this image.

     

     

     

    2020

     

  230. Which of the following surgical approaches to the knee has the greatest potential for denervation of the quadriceps muscle?

    1. Subvastus

    2. Midvastus

    3. Quadriceps sparing

    4. Median parapatellar

    5. Tibial tubercle osteotomy Corrent answer: 2

    Several studies have demonstrated excellent functional results and recovery after total knee arthroplasty (TKA) with a variety of minimally invasive approaches; however, studies have demonstrated abnormal electromyographic (EMG) studies in a significant number of TKAs performed using a midvastus exposure. Patients whose vastus medialis intervals were developed bluntly were significantly more likely to fully recover normal EMG activity than if the intervals were developed with sharp dissection.

     

     

     

    2020

     

  231. A 61-year-old female who reports left hip pain is seen in the emergency department. Figure 41a shows a radiograph obtained at that time. Ten months later, she reports excruciating left hip pain with ambulation. She notes that the pain has markedly worsened over the

    past several weeks. Figures 41b and 41c show a current radiograph and a coronal inversion recovery MRI scan of the pelvis. What is the most likely diagnosis?

     

     

     

     

     

     

     

     

     

     

    1. Infection of the hip

    2. Fracture of the hip

    3. Osteoarthritis of the hip

    4. Osteonecrosis of the hip

    5. Rheumatoid arthritis of the hip Corrent answer: 4

    The initial radiograph shows subtle flattening of the left femoral head, suggestive of osteonecrosis but without significant subchondral sclerosis. Figure 41b shows marked collapse in the left head over the intervening 10 months, and the MRI scan reveals collapse, significant edema in the head, and low signal intensity in the superior segment, all suggestive of osteonecrosis.

    Note that the right hip shows MRI changes, suggesting bilateral disease in this patient.

     

     

     

    2020

     

  232. During an anterior approach to the hip, what structure has the greatest potential for injury?

    1. Femoral nerve

    2. Femoral artery

    3. Femoral vein

    4. Lateral femoral cutaneous nerve

    5. Medial femoral circumflex artery Corrent answer: 4

    The anterior (Smith-Peterson) approach to the hip develops the superficial interval between the tensor fascia lata (TFL) and sartorius and the deep interval between the gluteus medius and rectus femoris. The lateral femoral cutaneous nerve penetrates the fascia overlying the interval between the TFL and sartorius approximately 1 cm distal to the anterior superior iliac spine.

    Identifying the interval between the TFL and sartorius distally can be helpful in preventing injury to the lateral femoral cutaneous nerve, which is the structure at greatest risk for injury during an anterior approach to the hip. The femoral artery, vein, and nerve are medial to the approach. The medial femoral circumflex artery runs posterior to the femoral neck. The ascending branch of the lateral femoral circumflex artery is routinely encountered during this approach, but is not one of the options.

     

     

     

    2020

  233. After completion of bone preparation during a total knee arthroplasty, the lateral compartment is tight in both flexion and extension. At what point during the release is the peroneal nerve at greatest risk for injury?

    1. Release of the posterior capsule

    2. Release of the posterolateral capsule with the knee extended

    3. Release of the posterolateral capsule with the knee flexed

    4. Release of the iliotibial band with the knee extended

    5. Release of the iliotibial band with the knee flexed Corrent answer: 2

    The peroneal nerve traverses the proximal aspect of the knee joint in the interval between the biceps femoris and lateral gastrocnemius. The lateral gastrocnemius muscle provides some protection for the peroneal nerve.

    Cadaveric studies have suggested that the peroneal nerve can be as close as 7 mm to 9 mm from the posterolateral corner with the knee in extension, where it is at greatest risk for injury. The iliotibial band is anterior to the course of the peroneal nerve at the joint line.

     

     

     

    2020

     

  234. A 60-year-old female underwent hybrid total hip arthroplasty with good position of implants and post-operatively is instructed not to extend, adduct, and externally rotate the hip to prevent dislocation. What approach was likely performed?

    1. Southern

    2. Stoppa approach

    3. Smith-Peterson

    4. Kocher Langenbach

    5. Pfannenstiel approach

     

    Corrent answer: 3

     

    The position to dislocate a hip anteriorly is extension, adduction, and external rotation which is the position at risk after an anterior approach. Hips are most likely to dislocate the direction of the approach assuming the implants are correctly positioned. Other important variables to THA stability include: component design, component alignment, soft-tissue tensioning, and soft tissue function.

     

     

     

    2020

  235. A 58-year-old man has had groin pain for the past 3 months. The patient reports pain with ambulation and at rest. Examination reveals an antalgic gait and range of motion is mildly restricted. He denies any history of trauma, or steroid or alcohol abuse. Radiographs are normal. An MRI scan is shown in Figure 77. What is the most appropriate management?

     

     

     

     

     

    1. Protected weight bearing and anti-inflammatory drugs

    2. Total hip arthroplasty

    3. Intraosseous steroid injection

    4. A vascularized fibula graft to the femoral head

    5. Core decompression of the femoral head Corrent answer: 1

    The patient has transient osteoporosis of the hip. The MRI findings show highly increased signal through the entire femoral head and neck that is diagnostic of transient osteoporosis of the femoral head.á This entity is usually a self-limited condition that is most frequently seen in women in the third trimester of pregnancy and in men in the sixth decade of life.á Transient osteoporosis is best treated nonsurgically with protected weight bearing and anti-inflammatory drugs. The natural history is that of self-resolution. A vascularized fibula graft to the femoral head and core decompression of the femoral head each have a described role in treating osteonecrosis (not transient osteoporosis) depending on the stagingáof the disease. Total hip arthroplasty indications include end-stage osteonecrosis of the hip as well as osteoarthritis. Steroid injections are generally reserved for simple cysts of bone.

     

     

    2020

     

  236. A 62-year-old man returns for evaluation of a painless total knee arthroplasty 6 months after his surgery. He notes recurrent, mild knee effusions. His initial postoperative radiographs are shown in Figures 84a and 84b. His current radiographs are shown in Figures 84c and 84d. What is the next step in evaluation of this patient?

     

     

     

     

     

     

     

    1. Clinical and radiographic follow-up in 3 months

    2. CT scan

    3. Laboratory testing

    4. Stress radiographs

    5. Technetium-99 bone scan Corrent answer: 3

    The development of a progressive radiolucency within the first year following knee arthroplasty surgery is concerning for infection. Infection work-up should include laboratory testing for erthrocyte sedimentation rate and C-reactive protein levels. A joint aspiration should be strongly considered, especially if the laboratory studies are elevated. A CT scan would be appropriate to assess component rotation for patellar instability, but does not benefit evaluation of this patient. Stress radiographs could be useful in confirming clinical instability noted on examination, but early component loosening is the clinical concern for this patient. A bone scan would be expected to show activity at 6 months after surgery and would not add useful information to the work-up.

     

     

     

    2020

     

  237. A 65-year-old woman who underwent right knee arthroplasty 12 years ago reports that she has had knee pain for the past year. Examination reveals that knee range of motion is from 0 degrees to 100 degrees. A standing AP radiograph obtained 3 years ago is shown in Figure 91a. Recent radiographs are shown in Figures 91b and 91c. Laboratory study findings include an erythrocyte sedimentation rate of 9 mm/h and a C-reactive protein level of 0.3 mg/L. What is the most likely cause of her knee pain?

     

     

     

     

     

     

    1. Infection

    2. Ligamentous instability

    3. Polyethylene wear

    4. Extensor mechanism dysfunction

    5. Technical error during the total knee arthroplasty Corrent answer: 3

    The radiographs show a change in femoral component position with proximal migration and a change in alignment from the initial near anatomic to a more varus position. Polyethylene wear is the most common contributor to both

    focal osteolysis and component loosening at long-term follow-up. The knee performed well for 12 years with good initial alignment, so a technical factor at the index surgery would not explain the development of loosening. Laboratory findings are not consistent with infection. There is no clinical information in the history that would suggest that the patient has either instability or poor function in the extensor mechanism.

     

     

     

    2020

     

  238. Range of motion after total knee arthroplasty is best described by which of the following statements?

    1. The principle predictive factor of the postoperative range of motion is the preoperative range of motion.

    2. Intraoperative range of motion is not correlated with the postoperative range of motion.

    3. Postoperative stiffness rarely impairs function.

    4. Excess distal femoral resection with a thick tibial polyethylene is associated with a flexion contracture.

    5. Inadequate distal femoral resection and a tight posterior capsule are associated with loss of flexion.

    Corrent answer: 1

     

    The cause of postoperative stiffness after total knee arthroplasty is multifactorial. Whereas there is no universally accepted definition of stiffness, 90 degrees of flexion is needed to perform tasks such as stair climbing and getting out of a chair and nearly full extension is necessary for efficient gait. Predictors of postoperative range of motion include preoperative and intraoperative range of motion. Capsule release, ligament release, osteophyte removal, and properly sized components are often necessary to optimize range of motion. Excess distal femoral resection with a thick polyethylene will cause a tight flexion gap and loss of flexion. Inadequate distal femoral resection with retained osteophytes and a tight posterior capsule will lead to a flexion contracture.

     

     

     

    2020

     

  239. Figure 49 is the radiograph of a 73-year-old woman who underwent a left knee revision 9 months ago. She states that she has been unable to extend her knee since she fell 6 months ago. Treatment should consist of which of the following?

     

     

     

    1. Knee fusion

    2. Extensor mechanism allograft

    3. Patellectomy with primary repair

    4. Open reduction and internal fixation

    5. Cast immobilization in full extension Corrent answer: 2

    The patient has a chronic extensor mechanism disruption. Attempts at primary repair or open reduction and internal fixation have a low probability of clinical success. Similarly, cast immobilization is not advised as the patient already has a large diastasis between the superior and inferior pole of the patella. An extensor mechanism allograft will provide the most predictable outcome in patients with chronic extensor mechanism disruption following total knee arthroplasty. A knee fusion remains as a surgical option but this should be considered a salvage procedure.

     

     

     

    2020

     

  240. A patient reports startup pain 3 months after undergoing a primary total hip arthroplasty. Figures 69a and 69b show postoperative radiographs at 6 weeks and 3 months, respectively. Laboratory studies reveal a normal CBC count, C-reactive protein, and erythrocyte sedimentation rate. Which of the following options is most appropriate?

     

     

     

    1. Continued observation

    2. Revision of the femoral component

    3. Hip aspiration for cell count and culture

    4. Physical therapy for quadriceps strengthening

    5. Resection arthroplasty, antibiotic spacer, and intravenous antibiotics Corrent answer: 2

    The patient has a loose femoral component, which has subsided at least 1 cm. The stem is undersized which is a risk factor for subsidence, especially with tapered stems. Continued observation is not indicated. Revision total hip arthroplasty is the best option. With a normal erythrocyte sedimentation rate and C-reactive protein, further workup and treatment for infection is not indicated.

     

     

     

    2020

     

  241. Figures 81a and 81b are the radiographs of a 44-year-old woman who reports the development of significant left hip pain over the past 6 months with symptoms located in the groin and buttock. She notes pain while sleeping and increased pain with walking up stairs or sitting for prolonged periods. Examination reveals full range of motion, and internal rotation impingement is absent. The left lower extremity is shorter than the contralateral leg by 1.5 cm. She denies

    lumbar spine symptoms and has a normal neurologic examination. Treatment should consist of which of the following?

     

     

     

     

     

     

     

     

    1. Hip resurfacing

    2. Total hip arthroplasty

    3. Periacetabular osteotomy

    4. Trochanteric varus osteotomy

    5. Trochanteric valgus osteotomy Corrent answer: 3

    Periacetabular osteotomy is an excellent reconstructive procedure in middle-aged adult patients with early hip arthritis and symptoms. The best candidates have a very low vertical center edge angle of 0 degrees to 15 degrees, a minimum of 2 mm of cartilage joint space remaining, and a concentric articulation throughout the range of motion. In these cases, rotational repositioning consisting of moving the dome of the acetabular sourcil both laterally and anteriorly for improved surface area and coverage of the femoral head during weight bearing can produce a long-term solution for this condition. Whereas the surgical technique is challenging, complication rates are low with surgical experience and offer a better alternative than a salvage procedure such as a total joint arthroplasty or hip resurfacing arthroplasty.

    Trochanteric osteotomy has been used for this condition, but does not offer the ability to significantly improve the surface area on the acetabular side of the joint.

     

     

     

    2020

     

  242. Figures 92a and 92b are the radiographs of an elderly patient who underwent revision total hip arthroplasty and was asymptomatic until falling; the patient is now unable to bear weight. What is the most appropriate management?

     

     

     

     

     

    1. Revision of the femoral component with a longer stem

    2. Revision of the femoral component with open reduction and internal fixation with a plate, screws, and cables or wires

    3. Open reduction and internal fixation of the fracture with a plate, screws, and cables or wires

    4. Intramedullary fixation after revision of the stem with a cemented device

    5. Repair with cortical strut allograft and cerclage wires Corrent answer: 3

    The patient has a periprosthetic femur fracture below a well-fixed, long stem femoral component. Because the patient was asymptomatic prior to the fall and the radiographs do not indicate loosening of the femoral component, revision of the femoral component is not indicated. The fracture is a Vancouver type b-1 fracture and repair of the fracture with plates and screws is indicated. Repair with cortical allograft and cerclage wires may serve as an augment to plates and screws but if used alone (without a plate and screw construct), it will not provide adequate rotational control.

     

     

     

    2020

     

  243. An 82-year-old woman underwent cemented right total hip arthroplasty approximately 15 years ago. She fell and sustained the injury shown in Figure 99. What is the most appropriate management for this injury?

     

     

     

     

     

    1. Open reduction and internal fixation of the femur with a plate, screws, and cerclage wires

    2. Open reduction and internal fixation of the femur with a plate, screws,

      cerclage wires, and cortical strut allograft

    3. Revision of the acetabular component with open reduction and internal fixation of the femur with a plate, screws, and cortical strut allograft

    4. Revision of the acetabular and femoral components

    5. Revision of the femoral component with a long cemented stem Corrent answer: 4

    The radiograph demonstrates a Vancouver type b-3 periprosthetic femur fracture with a loose femoral stem and acetabular component. Open reduction and internal fixation would not be appropriate in the setting of loose components. The femur has circumferential radiolucent lines in the bone-cement interface and the bone quality of the proximal femur is poor. In addition, the cemented acetabular component is loose and has migrated proximally. Appropriate treatment includes acetabular revision and femoral revision. Options for femoral revision include an allograft-prosthesis complex, extended osteotomy with cables, or a proximal femoral replacement.

     

     

     

    2020

     

  244. Five weeks following total knee arthroplasty, a woman has intermittent knee drainage for 1 week. Clear serous drainage is coming from her wound from a small area in the central portion of her incision. Her medical comorbidities include hypertension and a BMI of

    50. Fluid aspirated from the knee shows a WBC of 11,500/mm3 with 92% polymorphonuclear cells. Methicillin-resistant Staphylococcus aureus grows from an aspirate on day 2. What is the next step in management?

    1. One-stage revision using antibiotic-containing cement

    2. Two-stage revision using an antibiotic cement spacer

    3. Irrigation and debridement with polyethylene exchange

    4. Infectious disease consult and appropriate oral antibiotics for 6 weeks

    5. Infectious disease consult and appropriate IV antibiotics for 6 weeks Corrent answer: 2

    Infections that are diagnosed early have historically been treated with irrigation and debridement and IV antibiotics. However, published literature shows that this treatment is associated with success rates of less than 50%. The presence of resistant bacteria in the setting of morbid obesity and persistent drainage further decreases the success rate. A recent paper presented at the AAOS in 2010 showed poorer outcomes following two-stage revision in those patients in whom an attempt at component retention with

    irrigation and debridement had been performed first. Therefore, a two-stage revision with the use of an antibiotic cement spacer is likely to give a better outcome in this patient.

     

     

     

    2020

     

  245. Which of the following is a characteristic change in cartilage involved in moderate osteoarthritis?

    1. Decreased thickness of subchondral bone

    2. Decreased chondrocyte activity

    3. Increased proteoglycan quantity

    4. Increased water content

    5. Increased collagen quantity Corrent answer: 4

    The changes observed in articular cartilage affected by osteoarthritis include: increased water content (as a result of the disruption in architecture of the matrix molecules), decreased quantity of proteoglycans, decreased quantity of collagen with decreases in cross-linking and stiffness associated with degradation, increased thickness of subchondral bone, and increased chondrocyte activity and proliferation of chondrocytes.

     

    This is different than the the changes seen during the natural aging process which include: decreased water content, decreased proteoglycan content and size (decreased length of protein core and GAG chains), normal quantity of collagen, increased collagen stiffness and cross-linking, unchanged chondrocyte activity/proliferation, unchanged thickness of subchondral bone, and decreased ratio of chondroitin sulfate to total glycosaminoglycans.

     

    Hunziker reviewed the literature and found the existence of many novel and very promising biologically-based approaches for the induction of articular cartilage repair, the vast majority of which are still at an experimental phase of development.

     

    Incorrect Answers:

    Answer 1: Subchondral bone thickness is increased in osteoarthritis. Answer 2: Chondrocyte activity is increased in moderate osteoarthritis. Answer 3: Proteoglycan content is decreased in both osteoarthritis and the normal aging process.

    Answer 5: Collagen quantity is decreased in severe osteoarthritis and remains unchanged in the normal aging process.

     

     

    2020

     

  246. All of the following are risk factors for developing a sciatic nerve palsy following total hip arthroplasty EXCEPT:

    1. Female gender

    2. Developmental dysplasia of the hip

    3. Revision surgery

    4. Rheumatoid arthritis

    5. Lengthening of the extremity Corrent answer: 4

      Female gender, DDH, revision surgery, and extremity lengthening are all risk factors for the development of a post-operative sciatic nerve palsy following total hip arthroplasty. Rheumatoid arthritis is not a risk factor.

       

      Nerve palsy is an uncommon but devastating injury following total hip arthroplasty. Common causes include compression from hematoma or tight dressings, direct injury, and excess heat from polymethylmethacralate polymerization. Initial management in the post-operative period is to place the hip in extension and the knee in flexion to take tension off the sciatic nerve. If a hematoma is thought to be the cause, evacuation in the operating room is the preferred treatment.

       

      Schmalzried et al in CORR 1997 found the risk for post-operative palsy increased in females, those with a diagnosis of developmental dysplasia, and patients undergoing revision surgery. Obesity, RA, Perthes, and SCFE were not associated with nerve palsy following THA.

       

      Farrell et al in a retrospective analysis of 27,000 primary THA found a 0.17% incidence of nerve plasy. Risk factors included preoperative diagnosis of DDH or posttraumatic arthritis, the use of a posterior approach, lengthening of the extremity, and cementless femoral fixation. Only 10 of the 28 patients at final follow-up had complete resolution of symptoms.

       

      Illustration A shows an excellent representation of the sciatic nerve as it courses beneath the piriformis.

       

       

       

       

       

      2020

       

  247. Figure 136 is the radiograph of a 68-year-old man who reports persistent pain after undergoing total hip arthroplasty. Examination reveals equal limb lengths and there is minimal discomfort with straight-leg raise or hip rotation. When asked to ambulate, however, he has discomfort with the first few steps, and then can walk more comfortably. C-reactive protein and erythrocyte sedimentation rates are normal. Management should now consist of

     

     

     

     

     

    1. an indium scan.

    2. a three-phase bone scan.

    3. revision of the femoral component with a cemented stem.

    4. revision of the femoral component with a cementless stem.

    5. cortical strut allografting of the femoral stress fracture.

    Corrent answer: 4

     

    The patient has a 100% radiolucent line around the femoral component as well as a distal pedestal indicating loosening of the femoral component. With these findings, there is no need to do further workup for loosening with a bone scan. Infection is unlikely with the normal laboratory findings so an indium scan is not necessary. Allograft for the femur may improve pain with a stress fracture but not in the setting of a loose femoral component. Revision of the femoral component with a cementless stem after removing the fibrous endosteal tissue and the distal pedestal is associated with the best results. Cemented revision stems in this setting are associated with early failure.

     

     

     

    2020

     

  248. Figure 146 is the radiograph of a 72-year-old woman with a history of Parkinson's disease and a multiply revised right total hip arthroplasty with a constrained implant. She is seen in the emergency department, reporting pain. Treatment should consist of which of the following?

     

     

     

     

     

    1. Closed reduction

    2. Open reduction

    3. Open reduction with soft-tissue repair

    4. Acetabular revision with a constrained implant

    5. Acetabular revision with an unconstrained implant Corrent answer: 4

    The patient has a hip dislocation with a previously placed constrained acetabular component. The ring around the femoral neck is the locking ring of a constrained implant that has dissociated. The acetabular component demonstrates increased vertical inclination and retroversion. The acetabular component malposition contributed to the dislocation along with the patient's deficient abductor musculature. The appropriate treatment would be to

    perform an acetabular revision to improve the component position along with a constrained liner due to the deficient abductors. A closed reduction will be extremely difficult to achieve because of the presence of a constrained liner, whereas an open reduction is not advised because of the persistent problem of component malposition.

     

     

     

    2020

     

  249. A 45-year-old woman undergoes an uncomplicated total knee arthroplasty. Nine months later she has not yet returned to work because of pain and stiffness. Her range of motion is 5 to 80 degrees. She has no instability, is unable to climb stairs normally, sitting in low chairs is uncomfortable, and she no longer participates in physical therapy. She has pain with prolonged standing. Radiographs show a well-aligned, cruciate-retaining implant. Work-up for infection, including joint aspiration, is negative. What is the next step in management?

    1. Dynamic splinting

    2. Open release of adhesions

    3. Manipulation under anesthesia

    4. Revision to a cruciate-sacrificing implant

    5. Arthroscopic release of adhesions and manipulation Corrent answer: 5

    Arthrofibrosis after total knee arthroplasty (TKA) affects 1% of patients. Ninety degrees of motion allows for most activities, including ascending and descending stairs. Risk factors for postoperative stiffness include preoperative stiffness, younger age, posttraumatic arthritis, and multiple prior surgeries.

    Whether range of motion is affected by the choice of cruciate retaining versus sacrificing implants is subject to much debate, but has not been shown to be related to arthrofibrosis following TKA. At earlier time points, physical therapy, use of dynamic splinting, and manipulation under anesthesia may be beneficial in restoring motion but at 9 months is unlikely to prove successful. However, the patient has both pain and motion loss and is 9 months from her original surgery. Late manipulation may have an increased risk of complications such as fracture or tendon rupture. Moreover, arthroscopy would allow for lysis of adhesions and assessment for other causes of pain and has been shown to be safe and effective following TKA; however, it does carry the risk of infection.

    Care must be taken not to scratch or otherwise damage the implants during surgery.

     

     

    2020

     

  250. Figures 183a and 183b are the radiographs of an otherwise healthy 62-year-old man with a history of a total knee arthroplasty followed 1 year later by a periprosthetic fracture treated with open reduction and internal fixation. The surgery was complicated by multiple wound infections with a sensitive organism. He eventually had hardware and implant removal and placement of an antibiotic spacer that was subsequently removed. After a full course of antibiotics, retesting reveals persistent infection and he is referred for further treatment. His subsequent treatment should be

     

     

     

     

     

    1. knee fusion.

    2. above-knee amputation.

    3. antibiotic suppression.

    4. arthroscopic irrigation and debridement.

    5. repeat debridement and placement of an antibiotic spacer.

     

    Corrent answer: 5

     

    Two-stage resection with placement of an antibiotic impregnated spacer followed by reimplantation has been demonstrated to have success rates as high as 80% and has become the standard treatment for an infected total joint arthroplasty in the United States. Failure to eradicate the infection can be due to the virulence or drug resistance of the organism, the appropriateness of the antibiotic selection, or the adequacy of the debridement. Retained metal fragments, cement, or devitalized bone can result in failure to clear the

    infection. Special attention should be made to the patella because the exposure for a total knee arthroplasty can remove the majority of its blood supply. A lateral release (or lateral dissection as in this case) can compromise the primary remaining vessel to the patella (the superior lateral genicular) and result in osteonecrosis. In the setting of infection, the devitalized patella may become a large sequestrum and patellectomy should be considered. Antibiotic suppression should be used rarely and would not be a viable option in an otherwise healthy 62-year-old that would require decades of treatment.

    Above-knee amputation is a last resort, and in most situations at least a second attempt at two-stage resection and reimplantation should be attempted first. Knee fusion would not be indicated until the infection was eradicated and is also considered a last resort. Arthroscopic irrigation and debridement would not allow for adequate debridement of the joint and should not be used in the treatment of an infected arthroplasty.

     

     

     

    2020

     

  251. Figures 197a through 197c are the radiograph and MRI scans of a 63-year-old woman who reports the insidious onset of severe right hip pain. Her pain is worse with weight bearing and alleviated with rest. She takes no medications and is otherwise healthy. What is the next best step in her treatment?

     

     

     

     

     

     

     

     

    1. Bone resection and mega-prosthetic reconstruction

    2. Radiation therapy

    3. Prolonged course of antibiotics

    4. Partial weight bearing and observation

    5. Core decompression

     

    Corrent answer: 4

     

    The patient has transient osteoporosis, which most commonly involves the hips. The etiology is unknown but may be related to an interruption of the intraosseous blood supply. Patients have joint pain and usually have normal findings on radiographs or CT scans. The MRI scan shows complete replacement of the marrow on T1-weighted images and marked hyper-intensity of the marrow on T2-weighted sequences. Osteonecrosis of bone would show focal marrow changes and a serpentine line of demarkation.

    Crescent-shaped bone collapse can later be seen on the radiographs. This case does not show radiographic changes of osteonecrosis, but does show early subchondral bone formation in the femoral head. Osteonecrosis would not show early subchondral bone healing. The findings of transient osteoporosis are commonly mistaken for metastatic bone disease; however, the MRI scan does not show a focal mass. The diagnosis of transient osteoporosis can be made by correlating the clinical history of severe pain with the markedly abnormal MRI scan in the face of a normal radiograph and CT scan. Transient osteoporosis is a self-limiting disease. Therefore, surgeons should use a treatment approach based on the clinical symptoms. Current, therapeutic strategies include partial weight bearing, mild analgesics, and administration of nonsteroidal anti-inflammatory drugs. Treatment protocols to avoid include bone resection (malignancy), radiation (malignancy), antibiotics (osteomyelitis), or core decompression (osteonecrosis).

     

     

     

    2020

     

  252. Figure A depicts an individual seen from behind during a single-leg stance on the left lower extremity. Which of the following modifications during a left-sided total hip arthroplasty would exacerbate the abnormal findings present in Figure A?

     

     

     

    1. Decreasing femoral offset

    2. Changing from a standard offset neck to an extended offset neck

    3. Increasing femoral head size

    4. Increasing femoral neck length

    5. Moving acetabular cup inferiorly Corrent answer: 1

    Figure A shows a patient with a Trendelenburg sign as evident by the pelvis tilting down on the right during a single-leg stance on the left lower extremity secondary to weak abductors of the left hip. Decreasing femoral offset would exacerbate this condition because this shortens the abductors, putting the gluteus medius under less tension and therefore placing it at a decreased mechanical advantage and exacerbating any baseline abductor deficit.

     

    Hip biomechanics can be simplified by the equation (Force Abductors x Distance to center of head) = (Force from body weight x Distance to center of head). This relationship can be altered by changing the offset (affects distance of abductor to center of head) or cup position (medial or lateral position of cup affects distance of body weight to center of head). If the force of the abductor can't be improved by changing the distance to the head (offset) patients get a trendelenburg gate. The reason they therefore lurch to the affected side is so that they shift the force of body weight laterally thus decreasing the BW to center of head distance.

     

    Pai et al. reviewed 3 different lateral approaches (transtrochanteric, Liverpool,

    and Hardinge approach) in 264 patients and found no difference in functional level, range of movement, and rates of Trendelenburg gait.

     

    Decreasing femoral offset is shown in Illustration A. Illustration B demonstrates abduction at the hip joint, viewed from behind. Part A of the diagram shows that when weight is borne on the right leg, the pelvis tends to sag on the free, or swing on the left side due to gravity. This is counteracted by abduction of the hip on the right side, primarily by strong contraction of the gluteus medius. In part B of the diagram there is a positive Trendelenburg's sign because the pelvis sags,unsupported, as a result of weak abductors on the stance side. Illustration C shows Force Abductors x Distance to center of head = Force from body weight x Distance to center of head.

     

     

     

     

     

     

     

     

     

     

     

     

    2020

     

  253. The posterior condylar axis may be used to determine the rotation of the femoral component in total knee arthroplasty. Which of the following describes the normal relation of the posterior condylar axis?

    1. Parallel to the transepicondylar axis

    2. Perpendicular to the anteroposterior axis (Whiteside's line)

    3. 3 degrees externally rotated to the transepicondylar axis

    4. 3 degrees internally rotated to the transepicondylar axis

    5. 7 degrees externally rotated to the transepicondylar axis Corrent answer: 4

    To recreate an appropriate rectangular flexion gap, the femoral component should be aligned parallel to the transepicondylar axis. The transepicondylar axis is perpendicular to the anteroposterior axis (Whiteside's line). The posterior condylar axis is normally 3 degrees internally rotated to the transepicondylar axis. Thus, the posterior femoral cut should be made in 3 degrees of external rotation from the posterior condylar axis in order to produce a rectangular flexion gap.

     

    Whiteside reviews this concept and others in his technique review of balancing a total knee arthroplasty.

    An illustration of these axes is provided in Illustration A.

     

     

     

     

     

     

    2020

     

  254. Highly cross-linked ultra-high-molecular-weight polyethylene (UHMWPE) liners used in total hip arthroplasty have which of the following characteristics compared to conventional UHMWPE liners?

    1. Increased average wear particle size

    2. Increased resistance to abrasive wear

    3. Increased number of wear particles generated

    4. Increased resistance to fatigue crack propogation

    5. Increased ductility

     

    Corrent answer: 2

     

    Highly cross-linked ultra-high-molecular-weight polyethylene (UHMWPE) has improved resistance to adhesive and abrasive wear. Highly cross-linked UHMWPE is created by irradiating the polyethylene, which prompts free radicals from different polymer sections to combine to form chemical bonds between two polymer chains (cross-links). The amount of radiation exposure utilized to create these cross-links is between 5 and 10 Mrad with either gamma or electron beam radiation.

     

    The Ries et al study showed that although cross-linking UHMWPE can reduce wear, they also demonstrate decreased mechanical properties (e.g. toughness, ductility, tensile strength, and fatigue strength) when compared with conventional UHMWPE.

     

    McCalden et al peformed a Level 1 study of 100 patients randomized to highly cross-linked UHMWPE or conventional PE. The found that at an average of 6.8 years that the highly cross-linked polyethylene liner had significantly lower wear rates (as determined by femoral head penetration rates on radiographs).

     

    Jacobs et al present a Level 5 review stating that highly cross-linked

    polyethylene has shown significantly decreased wear rates compared with conventional polyethylene in hip simulator studies. Additionally, they state that there have been no reports of clinically relevant osteolysis witnessed with highly crosslinked PE.

     

     

     

    2020

     

  255. Malrotation of total knee components leading to patellar tracking problems is best diagnosed by what radiographic modality?

    1. 3-joint standing x-rays

    2. dynamic examination under anesthesia with fluoroscopy

    3. CT scan of the knee

    4. 45 degree flexed PA x-rays of the knee

    5. magnetic resonance arthrogram of the knee Corrent answer: 3

    Patellofemoral complications (patella subluxation or dislocation, patellar clunk, wear or loosening of the patellar component, and patella fracture) are the most common complications after total knee arthroplasty, occurring in 30% of cases. Poor patella tracking or dislocation can be the result of malrotation of the tibial and/or femoral components, an excessively tight lateral retinaculum, improper patellar component positioning, patellar component loosening, improper axial orientation of the tibial and femoral components, or an overstuffed joint. Although most of these causes, including improper axial alignment, can be determined from plain radiographs or physical examination, as described by Jazrawi et al, implant rotational malalignment is more difficult to assess with plain radiographs alone. Multiple CT protocols have been devised, however, that enable surgeons to ascertain component malrotation and plan subsequent revisions for symptomatic knees, making answer #3 – CT of femoral and tibial components – the correct choice.

     

     

     

    2020

     

  256. Figures 64a and 64b are the radiographs of a 62-year-old woman who is seen in the emergency department after a fall to a flexed knee. She underwent posterior-stabilized total knee arthroplasty 6 years ago. She has no other injuries and was previously a community ambulatory without assistance. What is the most appropriate treatment?

     

     

     

    1. Closed reduction and casting

    2. Closed reduction and fracture bracing

    3. Retrograde intramedullary nailing

    4. Minimally invasive plate osteosynthesis

    5. Revision total knee arthroplasty with distal femoral replacement Corrent answer: 4

    The patient has a displaced periprosthetic fracture just proximal to a posterior-stabilized total knee arthroplasty. Whereas distal bone stock is limited and there is evidence of radiographic osteopenia, there does not appear to be disruption of the bone-prosthetic interface and the prosthesis appears to be well fixed. Surgical fixation is indicated to restore alignment and allow for early range of motion and mobilization in an effort to optimize functional outcomes. Unless the patient is unable to tolerate surgical management, closed reduction and casting or functional bracing should be reserved for stable, minimally displaced or impacted fracture patterns. Whereas retrograde nailing is a viable treatment option, it can be difficult in the presence of a posterior-stabilized prosthetic design and distal fixation can be problematic in the setting of poor bone stock or quality. Minimally invasive plate osteosynthesis using modern-locking periarticular plates allows for indirect reduction techniques in an effort to optimize the biologic milieu and offers optimal biomechanical stability.

    Revisional total knee arthroplasty with a distal femoral replacement is not a good option for a young patient.

     

     

    2020

     

  257. What process has the most positive effect on the wear-resistance characteristics of UHMWPE?

    1. Radiation

    2. Ethylene oxide

    3. Remelting

    4. Annealing

     

    Corrent answer: 1

     

    UHMWPE is a long polyethylene polymer with a high molecular weight. It is manufactured via ram bar extrusion and compression molding. Radiation is used to sterilize and cross-link. Historically, gamma was irradiated in air at 2.5-4.0 Mrad. This created free radicals, resulting in early oxidative degradation with high wear rates, delamination, and fracture. Modern UHMWPE is highly cross-linked with higher doses of radiation (5.0-10 Mrad) in an inert environment. The level of radiation directly increases the amount of cross-linking and also improves wear resistance via cross-linking. Free radicals are still generated during the radiation process and are quenched by either remelting or annealing. Remelting is the heating of the polyethylene above its melt point, changing it from the partial crystalline state to the amorphous state and removing all free radicals, but also reducing wear characteristics. In annealing, the UHMWPE is heated below the melting point, which avoids the reduction in crystallinity but leaves more free radicals. Ethylene oxide is used in the sterilization process. Highly cross-linked polyethylene has shown significantly decreased wear rates compared to conventional polyethylene in both clinical and simulator studies. There is still some concern regarding decreased mechanical properties.

     

     

     

    2020

     

  258. What process changes polyethylene from its partial crystalline state to its amorphous state?

    1. Radiation

    2. Ethylene oxide

    3. Remelting

    4. Annealing

     

    Corrent answer: 3

     

     

     

    2020

  259. What polyethylene processing step results in increased polyethylene wear and subsequent osteolysis?

    1. Gamma irradiation in air

    2. Remelting after gamma irradiation in nitrogen

    3. Heat annealing

    4. Ethylene oxide sterilization Corrent answer: 1

    Gamma irradiation in air is expected to result in oxidized polyethylene, which wears poorly and is expected to result in osteolysis. Many techniques have been used to reduce polyethylene wear and subsequent osteolysis. Crosslinking and thermal stabilization are 2 important techniques. Remelting and annealing are thermal stabilization methods intended to reduce the number of free radicals that are present as a result of the cross-linking process. Both remelting and heat annealing have been shown to reduce wear and osteolysis. Ethylene oxide sterilization should have no effect on the wear properties of polyethylene and should not affect its potential to contribute to osteolysis.

     

     

     

    2020

     

  260. A 62-year-old man developed a pelvic mass 5 years after undergoing a metal-on-metal hip resurfacing arthroplasty (MOMHRA). A biopsy was performed and the histologic diagnosis of pseudotumor was made. What mechanism is associated with MOMHRA-related pseudotumors?

    1. Femoral head scratching from third-body debris

    2. Edge loading and associated loss of fluid-film lubrication

    3. Increased macrophage reactivity to <em>Propionibacterium acnes (P. acnes)</em>

    4. Increased lymphocyte reactivity to nickel ion Corrent answer: 2

    Several reports have documented the development of pseudotumors in patients undergoing MOMHRA. The incidence of pseudotumor is likely lower than 1% in this group of patients. Patients usually are asymptomatic. Recent investigation suggests that pseudotumors occur more often with increased wear attributable to edge loading. Edge loading most commonly results from abnormal contact stresses that occur in the setting of a malpositioned cup orientation. Lymphocyte reactivity to cobalt, chromium, and nickel did not significantly differ in patients with pseudotumors compared to patients without

    pseudotumors. Macrophage reactivity to P. acnes would be present in the setting of infection, but not present in the development of pseudotumors.

     

     

     

    2020

     

  261. What factor most significantly contributes to increased risk for pseudotumor formation in patients with metal-on-metal implants?

    1. Obesity

    2. Edge loading

    3. Female gender

    4. Age older than 65

     

    Corrent answer: 2

     

    Obesity has not been directly associated with implant wear and pseudotumor formation. Patient factors that have been associated with increased pseudotumor formation are female gender and hip dysplasia. Studies have shown that edge loading, double-heat treatment of metal implants, and a low carbon content of the bearing surfaces have all been associated with an increased incidence of pseudotumor formation and increased metal wear debris formation.

     

     

     

    2020

     

  262. A 62-year-old woman has advanced osteoarthritis of the knee that has been refractory to nonsurgical treatment. She wishes to discuss total knee arthroplasty. She reports a lifelong history of intolerance to most jewelry and is concerned about having an allergic reaction to the metallic knee implant. Hypersensitivity to metal implants is usually classified as what type of Gell-Coombs reaction?

    1. I (allergic)

    2. II (cytotoxic, antibody-dependent)

    3. III (immune complex)

    4. IV (delayed type) Corrent answer: 4

    Most "metal allergy" is classified as type IV, or delayed-type hypersensitivity response, which is a cell-mediated response. Types I, II, and III are not generally associated with metal hypersensitivity responses. Type I reactions are typically anaphylaxis. Type II reactions are antibody mediated, such as

    seen in Grave's disease or hemolytic anemia. Type III reactions are immune complex diseases such as serum sickness or systemic lupus erythematosus.

     

     

     

    2020

     

  263. How are free radicals removed from highly cross-linked polyethylene?

    1. Low-dose irradiation

    2. High-dose irradiation

    3. Oxidation

    4. Thermal processing

     

    Corrent answer: 4

     

    Cross-linking of polyethylene improves its wear property via a series of steps including irradiation, thermal processing, and sterilization. Irradiation breaks carbon-hydrogen bonds in polyethylene and creates covalent bonds during cross-linking. Both low- and high-dose irradiation create free radicals that can interact with oxygen to weaken the polyethylene. Oxidation is the process through which oxygen is introduced into the polyethylene and possibly generates free radicals. Thermal processing of irradiated polyethylene removes these free radicals. Thermal processing is the heat treatment of cross-linked polyethylene to remove free radicals. Annealing and remelting are 2 thermal processing types, with annealing being the preferred technique because it results in better mechanical properties of polyethylene. Another strategy that can remove free radicals is to add an antioxidant such as vitamin E. Sterilization is the final step before the packaging of polyethylene implants.

     

     

     

    2020

     

  264. Figure 74a is the radiograph taken after of a 54-year-old man underwent an uncomplicated metal-on-metal left total hip replacement 2 years ago. He continued to have hip pain and discomfort after undergoing surgery. Figure 74b is the MRI scan taken 2 years later. Figure 74c shows the hematoxylin and eosin (H&E) stain of the subsequent biopsy. The patient is noted to have an erythrocyte sedimentation rate (ESR) of 100 mm/h (reference range, 0-20 mm/h) and a C-reactive protein (CRP) of 20 mg/dL (reference range, 0.0-.05 mg/dL). This patient should have surgery for debridement and receive administration of

     

     

     

     

     

     

     

     

    1. radiation.

    2. chemotherapy.

    3. radiation and chemotherapy.

    4. systemic antibiotics.

     

    Corrent answer: 4

     

    The radiograph shows a fairly normal-appearing metal-on-metal implant; however, the T1-weighted MRI scan with gadolinium contrast shows a large mass in close proximity to the joint, with probable continuity of the joint with the mass. The H&E stain shows an abundant number of neutrophils and other white blood cells, suggesting acute inflammation. This is an example of an infected pseudotumor. The abundant number of acute inflammatory cells on pathology, as well as the elevated ESR and CRP, are strong indicators of infection. In this patient, the large pseudotumor needs to be appropriately debrided, as one would do to treat an abscess, and the implants need to be removed, an antibiotic spacer placed, and systemic antibiotics started.

    Radiation and/or chemotherapy are not required.

     

     

     

    2020

     

  265. Particulate wear debris is central to the mechanisms responsible for aseptic loosening. The severity of the biologic response to particulate polyethylene wear debris is directly proportional to particle

    1. size.

    2. number.

    3. composition.

    4. molecular weight.

    Corrent answer: 2

     

    The biologic response to polyethylene is highly dependent on the type and quantity of particles. The response is clearly dose-dependent. Multiple cytokines are involved in the biologic response to wear particles, and tumor necrosis factor alpha seems to be among the most important. Its secretion increases during the biologic response to particulate wear debris. Research has shown that particles sized between 0.2 µm and 7-8 µm are the most stimulatory, but a direct proportional correlation does not exist with sizes outside of this range. A lack of correlation also applies to molecular weight and composition of polyethylene debris.

     

     

     

    2020

     

  266. Several techniques have been developed to improve the wear characteristics of polyethylene. What technique results in the largest reduction of free radicals?

    1. Thermal stabilization through remelting

    2. Thermal stabilization through annealing

    3. Stabilizing the polyethylene with vitamin C

    4. Gamma irradiation with 10 Mrads in nitrogen Corrent answer: 1

    Free radicals, which are generated when polyethylene is exposed to ionizing radiation, are highly reactive and can be quenched by cross-linking with each other or via oxidation in the presence of oxygen. Thermal stabilization techniques are designed to minimize the number of remaining free radicals available to react with oxygen. Remelting is the most efficient method to reduce free radicals. Annealing also greatly reduces the number of free radicals, but cannot quench as many free radicals as remelting. Vitamin E has been shown to protect against oxidation by quenching free radicals; however, vitamin C has not shown the same benefit. Gamma irradiation with 10 Mrads will generate many free radicals regardless of the presence or absence of oxygen.

     

     

     

    2020

     

  267. Cementless knee replacements are associated with

     

    1. increased risk for revision.

    2. increased wear of the polyethylene insert.

    3. increased infection risk.

    4. significantly higher Knee Society Scores (KSS).

     

    Corrent answer: 1

     

    Many published series comparing cementless to cemented knee replacements show no difference in KSS or infection risk, but data in several registries demonstrate a significant increase in the revision rates for cementless knees. Although no difference in poly wear occurred, loosening of the tibial component was the most common cause of failure.

     

     

     

    2020

     

  268. A healthy, active 72-year-old man tripped and fell, landing on his left hip 10 weeks after an uncomplicated left primary uncemented total hip replacement. A radiograph taken 6 weeks after surgery and before the fall is shown in Figure 10a. A radiograph taken after the fall is shown in Figure 10b. He was unable to bear weight and was brought to the emergency department. Examination revealed a slightly shortened left lower extremity and some mild ecchymosis just distal to the left greater trochanteric region, but his skin was intact without abrasions or lacerations. What is the most appropriate treatment?

     

     

     

     

     

    1. Open reduction and cerclage fixation of the fracture

    2. Open reduction and revision of the femoral implant to a long cemented stem

    3. Open reduction and revision of the femoral implant to a long fluted and tapered uncemented stem

    4. Application of balanced traction and surgery after the ecchymosis has resolved

    Corrent answer: 3

     

    This patient has a periprosthetic femoral fracture with a loose femoral stem and normal femoral bone stock (Vancouver type B2). The most appropriate treatment is fixation of the fracture along with revision of the stem.

    Considering his age, bone quality, and activity level, a longer uncemented stem is most predictable. Although a cylindrical stem may also be used, the fluted stem option is the only uncemented choice listed and the most appropriate response. A cemented stem is a poorer choice because it is difficult to keep the cement out of the fracture site; this would pose risk for nonunion at the fracture, and overall poorer results have been associated with long cemented stems in healthy, active people. Surgery does not need to be delayed to allow the ecchymosis to resolve, and simple open reduction and fixation does not address the loose stem.

     

     

     

    2020

     

  269. A 68-year-old woman with a 9-year history of type II diabetes is seen 11 weeks after an uncemented left total hip replacement. When seen 6 weeks after surgery, some mild erythema and induration at the distal incision was noted, but no drainage. She states that drainage started 2 weeks ago. Examination shows turbid drainage coming from the distal third of the incision with mild surrounding erythema. Hip range of motion causes mild discomfort. Investigations reveal an erythrocyte sedimentation rate of 45 mm/h (reference range, 0-20 mm/h) and C-reactive protein of 54 mg/L (reference range, 0.08-3.1 mg/L). A rapid polymerase chain reaction of the swabbed fluid is positive for methicillin-resistant Staphylococcus aureus. Hip aspiration under fluoroscopy is attempted but no fluid is obtained. What is the most appropriate treatment?

     

    1. Debridement of the skin and superficial tissues

    2. Debridement and removal of the implants and insertion of an antibiotic spacer

    3. Debridement of superficial and deep tissues including the joint with exchange of the modular head and liner

    4. Prescription for sulfamethoxazole and trimethoprim (Bactrim DS), 1 tablet, twice daily for 14 days, and then re-evaluate the patient

    Corrent answer: 2

     

    This case illustrates the treatment choices to address a postsurgical deep infection at 11 weeks postsurgery. Considering the progression of symptoms and persistent drainage, one needs to assume the infection is deep. Wound drainage beginning at 9 weeks after surgery is unlikely to be the result of a superficial infection. The absence of fluid on the attempted aspiration may occur in situations in which a sinus tract allows most of the fluid to escape the joint. Injection of contrast could confirm the presence of a sinus tract.

    Debridement and removal of the implants and insertion of an antibiotic spacer are most appropriate because the results of a single debridement at 11 weeks with a resistant organism are poor for curing or controlling infection. The use of antibiotics alone or a superficial debridement is inadequate in this setting.

     

     

     

    2020

     

  270. A 78-year-old active woman who weighs 227 pounds has a long history of type 2 diabetes mellitus and had a total knee replacement 15 years ago. She underwent revision arthroplasty for loose components. After surgery, she fell and now has a 35-degree extensor lag with a high-riding patella on the lateral radiographic view. When attempting to surgically repair the torn extensor, gross purulence is found, leading to a resection with an antibiotic cement spacer.

Enterococcus bacteria are recovered on cultures a few days later; this species is resistant to several antibiotics. The wound drains for 3 weeks and then heals with continued redness, pain, and swelling despite intravenous antibiotics. What is the most effective long-term treatment for this patient?

  1. Knee fusion

  2. Primary extensor repair with a hinged total knee

  3. Revision total knee with extensor mechanism allograft

  4. Repeat debridement and chronic antibiotic suppression Corrent answer: 1

Arthrodesis may be the most satisfactory long-term option for this elderly, obese patient even though all knee motion will be lost. The other options presented are associated with unpredictable results and higher failure rates.

 

 

 

2020